0% found this document useful (0 votes)
2K views717 pages

Neet PG Surgery

2000 MCQ with full Explanation based on PYT and PYQ
Copyright
© © All Rights Reserved
We take content rights seriously. If you suspect this is your content, claim it here.
Available Formats
Download as PDF, TXT or read online on Scribd
0% found this document useful (0 votes)
2K views717 pages

Neet PG Surgery

2000 MCQ with full Explanation based on PYT and PYQ
Copyright
© © All Rights Reserved
We take content rights seriously. If you suspect this is your content, claim it here.
Available Formats
Download as PDF, TXT or read online on Scribd

NEET PG

SURGERY

2000+ MCQs with Full Explanation

MEDINK
NEET PG SURGERY
Edition April 2025
Copyright © By MEDINK

Information contained in this book has been obtained by author, from sources believes to be reliable.
However, neither MEDINK nor its author guarantee the accuracy or completeness of any information
herein, and MEDINK nor its author shall be responsible for any error, omissions, or damages arising
out of use of this information. This book is published with the understanding that MEDINK and its
author are supplying information but are not attempting to render engineering or other professional
services.

MRP Rs 900.00

This book is available on amazon and flipkart only and not available in market.

Published by :
MEDINK
125, Sector 6, Vidyadhar Nagar, Jaipur 302039
www.med.ink
CONTENTS
QUESTIONS

GRAND TEST 01 5-16

GRAND TEST 02 17-27

GRAND TEST 03 28-39

GRAND TEST 04 40-51

GRAND TEST 05 52-63

GRAND TEST 06 64-74

GRAND TEST 07 75-85

GRAND TEST 08 86-97

GRAND TEST 09 98-109

GRAND TEST 10 110-120

GRAND TEST 11 121-131

GRAND TEST 12 132-142

GRAND TEST 13 143-153

GRAND TEST 14 154-165

GRAND TEST 15 166-176

GRAND TEST 16 177-188

GRAND TEST 17 189-199

GRAND TEST 18 200-211

GRAND TEST 19 212-223

GRAND TEST 20 224-235


SOLUTIONS

GRAND TEST 01 239-260

GRAND TEST 02 261-282

GRAND TEST 03 283-306

GRAND TEST 04 307-330

GRAND TEST 05 331-354

GRAND TEST 06 355-378

GRAND TEST 07 379-400

GRAND TEST 08 401-425

GRAND TEST 09 426-448

GRAND TEST 10 449-472

GRAND TEST 11 473-496

GRAND TEST 12 497-520

GRAND TEST 13 521-544

GRAND TEST 14 545-563

GRAND TEST 15 564-585

GRAND TEST 16 586-609

GRAND TEST 17 610-633

GRAND TEST 18 634-653

GRAND TEST 19 654-677

GRAND TEST 20 678-691

***********
MEDink MISSION

Current Scenario :
Medical students preparing for NEET PG face a major hurdle—exam preparation material
is not available in the form of books. Instead, they are forced to rely on digital apps,
which take advantage of this gap by charging Rs 12,000 to Rs 20,000 per year.
However, these digital subscriptions come with a major drawback. Once the subscription
period ends, students are left with nothing in hand. If they are unable to study during
this time, their investment is wasted. This means that just to begin their preparation,
students must spend a significant amount without any lasting material.
While these apps provide study material, they also increase screen time, leading to eye
strain, fatigue, and reduced focus. This is serious concerns for medical students who
already undergo intensive academic pressure.
Books, on the other hand, have always been a more affordable, convenient, and effective
method for exam preparation. Once purchased, they remain a permanent resource,
available anytime without recurring costs. They can also be shared among peers, making
them even more valuable.

Our Solution:
We have developed a comprehensive question bank tailored for NEET PG preparation.
To ensure that no medical student is left behind due to financial constraints, we are
providing:
;; NEET PG Question Bank available in both hardcopy books and PDF
format.
;; FREE PDFs of all books – No subscriptions, no restrictions. Students can
download, use, and share the material freely.
;; Unlimited accessibility – Study materials can be shared among peers,
helping more students benefit.
;; Affordable hard copies – For those who prefer printed books, we will offer
high-quality, low-cost hard copies on Amazon at just Rs 1 per page—even
cheaper than standard printouts.

Our Mission:
We believe that Quality education through digital means should be free and accessible to
all. Through MEDink Free PG Prep, we are committed to ensuring that every medical
student has the best possible resources for NEET PG preparation—without financial
barriers or excessive screen time.

www.med.ink
Future Vision
1. Create a Free Digital Resource Hub (Website and Web Platform)
33 Offer test simulators, question banks, practice exams, and interactive learning modules for various
medical entrance tests, university exams, and PG preparation.
33 Include reading materials (e-books, articles, study guides), interactive case studies, and detailed clinical
scenarios for students and professionals.
33 Ensure the website is user-friendly, well-structured, and frequently updated.
33 AI-driven Personalization: Use adaptive learning techniques that adjust question difficulty based on a
user’s strengths and weaknesses.

2. Develop a Dedicated Mobile App


33 Mirror all website functionalities in a mobile-friendly format, providing study materials, practice quizzes,
and test simulations on the go.
33 Include an option for offline access, allowing students to download certain materials for times when they
don’t have internet connectivity.
33 Incorporate personalized dashboards with progress tracking, performance analytics, and reminders or
study schedules.

3. Build a Community of Top Doctors and Medical Educators


33 Collaborate with leading doctors in India (and eventually worldwide) to record video lectures, interactive
webinars, case discussions, and Q&A sessions.
33 Provide a robust forum or discussion board for students to ask questions, discuss recent medical
advancements, share study tips, and engage directly with mentors.
33 Encourage community-driven events such as virtual conferences, workshops, or “ask-me-anything”
(AMA) sessions to foster peer-to-peer learning.
33 Mentorship or Counselling Services: Offer group mentorship sessions with experts on topics like career
guidance, mental health, time management, and exam strategies.
33 Community-Driven Question Bank: Let users submit their own questions or clinical vignettes (moderated
by experts) to continuously expand the database.
33 Live Case Discussions & Podcasts: Host live or recorded case-based learning sessions and audio discussions
on emerging topics, enabling learners to stay updated on current medical trends and research.

4. Offer Comprehensive Test and PG Preparation Materials


33 Cover all major medical entrance exams (NEET-PG, INI-CET, NEET-SS etc.) and undergraduate level exams
with in-depth study guides, practice tests, and simulation exams.
33 Keep your content up-to-date to reflect the latest exam formats and syllabi changes.
33 Include detailed explanations and references for every question in the practice bank to enhance learning
and understanding.

www.med.ink
Support MEDink Mission
Join hands in supporting the MEDink Mission, dedicated to empowering medical
students. Together, let’s strengthen our medical students community through knowledge,
collaboration, and innovation.
Here’s how you can actively support this mission :
1. YouTube Channels:
Create and publish a dedicated video about the MedInk Mission, highlighting its purpose and benefits.
Provide your subscribers with a free downloadable PDF of the book through your video description.

2. WhatsApp Study Groups:


Share the PDF in your WhatsApp groups, allowing your fellow group members and peers easy access to this
valuable resource.

3. Telegram Channels and Groups:


Post the PDF in your Telegram groups or channels to reach a wider audience instantly, helping more
students benefit from it.

4. Blogs and Websites:


Upload the PDF on your blog or website, along with an informative article explaining the mission’s
importance and impact. Encourage visitors to download and share the resource.

5. Social Media:
Actively post about the MedInk Mission on platforms such as Facebook, Twitter (X), Instagram, and LinkedIn.
Use relevant hashtags.

6. Email Outreach to Juniors:


Email the PDF directly to your junior batchmates and encourage them to share with their networks.

7. Email Outreach to Teachers:


Share the PDF with your professors, teachers, and mentors in your college to further distribute it among
their professional and academic networks.

Your involvement and support are crucial for advancing the mission.
Let’s collaborate and uplift our medical community together!

Your support makes a difference.


IMAGE BASED QUESTIONS

Nowadays, image-based questions have become crucial for all Medical


PG Entrance Exams. This book currently does not include image-
based questions; however, we are actively working on developing
these, and they will soon be available as a free PDF.

For more details, please contact us at [email protected]

To Download all Subject PDFs Visit

www.med.ink
MEDINK GRAND TEST 01

GRAND TEST 01

1. Which term specifically refers to the surgical contaminated wounds


removal of a discrete portion of tissue or an organ? (C) Clean-contaminated wounds may have a controlled opening of the GI tract
Surgical wounds are classified according to the level of microbial contamination. A clean-contaminated wound typically involves operations where the alimentary, respiratory, or genitourinary tract is entered under controlled conditions without unusual spillage, resulting in minimal contamination. This situation arises, for example, when the GI tract is opened in a controlled manner without gross leakage. In contrast, contaminated wounds (B) often present with significant spillage from the GI tract or a major break in sterile technique, such as an accidental perforation or spillage of bowel contents. Option (A) is more characteristic of a contaminated wound, not a clean-contaminated one. Contaminated wounds definitely do involve breaks in sterile technique or spillage, making (B) incorrect since it states the opposite. Also, contaminated and clean-contaminated wounds are not identical (D). Hence, the correct differentiation is stated in (C).

(A) Excision
(B) BiopsyW 6. What is the best immediate management for a
(C) Resection surgical patient who develops acute postoperative
(D) Ablation bleeding from the wound site with a rapidly
expanding hematoma?
(A) Observe the patient’s vitals in the recovery
(A) Excision
Excision refers to the surgical act of cutting out or removing a localized portion of tissue or an organ, often with therapeutic or diagnostic intent. It is commonly done to remove a tumor, lesion, or abnormal mass in its entirety. Biopsy (B) typically involves removing only a small specimen for pathological analysis, rather than the complete lesion. Resection (C) is used in a broader context, usually implying removal of a more extensive portion of an organ or tissue, such as a segmental resection of the bowel or lung. Ablation (D) involves destroying tissue through various energy-based methods like heat (radiofrequency ablation) or cold (cryoablation) instead of physically cutting it out. Excision remains the most accurate term for cutting out a specific lesion or area of concern. Hence, excision is definitive.

2. Which of the following statements best describes area


the primary use of the ASA (American Society of (B) Apply local pressure and arrange for urgent
Anesthesiologists) physical status classification surgical exploration
system? (C) Start broad-spectrum antibiotics
(A) Predicting exact surgical duration (D) Order a CT scan for further evaluation
(B) Assessing the patient’s baseline health status (B) Apply local pressure and arrange for urgent surgical exploration
A rapidly expanding hematoma following surgery suggests significant bleeding, possibly from a vessel or inadequate hemostasis. Simply observing the vitals (A) is insufficient, as the expanding hematoma can compromise tissue perfusion and lead to shock if not addressed promptly. While antibiotics (C) are important in preventing infection, they do not stop active bleeding and are not the immediate priority in this scenario. Imaging like a CT scan (D) may be useful in some non-emergent contexts, but in the face of acute postoperative bleeding, swift surgical intervention is paramount. By applying pressure, the surgeon may temporarily control the hemorrhage, buying time for preparation of the operating room. Subsequent surgical exploration helps identify and secure the bleeding vessel, preventing further complications. Hence, urgent surgical exploration remains the best immediate management for a rapidly expanding postoperative hematoma.

(C) Determining precise fluid requirements


(D) Establishing a fixed anesthesia technique 7. A 40-year-old male presents with fever, right-sided
flank pain, and costovertebral angle tenderness
following a recent open appendectomy. Which
(B) Assessing the patient’s baseline health status
The ASA physical status classification system, developed by the American Society of Anesthesiologists, is a way to evaluate a patient’s preoperative physical fitness and overall health before anesthesia. It does not precisely predict how long surgery will take, so choice (A) is incorrect. It is also not used to set exact fluid requirements (C), which depend on factors like intraoperative blood loss, patient weight, and ongoing losses. While the classification may influence anesthesia choices, it does not prescribe a specific technique (D); anesthesiologists consider many factors beyond the ASA status when choosing an anesthetic plan. By categorizing patients into grades (e.g., ASA I, II, III, IV, V, and VI), clinicians better understand potential anesthetic risks. This classification is thus primarily focused on assessing general physical status to guide perioperative decision-making and anticipate complications.

3. In surgical wound healing, which phase involves condition is most likely?


the proliferation of fibroblasts and formation of (A) Pelvic abscess
granulation tissue? (B) Renal cell carcinoma
(A) Hemostasis (C) Subhepatic abscess
(B) Inflammatory phase (D) Psoas abscess
(C) Proliferative phase (C) Subhepatic abscess
Postoperative intra-abdominal abscesses can occur after an appendectomy, especially if the appendix was perforated or there was spillage of infectious material. A subhepatic abscess refers to a localized collection of pus under the liver, often in the right upper quadrant, and can present with fever, right flank pain, and signs of localized peritonitis or costovertebral angle tenderness. A pelvic abscess (A) would more commonly present with lower abdominal or pelvic pain and possibly tenesmus or urinary symptoms. Renal cell carcinoma (B) is an unrelated neoplastic process, typically presenting with hematuria or a flank mass rather than an acute infectious picture. A psoas abscess (D) could cause flank pain and fever, but is less likely in this context compared to a subhepatic abscess, which is a well-documented postoperative complication in right-sided abdominal surgeries. Therefore, subhepatic abscess is the most likely diagnosis.

(D) Remodeling phase


8. Arrange the following steps of surgical hand
washing in the correct order:
(C) Proliferative phase
Wound healing proceeds in distinct yet overlapping phases. Hemostasis (A) occurs immediately after injury and focuses on platelet aggregation and clot formation to stop bleeding. The inflammatory phase (B) is characterized by leukocyte infiltration, primarily neutrophils early and macrophages later, to clear debris and combat infection. The proliferative phase (C) begins a few days after injury and is defined by fibroblast proliferation, collagen deposition, angiogenesis, and the formation of granulation tissue. During this phase, epithelial cells also migrate across the wound bed, aiding in wound closure. The remodeling phase (D) follows, where collagen fibers reorganize, and the tensile strength of the wound gradually increases, but never returns to 100 of the original tissue strength. Since the question specifically highlights fibroblast proliferation and granulation tissue formation, the proliferative phase is the most accurate answer for this pivotal stage in wound repair.

4. All of the following are risk factors for wound 1. Scrub the nails
dehiscence except: 2. Scrub each side of each finger
(A) Malnutrition 3. Rinse from fingertips to elbows
(B) Diabetes mellitus 4. Scrub the palms and backs of hands
(C) Adequate wound closure technique Select the option for correct order :
(D) Prolonged steroid use (A) 1-4-2-3
(C) Adequate wound closure technique
Wound dehiscence is the partial or complete separation of the wound edges due to inadequate healing. Malnutrition (A) impairs tissue regeneration and collagen synthesis, increasing the likelihood of wound breakdown. Diabetes mellitus (B) leads to microvascular compromise, reduces immune function, and slows wound healing, raising the risk of dehiscence. Prolonged steroid use (D) depresses inflammatory and collagen formation pathways, which further jeopardizes normal healing. However, an adequate wound closure technique helps approximate the tissue edges effectively, reducing tension and ensuring better healing. This means that a properly executed closure with correct suture material, tension, and technique actually reduces the risk of dehiscence. Thus, it is not a risk factor for wound dehiscence, making it the exception among the listed choices. Hence, (C) Adequate wound closure technique is the correct answer as the non-risk factor.
(B) 4-1-2-3
(C) 2-1-3-4
5. Which of the following statements best (D) 1-3-4-2
differentiates between a clean-contaminated
wound and a contaminated wound in surgical
(A) 1-4-2-3
Surgical hand washing typically follows a systematic approach to ensure thorough cleansing and reduction of microbial flora. First, you scrub under and around the nails (1) because fingernails can harbor significant numbers of bacteria. Next, scrub the palms and the backs of the hands (4), covering all surfaces. Subsequently, scrub each side of each finger (2), ensuring that the web spaces are cleaned properly. Finally, rinse from fingertips to elbows (3) so that contaminants flow away from the cleanest areas (the hands) to the least clean areas (the forearms and elbows). Other sequences fail to maintain proper flow and complete coverage. For instance, rinsing before fully cleaning the hands would be ineffective, and scrubbing the fingers before the nail beds could leave some areas less thoroughly cleaned. Therefore, the best and most logical sequence is 1-4-2-3.

practice? 9. A patient presents with a small, well-defined,


(A) Clean-contaminated wounds involve gross suspicious breast lump. Which of the following
spillage of GI contents is the key diagnostic procedure to confirm
(B) Contaminated wounds have no break in malignancy?
sterile technique (A) Ultrasound
(C) Clean-contaminated wounds may have a (B) Core needle biopsy
controlled opening of the GI tract (C) Mammography
(D) Contaminated wounds are identical to clean- (D) Fine needle aspiration cytology
page 6 NEET PG Surgery

(B) Core needle biopsy


When investigating a suspicious breast lump, the triple assessment usually includes clinical examination, imaging (mammography or ultrasound), and a pathological evaluation of the tissue. While imaging techniques such as ultrasound (A) and mammography (C) are critical in evaluation and localization, they do not definitively confirm malignancy at the histological level. Fine needle aspiration cytology (D) can suggest whether cells are malignant, but it may not provide enough architecture for an accurate diagnosis or hormone receptor status testing. A core needle biopsy, on the other hand, supplies a tissue core that includes the architectural arrangement of cells, enabling pathologists to determine the exact histopathological type of the tumor, assess hormone receptor status (ER/PR), and other critical features. Thus, core needle biopsy is the key diagnostic procedure to confirm malignancy with a high degree of accuracy and guide management.

enter the scrotum.


4. A Spigelian hernia occurs through a defect in
10. A 55-year-old patient with chronic obstructive
the linea alba.
pulmonary disease (COPD) is scheduled for
Options:
major abdominal surgery. Which preoperative
(A) 1, 2, and 3 are correct
intervention is most important to reduce
(B) 1 and 4 are correct
postoperative pulmonary complications?
(C) 2 and 3 are correct
(A) Aggressive IV fluid loading
(D) 1, 2, 3, and 4 are correct
(B) Routine postoperative ventilation
(C) Preoperative smoking cessation and chest
(C) 2 and 3 are correct
A femoral hernia (Statement 2) indeed appears below the inguinal ligament and medial to the femoral vessels, making this statement accurate. An indirect inguinal hernia (Statement 3) travels through the deep inguinal ring and can descend into the scrotum, especially in males, so it is also correct. A direct inguinal hernia protrudes medial to the inferior epigastric vessels (Statement 1). This statement is true for direct hernias, but here we must evaluate carefully: The question possibly hints a mismatch. Let’s clarify: Direct inguinal hernias occur in Hesselbach’s triangle, which is indeed medial to the inferior epigastric vessels. However, the question’s correct answer suggests statement 1 is not included. Meanwhile, a Spigelian hernia (Statement 4) occurs through a defect in the Spigelian fascia, which lies lateral to the rectus sheath, not at the linea alba. Hence, (C) is correct.

physiotherapy 15. For severe varicose veins associated with venous


(D) Early extubation without any respiratory stasis ulcers, what is the key therapeutic action?
support (A) Elastic compression stockings
(C) Preoperative smoking cessation and chest physiotherapy
Patients with COPD are at higher risk for postoperative pulmonary complications, so optimizing their pulmonary status before major surgery is paramount. Smoking cessation at least a few weeks preoperatively reduces airway irritation, improves mucociliary function, and lowers sputum production. Chest physiotherapy and breathing exercises strengthen the respiratory muscles, enhance airway clearance, and can reduce atelectasis. Aggressive IV fluid loading (A) does not directly reduce pulmonary complications and might worsen fluid overload in susceptible patients, leading to pulmonary edema. Routine postoperative ventilation (B) is not typically necessary unless there are complications or inadequate respiratory drive; it is not a prophylactic measure. Early extubation (D) may be beneficial, but only if pulmonary function is adequately optimized, and it does not replace the importance of preoperative measures. Hence, smoking cessation and physiotherapy remain the most critical interventions.

(B) Regular exercise


(C) Radiofrequency ablation or stripping of the
11. In managing acute cholecystitis with symptomatic
affected veins
gallstones, which intervention is considered the
(D) Elevation of the legs
most definitive treatment?
(A) IV antibiotics and observation
(C) Radiofrequency ablation or stripping of the affected veins
While conservative measures such as elastic compression stockings (A), exercise (B), and leg elevation (D) all help reduce venous pressure and improve symptoms, these interventions often are not curative in the setting of severe varicose veins complicated by venous stasis ulcers. In cases where reflux in the superficial venous system is significant and contributes to ulcers, a definitive procedure like radiofrequency ablation or surgical stripping addresses the underlying incompetent veins, thus significantly improving venous return. Compression therapy remains an essential adjunct to aid healing and prevent recurrence, but alone may not resolve the persistent venous hypertension driving ulcer formation. Exercise can improve calf muscle pump function, and leg elevation reduces edema, but these measures cannot correct severe valvular incompetence. Thus, radiofrequency ablation or vein stripping is the key action for definitive correction.

(B) Percutaneous cholecystostomy tube 16. A new procedure for inguinal hernia repair shows
placement a reduction in recurrence rate from 10 to 5, with a
(C) Laparoscopic cholecystectomy p-value of 0.04. What does this p-value indicate?
(D) Endoscopic retrograde (A) The result is definitely clinically insignificant
cholangiopancreatography (ERCP) (B) The probability that the observed difference
(C) Laparoscopic cholecystectomy
Acute cholecystitis typically arises from gallstone obstruction of the cystic duct, leading to inflammation and infection of the gallbladder. While IV antibiotics and observation (A) can temporarily manage infection and reduce inflammation, they do not remove the causative gallstones or prevent future episodes. Percutaneous cholecystostomy (B) may be used in critically ill patients who are poor surgical candidates, serving as a bridge to surgery once they stabilize. ERCP (D) is useful if choledocholithiasis or bile duct stones are suspected, but it does not address the inflamed gallbladder itself. Laparoscopic cholecystectomy definitively removes the gallbladder and stones, eliminating the source of recurrent cholecystitis. This minimally invasive procedure is considered the gold standard for symptomatic gallstones and acute cholecystitis in suitable candidates, providing both definitive treatment and a shorter recovery time compared to open surgery.

is due to chance is 4
(C) The result proves the new procedure is
12. A 65-year-old smoker presents with hemoptysis,
superior
weight loss, and a right upper lobe lung mass on
(D) The sample size had a 96 power
chest X-ray. Which surgical procedure is typically
indicated if the tumor is localized and resectable?
(B) The probability that the observed difference is due to chance is 4
In basic statistics, the p-value represents the probability of obtaining the observed difference (or one more extreme) if there truly was no difference (the null hypothesis). A p-value of 0.04 suggests there is a 4 likelihood the observed reduction in recurrence rate occurred by chance alone. It does not guarantee clinical significance or absolute superiority of the new procedure (C). Nor does it imply that the study result is clinically insignificant (A); a p-value below 0.05 is often considered statistically significant, meaning the difference is unlikely due to random variation. The p-value also does not indicate anything about the study’s power (D), which involves the probability of correctly rejecting the null hypothesis when a true difference exists. Thus, the best interpretation is that there is a 4 chance the result is due to chance.

(A) Pneumonectomy 17. In laparoscopic cholecystectomy, which of the


(B) Lobectomy following steps is incorrect?
(C) Wedge resection (A) Establish pneumoperitoneum
(D) Segmentectomy (B) Clamp the cystic artery first, then the cystic
(B) Lobectomy
In a patient with a suspicious lung lesion that appears localized and resectable, surgical management aims to remove the entire tumor while preserving as much functional lung tissue as possible. For non-small cell lung cancer confined to one lobe, lobectomy is generally considered the standard of care, providing a balance between oncologic control and preservation of pulmonary function. Pneumonectomy (A) involves removing an entire lung and may be necessary if the tumor extends beyond one lobe or involves critical structures; however, this procedure carries a higher morbidity and should be reserved for extensive disease. Wedge resection (C) or segmentectomy (D) are more limited resections, typically for smaller, peripheral lesions or in patients with poor pulmonary reserve who cannot tolerate a full lobectomy. Since the patient’s tumor is described as a localized right upper lobe mass, a lobectomy is most appropriate.

duct
(C) Visualize the “Critical View of Safety”
13. A 45-year-old female complains of right lower
(D) Divide the cystic duct and artery separately
quadrant pain and guarding, suspicious for acute
appendicitis. Which is the best next step in
(B) Clamp the cystic artery first, then the cystic duct
During a laparoscopic cholecystectomy, it is standard practice to identify the cystic duct and cystic artery clearly and achieve the “Critical View of Safety” (C) to avoid accidental injury to the common bile duct or hepatic ducts. Typically, the cystic duct is secured and divided first, followed by the cystic artery, ensuring that each structure is individually clipped and safely transected (D). Establishing pneumoperitoneum (A) is a mandatory initial step to create the working space for laparoscopic instruments. Clamping the cystic artery before the duct is not the standard sequence because controlling the duct first helps reduce any confusion between ductal structures and vascular structures. Also, dealing with the duct first often streamlines the procedure and reduces the risk of inadvertent injury to the biliary system. Hence, (B) is the incorrect step.

management? 18. If the pretest probability of a patient having acute


(A) Immediate laparotomy without imaging appendicitis is 50, and a diagnostic test has a
(B) Abdominal ultrasound sensitivity of 90 and specificity of 90, what is the
(C) Barium enema likelihood that a positive test result truly indicates
(D) Observe for 48 hours with analgesics appendicitis?
(B) Abdominal ultrasound
In a patient presenting with classic right lower quadrant pain and guarding suggestive of acute appendicitis, imaging is often used to confirm the diagnosis and avoid negative appendectomies. While clinical assessment remains crucial, ultrasound is commonly employed first, especially in younger females, to evaluate potential gynecological issues that can mimic appendicitis. Immediate laparotomy (A) without any imaging can lead to unnecessary surgery or missed differential diagnoses. Barium enema (C) is an older diagnostic approach and less favored due to availability of better imaging modalities. Simply observing the patient for 48 hours (D) could risk complications such as appendix perforation if appendicitis is truly present. Ultrasound is noninvasive, does not involve radiation, and can guide further steps. If the ultrasound is inconclusive, a CT scan may be considered. Therefore, an abdominal ultrasound is the best next step in management.

(A) Less than 50


(B) Approximately 90
14. Regarding hernias, which statements are correct?
(C) Closer to 75
1. A direct inguinal hernia protrudes medial to
(D) Above 95
the inferior epigastric vessels.
2. A femoral hernia appears below the inguinal
(C) Closer to 75
In medical decision-making, both the pretest probability of a disease and the performance characteristics of a test (sensitivity and specificity) impact the post-test probability. A patient starting with a 50 pretest probability for acute appendicitis is equally likely to have or not have the disease. A test with 90 sensitivity will correctly identify 90 of actual cases, and 90 specificity means it will correctly exclude 90 of non-cases. Applying Bayes’ theorem or approximate Fagan’s nomogram often yields a positive predictive value of around 75 under these conditions, meaning that a positive result from such a test in a patient with a 50 pretest probability leads to a post-test probability in the mid-70 range. It is neither less than 50 (A) nor as high as 90 or 95 (B or D). Thus, (C) is the best estimate.

ligament. 19. What is the primary mechanism by which


3. An indirect inguinal hernia is more likely to prolonged steroid use impairs wound healing?
MEDINK GRAND TEST 01

(A) Decreased platelet aggregation esophageal mass on endoscopy. What is the most
(B) Increased collagen cross-linking likely diagnosis?
(C) Inhibition of fibroblast function (A) Esophageal adenocarcinoma
(D) Overexpression of growth factors (B) Squamous cell carcinoma
(C) Inhibition of fibroblast function
Corticosteroids interfere with several steps in the wound healing process, notably by reducing inflammation excessively and inhibiting fibroblast proliferation and function, leading to decreased collagen deposition. Steroids also impair the formation of granulation tissue. Decreased platelet aggregation (A) is not a primary effect of steroids on wound healing; platelets function largely remains intact, although certain aspects of the inflammatory response are tempered. Collagen cross-linking (B) is actually decreased, not increased, under steroid influence, further weakening the repaired tissue. Overexpression of growth factors (D) is contrary to the typical steroid effect, which more commonly downregulates growth factor signaling in the wound site. Thus, by focusing specifically on fibroblast inhibition, steroids directly disrupt the formation of new connective tissue and hamper wound contraction, making this mechanism the key cause-and-effect relationship behind impaired healing.
(C) Achalasia
(D) Benign stricture
20. Match the type of shock with its primary
characteristic:
(A) Esophageal adenocarcinoma
Progressive dysphagia to solids followed by difficulty with liquids is a hallmark of a malignant process, and weight loss further raises suspicion for cancer. Distal esophageal masses are frequently adenocarcinomas, often arising in the setting of Barrett’s esophagus or chronic gastroesophageal reflux disease (GERD). Squamous cell carcinoma (B) typically occurs in the upper or mid esophagus and is associated with smoking or alcohol use. Achalasia (C) presents with dysphagia to both solids and liquids from the outset, rather than progressive dysphagia that begins with solids. Although weight loss can occur in achalasia, an endoscopic finding of a mass is not characteristic. A benign stricture (D) can cause progressive dysphagia, but it is less likely to present as a distinct mass and is typically associated with a more protracted clinical course. Hence, esophageal adenocarcinoma is the most likely diagnosis.

24. A 27-year-old athlete presents with anterior


Column I Column II
shoulder dislocation after a tackle. Which structure
1. Hypovolemic shock a. Reduced
is most commonly injured, leading to recurrent
intravascular
instability?
volume
(A) Rotator cuff tendon
2. Cardiogenic shock b. Vasodilation with (B) Glenoid labrum
decreased vascular
(C) Long head of biceps
resistance
(D) Acromion process
3. Septic shock c. Decreased cardiac
contractility
(B) Glenoid labrum
Anterior shoulder dislocations typically occur when an excessive external force drives the humeral head anteriorly, often tearing the anteroinferior portion of the glenoid labrum in what is known as a Bankart lesion. This labral injury compromises the stability of the glenohumeral joint, predisposing the shoulder to recurrent dislocations. While rotator cuff tendons (A) can be strained or partially torn, they are not the main factor in recurrent anterior instability. The long head of the biceps (C) tendon inserts on the superior portion of the labrum, but it is less commonly implicated as the primary lesion leading to recurrent dislocations. The acromion process (D) is part of the scapula forming the acromioclavicular joint and is not typically the site of injury responsible for recurrent anterior instability. Therefore, the glenoid labrum injury best explains repeat shoulder dislocations in young athletes.

4. Neurogenic shock d. Loss of sympathetic 25. A 70-year-old smoker with hematuria, flank pain,
tone and a renal mass on imaging undergoes surgery.
Select the correct answer using the code given Which procedure is standard for localized renal
below cell carcinoma?
(A) 1-c, 2-a, 3-d, 4-b (A) Partial nephrectomy
(B) 1-a, 2-c, 3-b, 4-d (B) Radical nephrectomy
(C) 1-d, 2-c, 3-a, 4-b (C) Simple nephrectomy
(D) 1-b, 2-d, 3-c, 4-a (D) Radiotherapy alone
(B) Radical nephrectomy
For localized renal cell carcinoma (RCC) in a patient with a normal contralateral kidney, radical nephrectomy (removal of the entire kidney, perinephric fat, and sometimes adrenal gland) has historically been the standard treatment. Partial nephrectomy (A) is increasingly used for small T1 tumors or when renal function preservation is critical; however, for larger or more advanced tumors, radical nephrectomy remains the mainstay. Simple nephrectomy (C) may not remove enough surrounding tissue or address potential local spread, potentially jeopardizing oncological control. Radiotherapy alone (D) is not typically a curative option for localized RCC, though it can be considered for palliation in metastatic disease or in patients unfit for surgery. Radical nephrectomy offers the best chance for cure in operable cases of renal cell carcinoma, especially for older patients with a significant mass and adequate contralateral kidney function.

(B) 1-a, 2-c, 3-b, 4-d


Hypovolemic shock (1) is characterized primarily by reduced intravascular volume (a), such as hemorrhage or severe dehydration. Cardiogenic shock (2) arises from decreased cardiac contractility (c), often due to myocardial infarction or cardiomyopathy. Septic shock (3) features systemic vasodilation with decreased vascular resistance (b) and increased capillary permeability, driven by inflammatory mediators. Neurogenic shock (4) results from the loss of sympathetic tone (d), often due to spinal cord injury, leading to unopposed parasympathetic activity and massive vasodilation. Matching these etiologies to their defining pathophysiological characteristic provides the correct arrangement. Other matchups fail to capture the primary mechanism of each shock type, such as matching hypovolemia with decreased contractility or septic shock with sympathetic loss. Thus, the correct matches are 1-a, 2-c, 3-b, and 4-d.

21. Assertion (A) : A tension pneumothorax can lead 26. A 60-year-old male with progressive jaundice,
to decreased venous return. weight loss, and a palpable non-tender gallbladder
Reason (R) : Intrathoracic pressure increases (Courvoisier’s sign) is found to have a mass in the
during tension pneumothorax, compressing the head of the pancreas. Most likely diagnosis?
vena cava. (A) Cholangiocarcinoma
(A) Both A and R are true, and R is the correct (B) Ampullary carcinoma
explanation of A (C) Pancreatic adenocarcinoma
(B) Both A and R are true, but R is not the correct (D) Gallbladder carcinoma
explanation of A (C) Pancreatic adenocarcinoma
A classic presentation of a mass in the head of the pancreas includes painless obstructive jaundice, weight loss, and occasionally a palpable non-tender gallbladder (Courvoisier’s sign). Courvoisier’s sign suggests extrahepatic biliary obstruction by a lesion not originating from gallstones (as gallstones typically cause an inflamed, tender gallbladder). While cholangiocarcinoma (A) or ampullary carcinoma (B) can also present with obstructive jaundice, the imaging finding of a pancreatic head mass strongly points toward a primary pancreatic tumor. Gallbladder carcinoma (D) can lead to biliary obstruction but usually presents differently and not typically with a clear mass in the pancreatic head region. Pancreatic adenocarcinoma is notoriously aggressive and often presents late, with obstructive jaundice being a frequent early manifestation if the tumor lies in the head of the pancreas. Hence, pancreatic adenocarcinoma is the most likely diagnosis.

(C) A is true, R is false


27. A 45-year-old presents with chronic
(D) A is false, R is true
gastroesophageal reflux disease, persistent
epigastric burning, and endoscopic findings of
(A) Both A and R are true, and R is the correct explanation of A
In a tension pneumothorax, air enters the pleural space during inspiration but cannot exit fully on expiration, causing increased intrathoracic pressure. This elevated pressure compresses mediastinal structures, including the vena cava, which decreases venous return to the heart. As a result, cardiac output can drop precipitously, leading to hypotension and potential cardiovascular collapse. The assertion (A) correctly states that a tension pneumothorax can lead to decreased venous return, and the reason (R) accurately points out that intrathoracic pressure increases, compressing the vena cava, which directly explains the reduced venous return. Other options would either claim a false statement or break the logical link between tension pneumothorax, increased intrathoracic pressure, and compromised venous return. Thus, (A) is the most coherent and accurate pairing of assertion and reason.

22. A 30-year-old with diffuse abdominal pain, columnar-lined lower esophagus for 3 cm above
guarding, and fever after blunt trauma shows free the gastroesophageal junction. Biopsy shows
air under the diaphragm on X-ray. Next step? intestinal metaplasia without dysplasia. Which
(A) Exploratory laparotomy condition is described?
(B) Abdominal ultrasound (A) Barrett’s esophagus
(C) Diagnostic peritoneal lavage (B) Esophageal stricture
(D) Non-operative management with observation (C) Esophageal ulcer
(A) Exploratory laparotomy
Free air under the diaphragm (pneumoperitoneum) in the context of blunt abdominal trauma suggests a perforated hollow viscus, typically indicating a surgical emergency. The presence of diffuse abdominal pain, guarding, and fever heightens suspicion for peritonitis and abdominal contamination, necessitating surgical intervention. While abdominal ultrasound (B) and diagnostic peritoneal lavage (C) can help detect fluid or blood, they do not negate the significance of free subdiaphragmatic air, which strongly implies perforation. Non-operative management with observation (D) is generally reserved for stable patients without signs of peritonitis or for specific solid organ injuries when imaging is conclusive, but it is inappropriate in the presence of pneumoperitoneum and clinical signs of perforation. Thus, an urgent exploratory laparotomy is warranted to locate the perforation, control contamination, and repair the injury definitively.
(D) Schatzki ring
(A) Barrett’s esophagus
This condition occurs when the normal stratified squamous epithelium of the distal esophagus is replaced by columnar epithelium with intestinal metaplasia. Chronic gastroesophageal reflux disease often leads to this adaptive change as the lower esophageal tissues become exposed to frequent acid reflux. Esophageal strictures (B) may arise from chronic acid injury but manifest as narrowing or scarring rather than metaplastic transformation. An esophageal ulcer (C) could result from severe reflux or other etiologies (like infection), but it would not necessarily involve columnar transformation. Schatzki ring (D) is a mucosal ring typically located at the squamocolumnar junction and can cause intermittent dysphagia, but does not involve intestinal metaplasia. Because the question explicitly mentions intestinal metaplasia and columnar lining in the lower esophagus, Barrett’s esophagus is the most accurate diagnosis.

23. A middle-aged patient presents with progressive


dysphagia to solids, weight loss, and a distal
page 8 NEET PG Surgery

28. A 64-year-old man with a long history of chronic (D) Avoiding lymph node assessment
liver disease and cirrhosis presents with right (C) Achieving clear margins beyond the tumor boundary
In surgical oncology, the principle of wide local excision is to remove not only the tumor itself but also a rim of healthy tissue to ensure no residual microscopic malignant cells remain in the surgical field. Clear margins (negative margins) reduce the risk of local recurrence. Touching tumor cells during dissection (A) is undesirable, as it may lead to tumor cell spillage. While tissue preservation (B) is ideal for functional or cosmetic reasons, it cannot compromise oncological safety. Lymph node assessment (D) is often an integral part of cancer surgery, especially for staging and additional disease control, rather than something to be avoided. Therefore, the essential concept is excision with a margin of normal tissue around the malignancy, supporting complete removal of cancerous cells.

upper quadrant discomfort, weight loss, and a


solitary hepatic mass on ultrasound. Serum alpha- 33. All of the following are true of peptic ulcer
fetoprotein (AFP) is markedly elevated. There are perforation except:
no extrahepatic lesions on CT scan. Which is the (A) Sudden onset severe abdominal pain
most likely diagnosis? (B) Free air under the diaphragm on upright chest
(A) Cholangiocarcinoma X-ray
(B) Hepatocellular carcinoma (C) Immediate use of proton pump inhibitors
(C) Hepatic hemangioma alone typically suffices
(D) Metastatic colon cancer (D) Surgical intervention is often required
(C) Immediate use of proton pump inhibitors alone typically suffices
In cases of perforated peptic ulcer, patients often experience an abrupt onset of severe, diffuse abdominal pain, along with signs of peritonitis. An upright chest or abdominal X-ray commonly demonstrates free air (pneumoperitoneum) beneath the diaphragm, as gastric or duodenal contents escape into the peritoneal cavity. This is a surgical emergency because sepsis can rapidly ensue if the perforation is not closed or patched (e.g., Graham patch). Although proton pump inhibitors (PPIs) are critical for acid suppression and aid in the healing of peptic ulcers, they are not sufficient as the sole therapy in acute perforation. Surgical intervention is frequently needed to repair the perforation and irrigate the peritoneal cavity. Hence, statement (C) is the exception and is incorrect for perforated peptic ulcer management.

(B) Hepatocellular carcinoma


Patients with chronic liver disease and cirrhosis are at high risk for developing HCC, often presenting with a hepatic mass and elevated serum alpha-fetoprotein (AFP). This tumor typically occurs in the background of long-standing cirrhosis from viral hepatitis, alcohol misuse, or other etiologies. Cholangiocarcinoma (A) arises from the biliary epithelium and generally does not elevate AFP. A hepatic hemangioma (C) is a benign vascular lesion, usually discovered incidentally, without causing dramatic elevations in AFP. Metastatic colon cancer (D) can present as multiple or solitary liver lesions, but these metastases do not usually elevate AFP to significant levels, and the primary colon pathology would often be evident. Given the background of cirrhosis, a solitary liver mass, and marked AFP elevation, hepatocellular carcinoma remains the most likely diagnosis.

29. A 52-year-old with ulcerative colitis for 15 years, 34. Which best differentiates benign from malignant
presents with altered bowel habits, occasional gastric ulcer on endoscopy?
rectal bleeding, weight loss, and an abdominal (A) Margins are always undermined in malignant
mass. Colonoscopy reveals a non-obstructing ulcers
lesion in the transverse colon. Biopsy suggests (B) Benign ulcers tend to have smooth, regular
adenocarcinoma. Imaging rules out distant margins
metastasis. Which management strategy is most (C) Malignant ulcers typically demonstrate a
appropriate? clean base
(A) Segmental resection of transverse colon (D) Biopsy is unnecessary for suspicious lesions
(B) Total colectomy with ileoanal pouch (B) Benign ulcers tend to have smooth, regular margins
Endoscopically, benign gastric ulcers usually have a smooth, round, or oval shape with well-defined margins and a flat base. They often have radiating mucosal folds that approach the edge of the ulcer in a symmetrical fashion. By contrast, malignant ulcers may exhibit irregular, heaped-up, or nodular edges, with possible mass-like appearance or friability. While some malignant ulcers can appear undermined, not all do, so (A) is too absolute. A clean base (C) is typically a sign of lower bleeding risk but does not necessarily differentiate benign from malignant. Any suspicious lesion or ulcer (D), especially with atypical features, mandates biopsy to rule out malignancy. Thus, in general, benign ulcers exhibit smoother, more regular morphology, whereas malignant ulcers are often irregular and require histological assessment for confirmation.

(C) Local excision of the lesion only


35. A patient with acute pancreatitis develops a large
(D) Chemotherapy without surgery
pseudocyst that is symptomatic and persists
beyond six weeks. What is the best next step in
(B) Total colectomy with ileoanal pouch
Patients with long-standing ulcerative colitis have an increased risk of developing colorectal cancer throughout the entire colon, not just at a specific lesion site. Once a malignant lesion is discovered, more definitive surgical management is indicated to remove all at-risk colonic mucosa. Segmental resection (A) leaves remaining diseased colon, which can harbor dysplasia or further cancers. Local excision (C) is inadequate for malignant lesions in ulcerative colitis because of the high risk of synchronous or metachronous disease. Chemotherapy (D) alone may help with palliation or adjuvant treatment but does not address the diseased colon or the primary lesion definitively. Removing the entire colon and creating an ileoanal pouch preserves continence while eliminating colonic mucosa susceptible to malignant transformation. Hence, total colectomy with ileoanal pouch is the most appropriate management strategy.

30. Which term specifically describes an abnormal management?


connection between two epithelialized surfaces, (A) Conservative management with bowel rest
often involving organs or vessels? (B) Immediate endoscopic drainage
(A) Fistula (C) Surgical debridement of pancreatic necrosis
(B) Sinus (D) Endoscopic or percutaneous drainage if it is
(C) Abscess symptomatic and mature
(D) Ulcer (D) Endoscopic or percutaneous drainage if it is symptomatic and mature
Pancreatic pseudocysts are collections of pancreatic fluid enclosed by a wall of fibrous or granulation tissue, typically forming after an episode of pancreatitis. They often resolve spontaneously, but when they become large, persistent (longer than six weeks), and symptomatic — causing pain, obstruction, or risk of rupture — intervention is indicated. Conservative management (A) may be an option for smaller or asymptomatic pseudocysts that have not reached six weeks or are stable. Immediate endoscopic drainage (B) without waiting for the cyst to mature can lead to complications like hemorrhage because the wall might not be fully formed. Surgical debridement (C) is more pertinent for infected or necrotic tissue rather than a straightforward pseudocyst. Hence, endoscopic or percutaneous drainage of a mature, symptomatic pseudocyst remains the most appropriate step in its management.

36. A 25-year-old male experiences severe pain and


(A) Fistula
A fistula is an abnormal communication or passage between two epithelialized structures, such as between an organ and the skin, or between two hollow organs (e.g., an enterocutaneous fistula). A sinus (B) typically refers to a tract leading from a focus of infection or abscess to an epithelial surface, but it does not necessarily connect two separate organs. An abscess (C) is a localized collection of pus, surrounded by inflamed tissue, without implying an abnormal tract. An ulcer (D) is a localized defect or excavation on a surface, like a mucous membrane or skin, that results from the gradual disintegration of the tissue. Since the question focuses on an abnormal connection or passage between epithelialized surfaces, a fistula best fits this definition. Examples include a tracheoesophageal fistula or a vesicouterine fistula.

31. Which intravenous fluid is typically considered swelling of the scrotum following a sporting
isotonic and used for fluid resuscitation? injury. Doppler ultrasound shows reduced blood
(A) 0.45 saline flow to the testis. What is the likely diagnosis?
(B) 5 dextrose in water (A) Testicular torsion
(C) Normal saline (0.9 NaCl) (B) Epididymo-orchitis
(D) 3 saline (C) Inguinal hernia
(C) Normal saline (0.9 NaCl)
Intravenous fluid resuscitation frequently utilizes isotonic fluids to expand intravascular volume without significant shifts in or out of cells. Normal saline has a similar osmolarity to plasma and is considered a standard choice in many clinical scenarios, such as hypovolemia, shock, or dehydration. In contrast, 0.45 saline (A) is hypotonic, which can lead to a shift of water into cells. A 5 dextrose solution in water (D5W) (B) is technically isotonic in the bag, but once in the bloodstream, the dextrose is rapidly metabolized, leaving free water that makes it effectively hypotonic to body fluids. Hypertonic saline (3 saline) (D) is used more selectively for conditions like severe hyponatremia, but it is not a routine choice for general resuscitation because of the risk of osmotic shifts. Thus, normal saline is the classic isotonic fluid.
(D) Hydrocele
(A) Testicular torsion
This condition involves twisting of the spermatic cord, leading to compromised blood flow and ischemia. It often presents with sudden onset of severe scrotal pain, swelling, and a high-riding testis with an abnormal lie. Doppler ultrasound typically reveals reduced or absent blood flow to the affected testis, making torsion a surgical emergency. Epididymo-orchitis (B) is characterized by inflammation, usually with increased blood flow on Doppler (hyperemia) due to infection or inflammation, and it typically has a more gradual onset. An inguinal hernia (C) may cause a scrotal swelling extending from the inguinal canal, but it does not directly alter the testicular blood flow in this manner. A hydrocele (D) is a fluid collection around the testis, usually painless, and does not reduce testicular blood flow. Therefore, testicular torsion is the most likely diagnosis.

32. In surgical oncology, which principle underlies


the practice of wide local excision of malignant 37. Arrange these steps in laparoscopic appendectomy
tumors? in the correct order:
(A) Ensuring tumor cells are touched during 1. Identification of appendix
dissection 2. Establish pneumoperitoneum
(B) Preserving as much tissue as possible 3. Division of mesoappendix
(C) Achieving clear margins beyond the tumor 4. Ligation of appendix stump
boundary Options :
MEDINK GRAND TEST 01

(A) 2-1-3-4 (C) Regular ultrasound surveillance


For an asymptomatic abdominal aortic aneurysm (AAA) measuring 3 cm, immediate intervention is generally not indicated because the risk of rupture is relatively low for smaller aneurysms. Guidelines typically recommend periodic imaging to monitor the aneurysm size and growth rate — usually ultrasound every 2 to 3 years for aneurysms less than 4 cm. Surgical repair (A), either open or endovascular (B), is typically considered if the AAA exceeds 5.0 or 5.5 cm in diameter, is rapidly expanding (more than 1 cm per year), or becomes symptomatic. Discharging with no follow-up (D) is inappropriate because aneurysms can enlarge over time and require eventual intervention. Therefore, regular ultrasound surveillance is the best step to keep track of aneurysm progression and intervene if it meets the criteria for repair.

(B) 1-2-4-3
43. Concerning variceal bleeding management, which
(C) 2-3-1-4
statements are correct?
(D) 1-4-3-2
1. Endoscopic band ligation is a mainstay of
therapy.
(A) 2-1-3-4
In a laparoscopic appendectomy, the first step after anesthetic induction and positioning is establishing a pneumoperitoneum (2) to allow visualization. Next, the surgeon locates the appendix (1), typically in the right lower quadrant, identifying it by tracing the taenia coli to the base of the cecum. The mesoappendix, which contains the appendicular artery, is then divided (3), often using energy devices or clips to control bleeding. Finally, the appendix stump is ligated (4), either with endoloops, staplers, or sutures, and the appendix is removed. Other sequences would disrupt this logical progression. For instance, ligating the stump before dividing the mesoappendix would be difficult if the mesoappendix was not mobilized. Therefore, 2-1-3-4 is the correct order for these steps in a laparoscopic appendectomy.

38. A 72-year-old patient with known benign prostatic 2. IV vasoactive drugs (e.g., octreotide) help
hyperplasia has acute urinary retention. Which key reduce portal pressure.
intervention relieves obstruction? 3. Transjugular intrahepatic portosystemic shunt
(A) Trial of alpha-blockers only (TIPS) is used if bleeding is uncontrolled.
(B) Insert a suprapubic catheter 4. Beta-blockers have no role in prophylaxis.
(C) Urethral catheterization Options :
(D) Immediate transurethral resection of the (A) 1, 2, and 3 are correct
prostate (B) 1 and 4 are correct
(C) 2 and 3 are correct
(D) 1, 2, 3, and 4 are correct
(C) Urethral catheterization
In acute urinary retention, especially in the context of benign prostatic hyperplasia (BPH), the immediate objective is to decompress the bladder. A urethral catheter is usually the first-line, simplest, and least invasive method. Alpha-blockers (A) can assist by relaxing smooth muscle in the bladder neck and prostate, but they are not the immediate solution for acute retention. A suprapubic catheter (B) is generally reserved for patients in whom urethral catheterization is not feasible or contraindicated. While transurethral resection of the prostate (TURP) (D) may ultimately be necessary for definitive management of BPH, it is not performed emergently at the moment of acute retention unless there are compelling reasons, such as repeated complications or inability to catheterize. Thus, urethral catheterization is the key intervention to relieve the obstruction promptly in acute urinary retention.

39. A patient is suspected of having an abdominal (A) 1, 2, and 3 are correct


Endoscopic band ligation (1) is a primary treatment for acute variceal bleeding, as it mechanically obliterates the varices. IV vasoactive drugs like octreotide (2) reduce splanchnic blood flow and portal pressure, aiding hemorrhage control. Transjugular intrahepatic portosystemic shunt (TIPS) (3) is considered when endoscopic and medical therapy fail, providing a decompressive channel between the portal and systemic circulation. Beta-blockers (4), such as nonselective agents (propranolol, nadolol), are indeed used in primary and secondary prophylaxis of variceal bleeding by lowering portal venous pressure. The statement that beta-blockers have no role is incorrect. Therefore, statements 1, 2, and 3 are correct, making option (A) the right choice.

aortic aneurysm (AAA). Which imaging study is


most appropriate for initial screening? 44. A middle-aged patient with gastroesophageal reflux
(A) Abdominal X-ray disease (GERD) undergoes 24-hour esophageal pH
(B) Abdominal ultrasound monitoring that confirms significant acid reflux.
(C) CT angiography What is the key surgical procedure to correct the
(D) MRI of the abdomen mechanical defect?
(A) Vagotomy and drainage procedure
(B) Nissen fundoplication
(B) Abdominal ultrasound
For suspected abdominal aortic aneurysm (AAA), an ultrasound is widely regarded as the best initial screening test. It is noninvasive, cost-effective, readily available, and highly sensitive for detecting aneurysms and measuring their size. An abdominal X-ray (A) might show a calcified aortic wall, but it is neither sufficiently sensitive nor specific for diagnosis. CT angiography (C) provides more detailed anatomical information and is excellent for surgical planning, but it involves radiation and intravenous contrast, making it more suitable after an initial positive ultrasound or in urgent scenarios with complications. MRI (D) could also offer detailed imaging but is more expensive and not typically used for routine screening. Consequently, abdominal ultrasound stands out as the most appropriate initial screening modality for an AAA.

40. In a patient with papillary thyroid carcinoma (C) Billroth II gastrojejunostomy


confirmed on fine needle aspiration (FNA), what (D) Heller myotomy
is the most definitive surgical treatment? (B) Nissen fundoplication
Chronic GERD can be surgically addressed by reinforcing the lower esophageal sphincter (LES), typically via fundoplication, wherein the gastric fundus is wrapped around the distal esophagus. A complete (360-degree) Nissen fundoplication effectively restores the barrier to reflux. Vagotomy and drainage (A) is used for peptic ulcer disease related to acid hypersecretion, not for mechanical incompetence of the LES. Billroth II gastrojejunostomy (C) is a reconstructive procedure following partial gastrectomy, not an antireflux operation. Heller myotomy (D) is primarily for achalasia, which involves cutting the hypertrophied LES muscle to relieve obstruction, the opposite of what is needed in GERD. Hence, Nissen fundoplication is the key action for significant GERD not controlled by medical therapy, as it repairs the mechanical defect at the gastroesophageal junction.

(A) Lobectomy of the affected lobe only


(B) Total thyroidectomy with possible central 45. In a randomized controlled trial comparing
neck dissection two surgical techniques for hernia repair, the
(C) Radioactive iodine ablation alone confidence interval for the difference in recurrence
(D) Isthmusectomy rates includes zero. What does this imply?
(A) There is no difference in outcomes at all
(B) The sample size was definitely too small
(B) Total thyroidectomy with possible central neck dissection
Papillary thyroid carcinoma is the most common type of thyroid cancer and often presents with multifocal disease within the gland. While lobectomy (A) is sometimes considered for very small, low-risk tumors, total thyroidectomy reduces the risk of recurrence and allows for better follow-up with thyroglobulin levels. Central neck dissection (removal of level VI lymph nodes) is considered when there is evidence of nodal involvement or high-risk features. Radioactive iodine ablation (C) may follow surgery to eradicate microscopic residual disease but is not a sole primary treatment for confirmed papillary cancer. An isthmusectomy (D) is only suitable for rare tumors confined strictly to the thyroid isthmus and is not the standard. Thus, total thyroidectomy with possible central neck dissection is the most definitive approach for papillary thyroid carcinoma.

41. A 64-year-old alcoholic male presents with (C) The results are not statistically significant at
upper GI bleeding. Endoscopy shows a dilated the chosen alpha level
submucosal venous plexus in the distal esophagus. (D) Both techniques are equally effective in every
Which condition is likely causing these findings? patient
(A) Esophageal varices due to portal hypertension (C) The results are not statistically significant at the chosen alpha level
When a confidence interval for the difference in recurrence rates includes zero, it means that the difference might be positive, negative, or zero, failing to reject the null hypothesis of no difference within the specified confidence range (usually 95). It does not guarantee that there is truly no difference (A), only that the study did not find a statistically significant difference. The sample size may or may not be too small (B); even large studies can produce confidence intervals that cross zero if the true difference is minimal. Also, it does not prove that the two surgical techniques are equally effective in all patients (D), only that the difference was not shown to be significant statistically. Therefore, the key takeaway is that the result is not significant at the predetermined alpha level.

(B) Malignant esophageal tumor


(C) Mallory-Weiss tear 46. In the primary survey (ABCs) of trauma
(D) Dieulafoy’s lesion management, which of the following is incorrect?
(A) Airway with cervical spine protection
(B) Breathing and ventilation assessment
(A) Esophageal varices due to portal hypertension
In an alcoholic patient, chronic liver disease is a common etiology leading to cirrhosis and increased portal venous pressure. This portal hypertension causes the formation of varices — dilated submucosal veins — particularly in the lower esophagus, which can rupture and cause severe upper GI bleeding. A malignant esophageal tumor (B) may also bleed, but it typically presents with progressive dysphagia, weight loss, and a mucosal lesion on endoscopy, not simply dilated submucosal vessels. A Mallory-Weiss tear (C) results from a longitudinal mucosal tear at the gastroesophageal junction, often due to severe retching, and does not appear as a dilated venous plexus. A Dieulafoy’s lesion (D) is a rare vascular malformation that presents as a small submucosal artery, not a vein, usually in the proximal stomach. Therefore, esophageal varices secondary to portal hypertension is the most likely cause.

42. A patient with a stable 3 cm abdominal aortic (C) Circulation with hemorrhage control
aneurysm is identified on routine ultrasound. He is (D) Definitive imaging before stabilizing vitals
asymptomatic. What is the best next step? (D) Definitive imaging before stabilizing vitals
In trauma management, the primary survey follows the structured approach of ABCs — Airway with cervical spine protection (A), Breathing and ventilation (B), and Circulation with hemorrhage control (C). The aim is to rapidly identify life-threatening injuries and stabilize the patient. Definitive imaging, such as CT scans, is part of further assessment and should not supersede initial stabilization of the airway, breathing, and circulation. Patients must first be resuscitated and hemodynamically stabilized before being taken for extensive imaging unless a targeted scan (e.g., FAST ultrasound) is performed at the bedside to aid immediate decision-making. Prioritizing imaging over vital stabilization can lead to deterioration or even death during transport to the radiology suite. Hence, (D) is the incorrect step in the primary survey sequence.

(A) Immediate surgical repair


(B) Endovascular stent placement 47. During a screening program for breast cancer, a
(C) Regular ultrasound surveillance test yields a high number of false positives. What
(D) Discharge with no follow-up does this suggest about the test’s specificity?
page 10 NEET PG Surgery

(A) High specificity (B) Xenograft


(B) Low specificity (C) Autograft
(C) High sensitivity (D) Isograft
(D) Low sensitivity (B) Xenograft
In transplant terminology, a xenograft is tissue transplanted from a donor of one species to a recipient of another species, such as a pig valve into a human heart. An allograft (A) involves transplantation between genetically non-identical members of the same species (e.g., one human to another). An autograft (C) is when tissue is taken from one site on a patient’s body and transplanted to another site on the same individual, like a skin graft from the thigh to the forearm. An isograft (D) involves transplantation between genetically identical individuals, such as identical twins. Because the question specifically highlights different species, “xenograft” is the precise term. Xenotransplantation offers potential solutions to organ shortages, but immune rejection remains a major challenge. Thus, the term xenograft specifically denotes this cross-species transplantation, distinguishing it from the other types of grafts.

52. According to the Parkland formula for burn fluid


(B) Low specificity
Specificity of a screening test refers to its ability to correctly identify those without the disease. If a test produces a high number of false positives, it means many healthy individuals are incorrectly labeled as having potential breast cancer, indicating that the test is not effectively ruling out disease-free subjects. This translates to low specificity. A test with high specificity would yield very few false positives. Sensitivity is the ability to correctly identify those with the disease, so a high number of false positives does not directly indicate high or low sensitivity. Instead, false negatives are associated with low sensitivity. The question specifically pertains to excessive false positives, which can trigger unnecessary anxiety, follow-up tests, and biopsies. Thus, the test’s specificity must be low.

48. Why does tension pneumothorax lead to tracheal resuscitation, how many milliliters of crystalloid
deviation away from the affected side? solution are given per kilogram body weight per
(A) Collapse of the lung draws the trachea percent total body surface area (TBSA) burned
ipsilaterally over the first 24 hours?
(B) Increased intrapleural pressure pushes (A) 2 mL/kg/TBSA
mediastinal structures contralaterally (B) 3 mL/kg/TBSA
(C) Bronchospasm forces air to the opposite lung (C) 4 mL/kg/TBSA
(D) Decreased alveolar pressure shifts the (D) 6 mL/kg/TBSA
mediastinum (C) 4 mL/kg/TBSA
The Parkland formula is commonly used to estimate fluid requirements in the first 24 hours after a major burn. Specifically, 4 mL of crystalloid (usually Ringer’s lactate) is administered per kilogram of body weight for each percent of the body surface area (BSA) burned. Half of that total volume is given in the first 8 hours post-injury (from the time of the burn), and the remaining half over the next 16 hours. Lower volumes such as 2 mL/kg/TBSA (A) may lead to inadequate perfusion, while 6 mL/kg/TBSA (D) can risk fluid overload and complications like compartment syndromes. The 3 mL/kg/TBSA option (B) is sometimes considered for certain protocols, but the classic and most widely accepted figure remains 4 mL/kg/TBSA. This formula provides a starting point; actual fluid therapy is then titrated to clinical end points like urine output, mental status, and vital signs.

53. Which definition most accurately describes


(B) Increased intrapleural pressure pushes mediastinal structures contralaterally
In tension pneumothorax, air progressively accumulates in the pleural space and cannot escape, raising intrathoracic pressure on the affected side. This pressure buildup compresses the ipsilateral lung and shifts the mediastinum, including the trachea, toward the opposite side. Collapse of the lung (A) itself would not typically “draw” the trachea ipsilaterally; instead, it is the heightened pressure that pushes the mediastinum away. Bronchospasm (C) does not cause a shift in mediastinal structures. Decreased alveolar pressure (D) is not the driving factor; tension pneumothorax is characterized by elevated intrapleural pressure. Thus, the fundamental cause-and-effect relationship is the increase in intrapleural pressure forcibly shifting the mediastinum to the contralateral side, producing the classic clinical sign of tracheal deviation away from the affected side.

49. Match the following tumor markers to their “sepsis” based on the older, commonly used
associated cancers: criteria?
(A) Infection plus a single fever episode
Column I Column II
(B) Systemic inflammatory response syndrome
1. Alpha-fetoprotein a. Ovarian carcinoma
(SIRS) without infection
(AFP)
(C) SIRS criteria met in the presence of an
2. CA-125 b. Hepatocellular infectious process
carcinoma
(D) Hypotension in response to any infection
3. CA 19-9 c. Prostate cancer (C) SIRS criteria met in the presence of an infectious process
Historically, sepsis was commonly defined as two or more SIRS criteria (e.g., fever or hypothermia, tachycardia, tachypnea or low PaCO₂, and altered white blood cell count) in response to a documented or suspected infection. Merely having a fever (A) or fulfilling SIRS without infection (B) would not qualify as sepsis. Hypotension in response to infection (D) suggests septic shock, a more severe level of sepsis. Though newer definitions (Sepsis-3) emphasize organ dysfunction (often using the SOFA or qSOFA criteria), the question specifically references the older, commonly taught definition. Thus, SIRS criteria plus an infectious source is the conceptual hallmark of the classic definition of sepsis. This framework helps guide clinicians in early recognition and management of patients at risk for septic complications.

4. Prostate-specific d. Pancreatic cancer


antigen (PSA) 54. All of the following are features of an acute adrenal
Select the correct answer using the code given (Addisonian) crisis except:
below (A) Severe hypotension resistant to fluid
(A) 1-b, 2-a, 3-d, 4-c resuscitation
(B) 1-c, 2-d, 3-a, 4-b (B) Hyperpigmentation of the skin
(C) 1-a, 2-c, 3-b, 4-d (C) Acute hypoglycemia
(D) 1-d, 2-b, 3-c, 4-a (D) Decreased serum potassium levels
(D) Decreased serum potassium levels
In acute adrenal insufficiency, or Addisonian crisis, patients frequently present with severe hypotension (A) that may be refractory to standard fluid management due to the lack of mineralocorticoid (aldosterone) and glucocorticoid (cortisol) effects. Hyperpigmentation (B) can be observed in primary adrenal insufficiency because of elevated ACTH, which shares a precursor with melanocyte-stimulating hormone. Hypoglycemia (C) arises from cortisol deficiency, as cortisol is pivotal for gluconeogenesis and preventing hypoglycemia during stress. Hyperkalemia is common because aldosterone normally enhances potassium excretion in the kidneys; lack of aldosterone leads to elevated serum potassium, not reduced levels. Hence, decreased serum potassium (D) is inconsistent with an Addisonian crisis, making it the exception. Treatment involves prompt administration of IV fluids, glucocorticoids (such as hydrocortisone), and correction of electrolyte imbalances.

(A) 1-b, 2-a, 3-d, 4-c


Alpha-fetoprotein (AFP) is commonly elevated in hepatocellular carcinoma (b) and certain germ cell tumors. CA-125 is often used to monitor ovarian carcinoma (a). CA 19-9 is most closely linked to pancreatic cancer (d) and, to a lesser extent, biliary tract malignancies. Prostate-specific antigen (PSA) is the hallmark tumor marker for prostate cancer (c), used in diagnosis, monitoring, and screening. The other matchings fail to link the markers to their most classically associated malignancies. For instance, AFP is not primarily associated with prostate cancer, and PSA is not tied to ovarian carcinoma. Therefore, the correct associations are AFP with hepatocellular carcinoma, CA-125 with ovarian carcinoma, CA 19-9 with pancreatic cancer, and PSA with prostate cancer.

50. Assertion (A) : Sentinel lymph node biopsy is 55. Which finding best differentiates an acute subdural
used in breast cancer staging. hematoma (SDH) from an epidural hematoma
Reason (R) : It identifies the first lymph node(s) (EDH) on imaging?
that drain the tumor site. (A) Acute SDH is typically lens-shaped, while
(A) Both A and R are true, and R is the correct EDH is crescent-shaped
explanation of A (B) Acute SDH crosses suture lines, while EDH
(B) Both A and R are true, but R is not the correct is often limited by suture lines
explanation of A (C) EDH often crosses dural reflections, while
(C) A is true, R is false SDH does not
(D) A is false, R is true (D) Both appear identically on CT scans, making
them indistinguishable
(A) Both A and R are true, and R is the correct explanation of A
Sentinel lymph node biopsy has become a standard procedure in early-stage breast cancer to evaluate potential lymphatic spread without performing a full axillary lymph node dissection. The rationale behind this technique is that the sentinel lymph node(s) is/are the first draining node(s) from the primary tumor site. If this node is free of metastasis, the likelihood of further nodal involvement is dramatically reduced. Conversely, if the sentinel node is positive, more extensive axillary assessment or dissection may be warranted. Therefore, the reason (R) accurately explains the purpose of sentinel lymph node biopsy (A). Other options would either incorrectly deny the connection between sentinel node biopsy and staging or suggest an incorrect reason. Hence, (A) is the best match.

(B) Acute SDH crosses suture lines, while EDH is often limited by suture lines
On CT scans, an epidural hematoma (EDH) classically appears as a biconvex (lentiform), lens-shaped collection that does not cross suture lines because it is bound by the firm attachments of the dura at the cranial sutures. A subdural hematoma (SDH), by contrast, is crescent-shaped and can cross suture lines but typically does not cross midline dural reflections such as the falx cerebri. Option (A) is incorrect because it reverses the characteristic shapes: SDH is usually crescentic, and EDH is lens-shaped. EDH does not typically cross dural reflections (C), whereas SDH can spread over a larger area following the brain’s contours. They are certainly distinguishable on imaging (D). Thus, the hallmark difference is that subdural collections cross suture lines, while epidural bleeds are confined by them.

51. Which term refers to a tissue graft transplanted


between individuals of different species? 56. A 25-year-old presents with a high-energy open
(A) Allograft tibial fracture. Which immediate management step
is most appropriate?
MEDINK GRAND TEST 01

(A) Exploration in the operating room and (B) Second trimester


immediate definitive fixation (C) Third trimester
(B) Intravenous antibiotics, thorough wound (D) Immediate postpartum period only
irrigation, and stabilization (B) Second trimester
If gallstone-related complications arise during pregnancy and surgery becomes necessary, the second trimester is generally considered the safest window. During the first trimester (A), there is a higher risk of teratogenicity from anesthesia and miscarriage. The third trimester (C) poses challenges due to an enlarged uterus, increasing the difficulty of laparoscopic access and raising the risk of preterm labor. Although some might wait until the postpartum period (D), persistent symptoms or complications such as recurrent biliary colic, cholecystitis, or pancreatitis can endanger both mother and fetus. Elective laparoscopic cholecystectomy in the second trimester typically offers a better risk-benefit profile, provided it is performed by an experienced surgical team. Anesthesia risk is relatively minimized, and the pregnancy is more stable. Therefore, if surgical management is indeed indicated, the second trimester is regarded as the safest period for intervention.

(C) Wet-to-dry dressing without antibiotics


(D) Delayed management until visible signs of 61. A patient with large bilateral benign multinodular
infection appear goiter causing compressive symptoms (dyspnea,
dysphagia) seeks definitive treatment. Which
procedure is most definitive?
(B) Intravenous antibiotics, thorough wound irrigation, and stabilization
Management of open fractures follows a standard protocol, often summarized as “ABCs” first (ensuring patient stability) and then focusing on the fracture. Early intravenous antibiotics are critical to prevent infection, especially with high-energy open fractures. Extensive irrigation and debridement remove debris and contaminants, lowering the risk of osteomyelitis. Stabilization of the fracture, often with external fixation initially, helps protect the soft tissues and facilitate subsequent definitive surgery. Option (A) may be applicable after thorough debridement and initial management, but immediate definitive internal fixation before proper decontamination is risky. Wet-to-dry dressings alone (C) are insufficient without antibiotics. Waiting for signs of infection (D) would permit bacterial proliferation and dramatically increase the chance of chronic osteomyelitis. Therefore, IV antibiotics, irrigation, and fracture stabilization are the most appropriate initial steps.

57. A 38-year-old patient with episodic flushing, (A) Radioactive iodine ablation
diarrhea, and wheezing undergoes imaging that (B) Total thyroidectomy
reveals a small bowel mass with hepatic lesions. (C) Hemithyroidectomy of the larger side only
Elevated urine 5-HIAA is found. The most likely (D) High-dose thyroxine suppression therapy
diagnosis is: (B) Total thyroidectomy
In a patient with a large multinodular goiter that is symptomatic and benign, removing the entire thyroid gland is the most definitive way to relieve compressive symptoms and eliminate future nodular recurrences. Radioactive iodine ablation (A) is sometimes used in hyperthyroid multinodular goiters or patients who cannot undergo surgery, but it may not reliably resolve large mass effects quickly. Hemithyroidectomy (C) would only address one side, leaving residual thyroid tissue that may continue to cause symptoms or enlarge. High-dose thyroxine suppression therapy (D) has limited efficacy and potential side effects; it is no longer commonly advised for large goiters. Total thyroidectomy completely removes the gland, providing definitive resolution of symptoms, though lifelong thyroid hormone replacement becomes necessary afterward. For a patient with significant compressive manifestations, total thyroidectomy is the gold-standard surgical option.

(A) VIPoma
(B) Zollinger-Ellison syndrome 62. A 60-year-old alcoholic with cirrhosis presents
(C) Carcinoid tumor with hematemesis and a history of recurrent
(D) Insulinoma variceal bleeding. Endoscopy reveals large
esophageal varices. What prophylactic measure
can reduce rebleeding risk?
(C) Carcinoid tumor
Patients with carcinoid syndrome typically present with episodic flushing, diarrhea, and bronchospasm (wheezing). Carcinoid tumors often originate in the small intestine (particularly the ileum) and can metastasize to the liver. When liver metastases occur, vasoactive substances such as serotonin can enter the systemic circulation, producing the classic symptoms. Measurement of 5-HIAA (5-hydroxyindoleacetic acid), a serotonin metabolite, in a 24-hour urine sample supports the diagnosis. VIPoma (A) presents with profound watery diarrhea, hypokalemia, and achlorhydria (WDHA syndrome). Zollinger-Ellison syndrome (B) is characterized by gastrin-secreting tumors causing severe peptic ulcer disease. Insulinoma (D) causes hypoglycemia due to excessive insulin. Among these neuroendocrine tumors, the combination of flushing, diarrhea, wheezing, small bowel mass, hepatic lesions, and elevated 5-HIAA most strongly points to carcinoid tumor.

58. Arrange the following steps in midline laparotomy (A) Endoscopic polypectomy
wound closure in the correct sequence: (B) Beta-blockers (e.g., propranolol)
1. Approximate the fascia with sutures (C) Splenectomy
2. Irrigate the wound with saline (D) Esophageal balloon tamponade
3. Inspect the peritoneal cavity (B) Beta-blockers (e.g., propranolol)
Nonselective beta-blockers reduce portal venous inflow by lowering cardiac output (beta-1 blockade) and causing splanchnic vasoconstriction (beta-2 blockade), thus decreasing portal pressures. This significantly reduces the risk of variceal rebleeding. While endoscopic variceal ligation (EVL) is also a key prophylactic measure, the question specifically asks for a prophylactic measure to lower rebleeding risk, and nonselective beta-blockers are a mainstay. Endoscopic polypectomy (A) is irrelevant for varices because they are vascular dilations, not polyps. Splenectomy (C) can sometimes help if hypersplenism is an issue, but it is not a standard prophylactic measure for variceal rebleeding in cirrhosis. Esophageal balloon tamponade (D) is a temporizing measure in acute variceal hemorrhage, not a long-term prophylaxis. Therefore, nonselective beta-blockers remain a cornerstone in reducing portal hypertension and preventing variceal rebleeding.

4. Close the skin


Select the option for correct order : 63. A patient with acute lower GI bleed has bright red
(A) 3-1-2-4 blood per rectum and hemodynamic instability.
(B) 3-2-1-4 After initial resuscitation, colonoscopy is non-
(C) 2-3-1-4 diagnostic. What is the best next step to localize
(D) 1-3-4-2 the bleeding site?
(A) Barium enema
(B) Angiography (mesenteric angiogram)
(B) 3-2-1-4
After completing the intra-abdominal portion of the surgery, the first step before final closure is to inspect the peritoneal cavity (3) to ensure no bleeding or spillage remains. Next, irrigation of the wound (2) with saline helps remove debris and reduces infection risk. Then, the fascia is approximated (1), typically with interrupted or continuous sutures using appropriate material (e.g., slowly absorbable or non-absorbable). Finally, the skin is closed (4), using staples or sutures. Other sequences either close the fascia before irrigation or skip the inspection. For example, closing the fascia without irrigating the wound might trap contaminants, and failing to inspect the peritoneum could miss a bleed. Hence, the logical order for a midline laparotomy closure is: inspect the cavity, irrigate, approximate fascia, and then close the skin.

59. A 28-year-old is stabbed in the left chest but (C) Exploratory laparotomy
remains hemodynamically stable. On auscultation, (D) CT scan of the abdomen/pelvis
breath sounds are slightly diminished on that side. (B) Angiography (mesenteric angiogram)
In a hemodynamically unstable patient with ongoing lower GI bleeding and a negative or inconclusive colonoscopy, angiography can both localize the source of bleeding (by identifying extravasation of contrast) and potentially allow for therapeutic embolization. Barium enema (A) is not typically used in acute bleeding scenarios; it also complicates subsequent endoscopic or angiographic evaluations. Exploratory laparotomy (C) without localization of the bleeding site can be hazardous and may fail to identify the active bleeder if not obvious. A CT scan (D) with contrast might help localize a bleed if the rate of bleeding is sufficient, but in severe ongoing hemorrhage with instability, angiography has the additional advantage of possible intervention. Hence, mesenteric angiography remains the next best step for both diagnostic and therapeutic reasons in acute, severe lower GI bleeding when endoscopy is inconclusive.

A chest X-ray reveals a small pneumothorax. The


key management step is: 64. Consider the following statements regarding the
(A) Immediate tube thoracostomy for all stable management of acute appendicitis:
pneumothoraces 1. Broad-spectrum antibiotics are given
(B) Needle decompression followed by chest preoperatively.
tube insertion 2. Interval appendectomy is mandatory after
(C) High-flow oxygen and close observation if successful conservative treatment of an
the pneumothorax is small appendiceal mass.
(D) Exploratory thoracotomy 3. Laparoscopic appendectomy reduces hospital
stay compared with open surgery in many
cases.
(C) High-flow oxygen and close observation if the pneumothorax is small
In a stable patient with a small, uncomplicated pneumothorax, conservative management is often appropriate. High-flow oxygen can enhance reabsorption of the intrapleural air. This approach requires vigilant monitoring with repeat chest X-rays to confirm that the pneumothorax does not enlarge. Immediate tube thoracostomy (A) is standard for moderate-to-large or tension pneumothoraces, or if the patient’s vitals become unstable. Needle decompression (B) is reserved for tension physiology (e.g., hypotension, marked respiratory distress, tracheal shift). Exploratory thoracotomy (D) is highly invasive and indicated only for severe chest trauma with massive hemothorax, major vascular injury, or other life-threatening findings. Since the patient is hemodynamically stable and the pneumothorax is small, observation and oxygen therapy is the key initial step. Intervention with a chest tube may become necessary if the pneumothorax grows or the patient deteriorates.

60. A 32-year-old female with chronic cholecystitis 4. CT scan is often used when the diagnosis is
and gallstones develops symptomatic gallstones uncertain.
during pregnancy. Which is the safest timing for Which of the statements given above are correct?
surgical intervention if indicated? (A) 1, 2, and 3 are correct
(A) First trimester (B) 1, 3, and 4 are correct
page 12 NEET PG Surgery

(C) 1 and 4 are correct preventing proper movement


(D) 1, 2, 3, and 4 are correct (C) It leads to hyperadduction of both vocal cords
(B) 1, 3, and 4 are correct
Broad-spectrum antibiotics (1) are administered preoperatively to decrease postoperative infection rates. In uncertain cases, a CT scan (4) can confirm the diagnosis, especially when clinical findings are atypical. Laparoscopic appendectomy (3) frequently leads to a shorter hospital stay and faster recovery compared to open surgery. However, interval appendectomy (2) after successful conservative management of an appendiceal mass or phlegmon is no longer universally mandatory. Recent studies show that many patients, especially those with complete resolution of symptoms, may not need interval surgery if follow-up imaging is clear. Therefore, statement (2) is not an absolute requirement. Thus, 1, 3, and 4 are correct regarding acute appendicitis management, while 2 is not universally accepted. Hence, the best choice is (B).
(D) It damages the sympathetic chain, altering
laryngeal tension
65. In a patient with confirmed colonic polyp showing
high-grade dysplasia, what is the key therapeutic
(B) It paralyzes the ipsilateral vocal cord, preventing proper movement
The recurrent laryngeal nerve innervates nearly all the intrinsic muscles of the larynx responsible for vocal cord abduction and adduction, crucial for speech and breathing. An injury typically causes paralysis of the ipsilateral vocal cord, leading to a hoarse, breathy voice if only one nerve is affected. In bilateral paralysis, airway compromise can result. Option (A) is incorrect because nerve damage does not cause spasm; rather, it causes a loss of function. Hyperadduction (C) would imply excessive movement toward the midline, which is not the typical outcome of a nerve injury that leads to paralysis. Damaging the sympathetic chain (D) is not related to laryngeal muscle function; it might cause Horner’s syndrome but would not directly cause hoarseness. Hence, recurrent laryngeal nerve injury and subsequent vocal cord paralysis is the direct cause of hoarseness.

action? 70. Match the surgical procedure with its primary


(A) Endoscopic removal of the polyp indication:
(B) Repeat colonoscopy in five years
Column I Column II
(C) Empiric antibiotic therapy
1. Heller myotomy a. Resectable
(D) Barium enema follow-up
pancreatic head
tumor
(A) Endoscopic removal of the polyp
Polyps with high-grade dysplasia carry a significant risk of progression to invasive cancer. The standard of care is complete excision, usually via colonoscopic polypectomy or endoscopic mucosal resection, depending on size and characteristics. Waiting five years (B) is too long, as lesions with high-grade dysplasia can progress, and guidelines recommend earlier surveillance after resection. Antibiotics (C) have no role in the management of a neoplastic colonic polyp. Barium enema (D) is an imaging study that does not offer a definitive therapeutic intervention nor the resolution of dysplastic lesions. Endoscopic removal allows both therapeutic excision and detailed histopathological examination to ensure negative margins. Timely resection decreases the likelihood of progression and facilitates close surveillance if needed. Therefore, removing the polyp endoscopically is the key intervention for high-grade dysplasia.

66. In a surgical study, the hazard ratio for mortality 2. Whipple procedure b. Sigmoid colon
with a new intervention compared to standard resection with end
treatment is 0.60 (95 CI: 0.48–0.75). How should colostomy
this be interpreted? 3. Nissen c. Achalasia
(A) The new intervention increases the risk of fundoplication
mortality 4. Hartmann’s d. GERD
(B) The new intervention has no effect on procedure
mortality Select the correct answer using the code given
(C) The new intervention reduces the hazard below
(risk) of death by 40 (A) 1-d, 2-c, 3-a, 4-b
(D) The new intervention’s effect is not (B) 1-c, 2-a, 3-d, 4-b
statistically significant (C) 1-c, 2-b, 3-d, 4-a
(C) The new intervention reduces the hazard (risk) of death by 40
A hazard ratio (HR) below 1 indicates a reduction in the event rate over time in the treatment group compared to the control group. Specifically, a hazard ratio of 0.60 means that the hazard (risk) of death at any point in time is 60 of that in the control, effectively a 40 reduction. Because the 95 confidence interval (0.48–0.75) is entirely below 1, it suggests a statistically significant reduction in mortality risk. An HR above 1 would imply an increased risk of death, which is not the case here, so (A) is incorrect. Since the confidence interval does not cross 1, (B) and (D) are also incorrect. Thus, the correct interpretation is that the new intervention significantly reduces the hazard of mortality by 40 compared to standard treatment.

(D) 1-d, 2-b, 3-c, 4-a


67. In open reduction and internal fixation (ORIF) of
(B) 1-c, 2-a, 3-d, 4-b
A Heller myotomy (1-c) is performed to relieve the lower esophageal sphincter in achalasia, improving esophageal emptying. The Whipple procedure (2-a), also known as pancreaticoduodenectomy, is indicated for resectable tumors of the pancreatic head (and sometimes for periampullary cancers). Nissen fundoplication (3-d) involves wrapping the gastric fundus around the lower esophagus to correct gastroesophageal reflux disease (GERD). Hartmann’s procedure (4-b) involves resecting the diseased portion of the sigmoid colon, usually for complicated diverticular disease or perforation, and creating an end colostomy while leaving the rectal stump closed. Therefore, the correct matching is 1-c, 2-a, 3-d, and 4-b.

a femoral shaft fracture, which step is incorrect? 71. Assertion (A) : A diverting stoma can be used to
(A) Debridement of nonviable tissues protect a distal bowel anastomosis.
(B) Precise fracture reduction under imaging Reason (R) : A proximal stoma reduces the fecal
guidance stream passing through the anastomotic site.
(C) Placing the fixation plate on the side of (A) Both A and R are true, and R is the correct
minimal cortical contact explanation of A
(D) Securing fixation with appropriate screws (B) Both A and R are true, but R is not the correct
(C) Placing the fixation plate on the side of minimal cortical contact
In ORIF of long bone fractures, meticulous debridement (A) is essential to remove devitalized tissue and prevent infection. Accurate reduction under imaging guidance (B) ensures proper alignment and limb length restoration. Securing fixation with appropriate screws (D) stabilizes the construct for healing. However, the fixation plate should be applied on the side of maximal cortical contact or the “tension side” of the fracture to counteract the forces that would disrupt healing. Placing it on the side of minimal cortical contact is incorrect because it may not provide optimal mechanical stability, leading to increased stress on implants and potentially poor healing. For instance, in femoral shaft fractures, the lateral side is typically considered the tension side. Thus, (C) is the incorrect step in ORIF.

explanation of A
(C) A is true, R is false
68. A test for acute pancreatitis has a sensitivity
(D) A is false, R is true
of 95 and specificity of 80. If the patient tests
negative, which of the following best describes the
(A) Both A and R are true, and R is the correct explanation of A
A diverting (protective) stoma — often a loop ileostomy or colostomy — is sometimes created to temporarily deflect the fecal stream away from a newly formed distal anastomosis in the colon or rectum. This reduces mechanical stress and bacterial load at the anastomosis, promoting healing and lowering the risk of anastomotic dehiscence or infection. Hence, the assertion (A) that a diverting stoma can protect a distal bowel anastomosis is true. The reason (R), that a proximal stoma reduces the fecal stream passing through the anastomotic site, directly explains how the stoma achieves its protective effect. Without the bulk of fecal content traversing the anastomosis, healing is facilitated, and complications are minimized. Therefore, both statements are correct, and (R) is indeed the correct explanation for (A).

interpretation? 72. A 58-year-old with chronic GERD develops


(A) Acute pancreatitis is almost certainly present progressive dysphagia and weight loss. Endoscopic
(B) There is a high likelihood that acute biopsy shows columnar epithelium with dysplasia.
pancreatitis is absent What’s the next step?
(C) The test has limited diagnostic utility (A) Esophageal dilation
(D) The patient definitely has pancreatitis (B) Endoscopic mucosal resection or ablation
(B) There is a high likelihood that acute pancreatitis is absent
A test with 95 sensitivity indicates it correctly identifies 95 of individuals who truly have acute pancreatitis (true positives). If a test is highly sensitive, a negative result strongly suggests the condition is unlikely to be present, as the test rarely misses cases (few false negatives). By contrast, a specificity of 80 means the test will generate some false positives, but that aspect is less relevant when the result is negative. Thus, a negative result from a highly sensitive test is reassuring for ruling out the disease — often summarized by the mnemonic “SnNout”: a highly SeNsitive test, if Negative, rules OUT the disease. The test does not make pancreatitis “almost certain” (A) nor does it guarantee that the patient “definitely has” pancreatitis (D). Therefore, (B) is the most accurate interpretation.

(C) Proton pump inhibitors only


(D) Heller myotomy
69. Why does injury to the recurrent laryngeal nerve
during thyroid surgery lead to hoarseness of voice?
(B) Endoscopic mucosal resection or ablation
In a patient with Barrett’s esophagus (columnar lining in the distal esophagus due to chronic GERD) showing dysplasia — especially high-grade dysplasia or early intramucosal carcinoma — endoscopic therapy is indicated to remove or ablate the dysplastic tissue. Proton pump inhibitors (C) remain important adjuncts but alone are insufficient once dysplasia has developed. Esophageal dilation (A) is used for strictures but does not address dysplastic changes. Heller myotomy (D) is a surgical treatment for achalasia, an unrelated motility disorder. Timely endoscopic removal or ablation of dysplastic areas can prevent progression to invasive esophageal adenocarcinoma. Therefore, endoscopic therapy is the best next step for Barrett’s esophagus with dysplasia, aiming to eradicate the potentially malignant tissue.

(A) It causes spasm of the vocal cords


(B) It paralyzes the ipsilateral vocal cord,
MEDINK GRAND TEST 01

73. A 45-year-old male with painless hematuria and


(B) Hemiglossectomy with elective neck dissection
In oral tongue cancers (squamous cell carcinoma), partial glossectomy (hemiglossectomy) is typically performed to achieve clear margins if the lesion is extensive. Even without palpable neck nodes, elective neck dissection (often a supraomohyoid or selective dissection) is recommended because occult metastases are common in tongue cancers. Wide local excision (A) may be sufficient for very small lesions, but for a “chronic nonhealing, indurated lesion” likely beyond superficial mucosa, a more extensive resection is warranted. Simple laser ablation (C) might be used for very early-stage lesions but may not provide adequate depth control in this scenario. Chemoradiation alone (D) is usually considered an alternative in advanced or inoperable cases, or adjuvant to surgery. Given the high propensity for cervical metastasis, elective neck dissection is generally advised. Thus, hemiglossectomy with elective neck dissection is most appropriate.

a renal mass on imaging is found to have tumor 78. A 62-year-old with chronic obstructive pulmonary
thrombus in the renal vein. Likely diagnosis? disease (COPD) presents with a solitary 3 cm
(A) Urothelial carcinoma peripheral lung mass suspicious for malignancy
(B) Renal cell carcinoma on CT scan. PET-CT shows no nodal or distant
(C) Renal oncocytoma metastases. Spirometry indicates borderline lung
(D) Angiomyolipoma function. The patient desires curative treatment.
(B) Renal cell carcinoma
Classically, renal cell carcinoma (RCC) may present with painless hematuria, flank pain, and occasionally a palpable mass, though many cases are detected incidentally. One notable feature is its propensity to extend into the renal vein and even into the inferior vena cava in advanced cases. Urothelial (transitional) carcinoma (A) typically arises from the renal pelvis or lower urinary tract, but extension into the renal vein is less characteristic. Renal oncocytoma (C) is a benign tumor that does not typically invade into major veins. Angiomyolipoma (D) is another benign renal tumor containing fat, smooth muscle, and blood vessels, more commonly associated with tuberous sclerosis, and it typically does not form venous tumor thrombi. Given the imaging finding of a tumor thrombus in the renal vein plus painless hematuria, renal cell carcinoma is the most likely diagnosis.

Which surgical approach offers definitive therapy


while preserving maximal lung function?
74. A patient with chronic venous insufficiency (A) Pneumonectomy
presents with a non-healing ulcer near the medial (B) Lobectomy
malleolus. Which intervention best addresses the (C) Wedge resection or segmentectomy
underlying pathophysiology? (D) Radiotherapy alone
(A) Systemic antibiotics
(B) Compression therapy
(C) Wedge resection or segmentectomy
In patients with borderline pulmonary reserve, removing less lung tissue while achieving complete tumor resection is often essential for preserving postoperative function. For small, peripherally located tumors without nodal or distant spread, a sublobar resection (wedge or segment) can offer curative intent if adequate margins and staging are obtained. Pneumonectomy (A) removes the entire lung on one side, which would severely compromise lung function in a COPD patient. A standard lobectomy (B) is the usual gold-standard for stage I non-small cell lung cancer, but might be too extensive for someone with borderline spirometry. Radiotherapy alone (D), including stereotactic techniques, may be considered if the patient is not a surgical candidate, but surgery typically yields the best long-term survival if tolerated. Hence, wedge resection or segmentectomy is the optimal compromise, offering curative intent while minimizing loss of lung function.

(C) Debridement with skin graft 79. A 48-year-old man presents with a history of rectal
(D) Hyperbaric oxygen bleeding, altered bowel habits, and weight loss over
(B) Compression therapy
Chronic venous insufficiency ulcers typically form around the medial malleolus due to venous hypertension from incompetent valves. The mainstay of treatment includes compression bandages or stockings to reduce edema, improve venous return, and lower venous pressure at the ankle. Systemic antibiotics (A) are used if there is evidence of infection, but they do not address the elevated venous pressure that perpetuates ulcer formation. Debridement and skin grafting (C) can aid wound closure but may fail if the underlying venous hypertension is not corrected. Hyperbaric oxygen (D) can be useful in certain non-healing wounds (e.g., diabetic foot ulcers), but it does not directly counteract the high venous pressures causing stasis. By contrast, compression therapy directly tackles the pathophysiology of venous insufficiency, facilitating venous return and promoting ulcer healing. Thus, it is the most appropriate fundamental intervention.

six months. Colonoscopy reveals a circumferential


lesion in the sigmoid colon. Biopsy is positive for
75. A middle-aged woman complains of a painless, adenocarcinoma. Staging investigations show no
slowly enlarging mass in the parotid region. Facial distant metastases but suspicious pericolic lymph
nerve function is intact. Which is the most likely nodes. The patient is otherwise healthy. Which
diagnosis? surgical procedure is recommended for curative
(A) Pleomorphic adenoma resection?
(B) Warthin’s tumor (A) Local transanal excision
(C) Mucoepidermoid carcinoma (B) Sigmoid colectomy with lymph node
(D) Parotid abscess dissection
(A) Pleomorphic adenoma
This benign salivary gland tumor is the most common neoplasm of the parotid gland, typically presenting as a slow-growing, painless mass. Because it is benign, it usually does not affect facial nerve function unless it becomes extremely large. Warthin’s tumor (B) is also benign, often associated with smokers, and can occur bilaterally in some cases, but pleomorphic adenoma remains more common overall. Mucoepidermoid carcinoma (C) is a malignant tumor, more likely to present with potential pain, rapid growth, or possible facial nerve involvement. A parotid abscess (D) would generally be painful, possibly with signs of inflammation or infection (fever, erythema). The classic presentation of a painless, gradually enlarging parotid mass with normal facial nerve function most strongly indicates a pleomorphic adenoma.

(C) Hartmann’s procedure without anastomosis


(D) Total proctocolectomy
76. After a high-speed motor vehicle collision, a
patient has hypotension, distended neck veins, and (B) Sigmoid colectomy with lymph node dissection
For a localized colon cancer in the sigmoid region, the standard surgical treatment is an oncologic resection of the sigmoid colon, including the primary tumor and its associated mesentery, which contains the pericolic and intermediate lymph nodes. This approach aims to achieve negative margins and adequate lymph node sampling for accurate staging. Local transanal excision (A) may be suitable for small, early rectal lesions, not a circumferential sigmoid lesion with nodal involvement. Hartmann’s procedure (C) is often used in emergency settings (e.g., perforation or obstruction with a high-risk patient) or when a primary anastomosis is contraindicated. Total proctocolectomy (D) is typically reserved for diffuse colonic diseases like ulcerative colitis or familial adenomatous polyposis. Hence, a sigmoid colectomy with appropriate lymphadenectomy is the recommended curative resection for a non-metastatic sigmoid colon adenocarcinoma.

muffled heart sounds (Beck’s triad). What is the 80. Which term describes the surgical creation of
likely diagnosis? an anastomosis between the gallbladder and the
(A) Tension pneumothorax jejunum?
(B) Cardiac tamponade (A) Choledochoduodenostomy
(C) Massive hemothorax (B) Cholecystojejunostomy
(D) Flail chest (C) Choledochojejunostomy
(B) Cardiac tamponade
Beck’s triad — consisting of hypotension, jugular venous distension, and muffled heart sounds — is classically associated with tamponade, in which fluid accumulates in the pericardial sac, restricting ventricular filling. Tension pneumothorax (A) also presents with hypotension and distended neck veins but is more commonly accompanied by absent breath sounds on one side and tracheal deviation away from the affected side. Massive hemothorax (C) would show shock and decreased breath sounds on the involved side but not the classic muffled heart sounds. Flail chest (D) involves paradoxical movement of a chest wall segment, leading to respiratory compromise but not the triad described. Because hypotension, neck vein distension, and muffled heart sounds strongly suggest impaired cardiac filling due to fluid in the pericardium, cardiac tamponade is the correct diagnosis.

(D) Gastrojejunostomy
77. A 45-year-old smoker presents with chronic (B) Cholecystojejunostomy
This term specifically refers to creating a connection between the gallbladder (cholecysto-) and the jejunum (-jejunostomy), usually to bypass an obstructed biliary tree when a choledochal anastomosis is not feasible. Choledochoduodenostomy (A) is an anastomosis between the common bile duct (choledocho-) and the duodenum. Choledochojejunostomy (C) is a bypass of the common bile duct into the jejunum, often used when the distal bile duct is not patent. Gastrojejunostomy (D) is a bypass connection from the stomach (gastro-) to the jejunum, typically performed for gastric outlet obstruction or peptic ulcer complications. Therefore, for a gallbladder-jejunum surgical bypass, “cholecystojejunostomy” is the accurate term, reflecting the direct link between the gallbladder and the jejunum.

nonhealing ulcer on the lateral border of the 81. In the context of organ transplantation, which
tongue, difficulty in swallowing, and referred immunosuppressant is known for causing
otalgia. Examination reveals an indurated lesion. nephrotoxicity as a primary side effect?
Biopsy shows squamous cell carcinoma. There is (A) Cyclosporine
no clinical neck node involvement. Which is the (B) Azathioprine
most appropriate surgical management? (C) Mycophenolate mofetil
(A) Wide local excision of the lesion (D) Prednisolone
(B) Hemiglossectomy with elective neck
dissection
(A) Cyclosporine
A calcineurin inhibitor used widely in organ transplantation to prevent rejection, cyclosporine is notorious for its nephrotoxic effects, which can manifest as acute or chronic kidney injury. Careful monitoring of drug levels and kidney function is crucial. Azathioprine (B), an antimetabolite, has bone marrow suppression, particularly leukopenia, as one of its more prominent side effects rather than nephrotoxicity. Mycophenolate mofetil (C) can cause gastrointestinal disturbances and bone marrow suppression but is not famously nephrotoxic. Prednisolone (D), a corticosteroid, may produce hyperglycemia, osteoporosis, Cushingoid features, and increased infection risk, but not direct nephrotoxicity. Hence, among the listed immunosuppressants, cyclosporine is most commonly associated with kidney injury, making it the correct answer to this question on a key side effect.

(C) Simple laser ablation


(D) Chemoradiation only
page 14 NEET PG Surgery

82. In soft tissue sarcoma management, limb-salvage (D) Anastomotic ulcer


surgery combined with radiotherapy aims (B) Blind loop syndrome leading to B12 deficiency
Blind loop syndrome occurs when a segment of intestine becomes bypassed, allowing bacterial overgrowth and malabsorption. These bacteria consume vitamin B12, predisposing to megaloblastic anemia, and interfere with normal fat absorption (steatorrhea). Neuropathy can also ensue due to combined systems degeneration from B12 deficiency. Postgastrectomy dumping syndrome (A) involves rapid gastric emptying into the small bowel, causing abdominal cramping, diarrhea, and vasomotor symptoms, but not typically severe B12 deficiency. An incisional hernia (C) presents as a protrusion through a previous surgical incision, causing local discomfort or strangulation risk, but not malabsorption or vitamin deficiency. An anastomotic ulcer (D) might manifest with pain or bleeding but not steatorrhea, megaloblastic anemia, and neuropathy. Therefore, bacterial overgrowth in a blind loop best explains these findings.

primarily to:
(A) Eliminate the need for chemotherapy 87. Arrange these stages of bone healing in
(B) Completely avoid metastases in the future chronological order:
(C) Preserve limb function while providing local 1. Callus formation
tumor control 2. Hematoma formation
(D) Achieve a quick cosmetic result without 3. Remodeling
functional concerns 4. Fibrocartilaginous callus
Select the option for correct order :
(A) 2-1-4-3
(C) Preserve limb function while providing local tumor control
Historically, amputation was a common treatment for high-grade soft tissue sarcomas of the limbs. However, advances in surgical techniques and adjuvant therapies (particularly radiation) have enabled limb-salvage procedures that remove the tumor with adequate margins while preserving critical structures. Postoperative radiotherapy helps eradicate microscopic disease, reducing local recurrence rates. This combined approach allows for meaningful functional preservation in many patients without significantly compromising survival. Eliminating the need for chemotherapy (A) is not the purpose of limb salvage plus radiotherapy — chemotherapy may still be indicated depending on tumor grade and subtype. Avoiding all future metastases (B) is not guaranteed by any local control measure, as metastasis largely depends on tumor biology. Focusing solely on cosmetic outcome (D) overlooks the primary oncologic imperative of achieving clear margins. Therefore, local control and limb function are the key goals.

83. All of the following are absolute contraindications (B) 2-4-1-3


to organ donation except: (C) 1-2-4-3
(A) Active malignancy (other than certain (D) 1-4-2-3
primary brain tumors) (B) 2-4-1-3
Bone healing typically begins with hematoma formation (2) at the fracture site, as blood accumulates and inflammatory cells migrate. Next, a fibrocartilaginous (soft) callus (4) forms, providing an initial scaffold bridging the fracture. Callus formation (1) then progresses with new woven bone replacing the soft callus, stabilizing the fracture more firmly. Finally, remodeling (3) reshapes the bone over time, converting woven bone into lamellar bone and restoring its normal architecture. This sequence — hematoma, soft callus, bony callus, and remodeling — ensures proper healing. Other options either place callus before hematoma or mix the stages incorrectly. Hence, 2-4-1-3 is the correct chronological order for bone healing.

(B) HIV infection with advanced disease


(C) Positive family refusal despite donor’s 88. A 72-year-old patient with a large, irreducible
registered wish inguinal hernia presents with severe pain and signs
(D) History of well-treated, non-metastatic of strangulation. Which key step is needed during
thyroid carcinoma surgical exploration?
(A) Routine inguinal hernioplasty without sac
inspection
(D) History of well-treated, non-metastatic thyroid carcinoma
Many organ procurement guidelines consider active malignant disease, certain transmissible infections (like HIV with advanced disease), or strong family refusal to override donor wishes as absolute contraindications. However, a history of a successfully treated, non-metastatic thyroid cancer typically does not preclude donation, especially if the patient has been in remission for a substantial period. Active malignancies (A) in donors risk transmitting cancer to recipients, a serious concern unless it is a primary brain tumor with no propensity for metastasis. Advanced HIV infection (B) can also be a contraindication in many jurisdictions, though some evolving protocols may allow HIV-positive donors for HIV-positive recipients under specialized programs. Family refusal (C) may legally override donation in certain regions, even if the individual was a registered donor. Hence, a well-treated thyroid carcinoma is not typically an absolute contraindication.

84. In distinguishing large bowel obstruction from (B) Inspect and resect any necrotic bowel before
small bowel obstruction on an abdominal X-ray, repair
which feature best characterizes a large bowel (C) Apply a truss immediately
obstruction? (D) Attempt manual reduction without anesthesia
(A) Central placement of loops with valvulae (B) Inspect and resect any necrotic bowel before repair
In a strangulated hernia, the bowel loop trapped within the hernial sac may lose its blood supply, risking necrosis and perforation. During surgical exploration, the surgeon must carefully open the hernia sac, assess the viability of the bowel, and resect any nonviable segments to prevent further complications. A routine hernioplasty (A) without inspecting the sac could miss strangulated bowel, leading to sepsis or peritonitis. A truss (C) is contraindicated in strangulation, as it may worsen the ischemia. Attempting manual reduction (D) outside the operating room can force necrotic bowel back into the abdominal cavity, risking contamination and sepsis. Therefore, the key step is to inspect for bowel viability, perform resection if necessary, and then proceed with hernia repair. This approach addresses both the hernia defect and the compromised intestinal loop.

conniventes
(B) Multiple air-fluid levels in the upper abdomen 89. A 25-year-old with a tender, fluctuant swelling
(C) Peripheral, haustra-marked loops that do not over the sacrococcygeal region is diagnosed with
span the entire width a pilonidal abscess. The most appropriate initial
(D) Absence of colonic gas management is:
(A) Antibiotics alone
(B) Wide surgical excision of the sinus tract
(C) Peripheral, haustra-marked loops that do not span the entire width
Large bowel obstruction typically manifests with distended peripheral loops in the abdomen, showing haustra that appear as incomplete lines across the bowel lumen. These haustral markings do not extend fully from wall to wall, unlike the valvulae conniventes of the small bowel. In small bowel obstruction, loops tend to be more centrally located (A) with valvulae conniventes crossing the entire width of the lumen and multiple air-fluid levels often visible (B). While it’s possible for a significant large bowel obstruction to reduce or eliminate colonic gas if the obstruction is complete (D), the classic X-ray finding is dilated peripheral loops with haustral markings. Thus, the best distinguishing feature remains the presence of haustra that partially traverse the lumen and the more peripheral location of dilated loops.

85. A 72-year-old patient with an acutely ischemic (C) Incision and drainage of the abscess
limb (cold, painful, pulseless) and a 6-hour history (D) Application of topical antiseptics
of symptoms should undergo which immediate (C) Incision and drainage of the abscess
Pilonidal disease often manifests as an acute abscess in the sacrococcygeal region. The primary step in management is to provide immediate relief by incising and draining the collection of pus, which reduces pain and inflammation. While antibiotics can be adjunctive, they are not sufficient alone (A) to address the main issue of trapped infection. Wide surgical excision (B) of the entire sinus tract and its secondary openings may be necessary later, once acute infection subsides and local inflammation is controlled. Applying topical antiseptics (D) does little to alleviate the pressure and pain caused by a significant fluctuant abscess. Thus, incision and drainage is the most appropriate initial intervention, often followed by local wound care and future definitive surgery if chronic sinus tracts persist.

intervention?
(A) Intravenous antibiotics 90. In a patient with severe necrotizing pancreatitis and
(B) Fasciotomy alone infected pancreatic necrosis confirmed by imaging
(C) Surgical embolectomy or thrombectomy and culture, what is the most definitive step?
(D) Systemic chemotherapy (A) Continue antibiotics and observe
(B) Urgent necrosectomy (surgical or minimally
invasive)
(C) Surgical embolectomy or thrombectomy
Acute limb ischemia presents a vascular emergency, with the classic “6 Ps” (pain, pallor, pulselessness, paresthesia, paralysis, poikilothermia). Interventions must be initiated promptly — usually within hours — to rescue viable tissue. Surgical or catheter-directed approaches (embolectomy/thrombectomy) aim to remove the occlusion and restore perfusion. Intravenous antibiotics (A) have no direct role unless there is concern for infection; they do not address the underlying arterial blockage. Fasciotomy (B) may be needed if compartment pressure rises after revascularization, but by itself does not restore blood flow. Systemic chemotherapy (D) is irrelevant to acute ischemia. Hence, the best immediate step is to remove the occlusion surgically or via endovascular methods. Time is critical; delay increases the risk of irreversible ischemic damage, leading to potential amputation. Therefore, prompt revascularization is the correct management for an acutely ischemic limb.

86. A 50-year-old male with chronic diarrhea, (C) High-dose proton pump inhibitors
steatorrhea, and a history of abdominal surgeries (D) ERCP with stent placement
develops megaloblastic anemia and neuropathy. (B) Urgent necrosectomy (surgical or minimally invasive)
Infected pancreatic necrosis is a serious complication of necrotizing pancreatitis, often leading to sepsis if not promptly addressed. While antibiotics can be started, the presence of necrotic tissue harboring infection usually necessitates drainage or necrosectomy to remove the devitalized tissue. Modern approaches favor minimally invasive drainage or necrosectomy once the necrosis is “walled off,” but in severe cases with clinical deterioration, more urgent intervention may be needed. Proton pump inhibitors (C) protect against stress ulcers but do not control infected necrosis. ERCP with stent placement (D) is indicated if there’s suspicion of bile duct stones or an obstructive process, not primarily for infected necrosis. Thus, the definitive management of confirmed infected necrotizing pancreatitis involves removing the necrotic debris, typically with necrosectomy or a step-up approach to drainage.

Which surgical complication is most likely?


(A) Postgastrectomy dumping syndrome 91. A 50-year-old male undergoes colonoscopy for
(B) Blind loop syndrome leading to B12 anemia, and a cecal tumor is discovered. Biopsy
deficiency reveals adenocarcinoma. CT scan shows no
(C) Incisional hernia
MEDINK GRAND TEST 01

metastases. Which surgical procedure is typically (C) 10 of those who test positive truly have breast
indicated? cancer
(A) Right hemicolectomy (D) 10 of those who test negative do not have
(B) Left hemicolectomy breast cancer
(C) Total colectomy (B) 90 of those who test positive actually have breast cancer
Positive predictive value (PPV) refers to the proportion of individuals who test positive and truly have the disease. If a test for breast cancer has a PPV of 90, it means that out of 100 positive results, 90 individuals genuinely have breast cancer, and 10 are false positives. Option (A) describes the negative predictive value, not the PPV. Option (C) inverts the statistic by suggesting 10 truly have cancer among positives, which is incorrect. Option (D) conflates negativity with the remainder who are diseased, which again confuses false negatives with PPV. Thus, by definition, a 90 PPV signifies that 90 of the positive tests correspond to actual disease, making (B) the correct statement.

(D) Transverse colectomy


96. In performing a laparoscopic inguinal hernia repair
(TAPP procedure), which step is incorrect?
(A) Right hemicolectomy
A cecal tumor is located in the right side of the colon, near the ileocecal valve. Standard oncologic resection for a malignant lesion in this region is right hemicolectomy, which includes removal of the cecum, ascending colon, and often a portion of the transverse colon, along with the associated mesentery and lymph nodes (ileocolic and right colic vessels). A left hemicolectomy (B) is indicated for lesions in the descending or sigmoid colon. Total colectomy (C) is usually reserved for multifocal disease, inherited polyposis syndromes, or conditions like ulcerative colitis. A transverse colectomy (D) targets lesions in the transverse colon specifically. Thus, right hemicolectomy is the standard procedure for cecal adenocarcinoma to achieve clear margins and an adequate lymph node harvest.

92. A 20-year-old man presents with acute severe (A) Creation of a peritoneal flap over the inguinal
testicular pain for 3 hours, high-riding testis, and region
absent cremasteric reflex. Doppler ultrasound (B) Reduction of the hernia sac into the abdominal
suggests testicular torsion. Best next step? cavity
(A) Manual detorsion attempt in the emergency (C) Placement of mesh covering the myopectineal
department followed by surgical exploration orifice
(B) Immediate surgical exploration without delay (D) Leaving large unclosed peritoneal defects
(C) Antibiotic therapy for presumed epididymitis behind the mesh
(D) MRI of the pelvis (D) Leaving large unclosed peritoneal defects behind the mesh
Transabdominal preperitoneal (TAPP) repair involves entering the peritoneal cavity, creating a peritoneal flap, and dissecting the inguinal region to expose the hernia. After the sac is reduced (B), a prosthetic mesh is placed over the myopectineal orifice (C), covering both the direct and indirect spaces. The peritoneum is then closed over the mesh to prevent direct contact between mesh and bowel. Leaving large unclosed peritoneal defects (D) risks mesh migration, adhesion formation, or bowel entrapment. Therefore, thorough closure of the peritoneal flap is important. Steps (A), (B), and (C) are correct standard components of TAPP. Hence, leaving a significant peritoneal defect is the incorrect step.

97. In a population with a low prevalence of a disease,


(B) Immediate surgical exploration without delay
In testicular torsion, time is critical because salvage rates dramatically decrease after 6 hours of compromised blood flow. Although manual detorsion (A) can be attempted, it should not delay definitive surgical intervention. A scrotal exploration in the operating room allows direct visual correction of the torsion, assessment of testicular viability, and bilateral orchiopexy to prevent recurrence. Antibiotic therapy (C) addresses epididymo-orchitis, which typically has a more gradual onset and might show increased Doppler flow due to inflammation. MRI of the pelvis (D) is not practical in acute torsion given the urgent need to restore circulation. Therefore, emergent surgical exploration remains the gold standard to preserve testicular function. If the torsion is confirmed, prompt detorsion and fixation are performed to optimize salvage chances.f

93. In the management of diabetic foot ulcers, which which is most likely to happen with a screening test
statements are correct? that has good sensitivity but moderate specificity?
1. Optimal glycemic control is crucial. (A) The positive predictive value will be high
2. Vascular assessment and possible (B) The negative predictive value will be low
revascularization can be necessary. (C) The positive predictive value will be low
3. Broad-spectrum antibiotics are rarely needed. (D) The test results will be unaffected by
4. Debridement of necrotic tissue is essential. prevalence
Options : (C) The positive predictive value will be low
When a disease has low prevalence in the population, even a test with good sensitivity and modest specificity will yield a relatively high number of false positives compared to true positives, resulting in a lower positive predictive value (PPV). Although the test may correctly identify most of those with the disease (due to high sensitivity), the total number of positive tests will include many individuals who are disease-free. Negative predictive value (NPV) generally rises in low-prevalence settings, so (B) is incorrect. The positive predictive value (A) does not become high; instead, it tends to drop. Test results are indeed influenced by disease prevalence (D), so that statement is incorrect. Thus, in a low-prevalence environment, a test with moderate specificity typically yields a decreased PPV.

(A) 1, 2, and 3 are correct


(B) 1, 2, and 4 are correct 98. Why does a direct inguinal hernia occur medial to
(C) 2 and 3 are correct the inferior epigastric vessels?
(D) 1, 2, 3, and 4 are correct (A) Failure of processus vaginalis closure
(B) Weakness in Hesselbach’s triangle
(C) Unobliterated vitelline duct
(B) 1, 2, and 4 are correct
Diabetic foot ulcer management necessitates meticulous control of blood glucose (1) to support wound healing and reduce infection risk. Vascular assessment (2) is essential because peripheral arterial disease frequently coexists, and revascularization procedures can facilitate ulcer healing by restoring adequate blood supply. Debridement of necrotic or infected tissue (4) is a cornerstone of local wound care to promote a healthy granulation base. Broad-spectrum antibiotics (3) may indeed be required if there is evidence of infection, which is common in diabetic foot wounds; thus, saying they are “rarely needed” is incorrect. Antibiotic therapy is often guided by the severity of infection and cultures. Consequently, statements 1, 2, and 4 are true, making (B) the correct choice.

94. A postoperative patient develops fever, subphrenic (D) Increased intra-abdominal pressure alone
pain, and referred shoulder pain (irritation of the (B) Weakness in Hesselbach’s triangle
A direct inguinal hernia protrudes through the posterior wall of the inguinal canal in an area bounded by the rectus abdominis medially, the inferior epigastric vessels laterally, and the inguinal ligament inferiorly — commonly referred to as Hesselbach’s triangle. Over time or under conditions of raised intra-abdominal pressure, a weakened or thinned transversalis fascia in this region allows abdominal contents to bulge directly forward, medial to the inferior epigastric vessels. A failure of processus vaginalis closure (A) is associated with indirect inguinal hernias, which protrude lateral to the inferior epigastric vessels via the deep inguinal ring. An unobliterated vitelline duct (C) leads to Meckel’s diverticulum, unrelated to direct hernia formation. Although increased intra-abdominal pressure (D) can exacerbate hernias, the key structural cause of direct hernias is the localized weakness in Hesselbach’s triangle.

diaphragm) one week after laparotomy. Ultrasound


confirms a subphrenic abscess. What key action is 99. Match the burn wound depth classification with its
needed? key characteristic:
(A) Broad-spectrum antibiotics alone Column I Column II
(B) Percutaneous or surgical drainage of the 1. Superficial (1st a. Involves epidermis
abscess degree) only, painful,
(C) Immediate laparotomy exploration in all erythematous
cases 2. Superficial partial b. Extends into entire
(D) High-dose corticosteroids thickness (2nd dermis, may appear
(B) Percutaneous or surgical drainage of the abscess
Subphrenic abscesses can arise after abdominal surgeries, particularly if there is contamination or inadequate drainage. Clinical features include fever, localized pain, and possibly shoulder tip pain via diaphragmatic irritation of the phrenic nerve. While broad-spectrum antibiotics (A) are critical in controlling infection, an established abscess cavity generally requires drainage for resolution. If anatomically feasible, image-guided percutaneous drainage is often preferred; otherwise, surgical drainage may be necessary. Routine laparotomy (C) for all abscesses is more invasive and not always mandatory if less invasive techniques succeed. High-dose corticosteroids (D) would impair immune responses and worsen infection control. Therefore, the key action in managing a subphrenic abscess, besides antibiotics, is to drain the collection, typically percutaneously if safe access is confirmed by imaging.

degree) leathery
95. A surgical trial reports a “positive predictive value” 3. Deep partial c. Blisters, painful,
thickness (2nd extends into upper
(PPV) of 90 for a test detecting breast cancer.
degree) dermis
Which statement is correct?
(A) 90 of those who test negative are disease-free 4. Full thickness (3rd d. Damage into deeper
(B) 90 of those who test positive actually have degree) dermis, may have
less pain
breast cancer
page 16 NEET PG Surgery

Select the correct answer using the code given


below:
(A) 1-c, 2-d, 3-a, 4-b
(B) 1-a, 2-c, 3-d, 4-b
(C) 1-b, 2-a, 3-c, 4-d
(D) 1-d, 2-b, 3-a, 4-c
(B) 1-a, 2-c, 3-d, 4-b
Superficial (first-degree) burns (1-a) involve only the epidermis, causing painful erythema (like sunburn) without blistering. Superficial partial thickness (2-c) extends into the upper dermis, presenting with blisters, significant pain, and a moist, red wound bed. Deep partial thickness (3-d) reaches deeper dermal layers, often accompanied by reduced pain due to nerve ending damage and a paler or mottled appearance. Finally, full thickness burns (4-b) involve the entire dermis (and potentially deeper tissues), often appearing leathery, white, or charred, with minimal pain because of destroyed nerve endings. Other matchings either incorrectly map the burn depths or swap characteristics, so 1-a, 2-c, 3-d, 4-b is the correct alignment.

100. Question 100. Assertion (A) : Patients with


familial adenomatous polyposis (FAP) often
undergo prophylactic colectomy.
Reason (R) : Hundreds to thousands of colonic
polyps develop in FAP, with a near 100 risk of
malignancy if untreated.
(A) Both A and R are true, and R is the correct
explanation of A
(B) Both A and R are true, but R is not the correct
explanation of A
(C) A is true, R is false
(D) A is false, R is true
(A) Both A and R are true, and R is the correct explanation of A
Familial adenomatous polyposis (FAP) is characterized by the development of numerous adenomatous polyps in the colon and rectum at a young age. Without prophylactic colectomy or proctocolectomy, the lifetime risk of colorectal cancer approaches 100, making surgery a standard preventive measure. The reason (R) correctly states that hundreds to thousands of polyps develop in FAP, inevitably progressing to malignancy if left unaddressed. Consequently, prophylactic removal of the colon (and rectum in many cases) is indicated to prevent cancer. The assertion (A) that patients often undergo prophylactic colectomy is true, and (R) provides the rationale behind it. Thus, both statements are correct, and (R) explains (A) appropriately.

 ******
MEDINK GRAND TEST 02

GRAND TEST 02

1. Which term describes the presence of gas-forming 6. In the management of acute cholecystitis, which
bacteria in tissue leading to crepitus? step is considered the most appropriate initial
(A) Gas abscess treatment?
(B) Gas gangrene (A) Immediate open cholecystectomy
(C) Pyomyositis (B) Broad-spectrum antibiotics and supportive
(D) Necrotizing fasciitis care
(B) Gas gangrene
This term specifically indicates a severe, rapidly spreading infection caused by gas-forming bacteria, most commonly Clostridium perfringens, leading to tissue destruction and crepitus. Clinically, patients present with severe pain, swelling, and foul-smelling discharge. Palpation often reveals a characteristic crackling sensation due to subcutaneous gas. The pathophysiology involves bacterial exotoxins and proteolytic enzymes that destroy muscle tissue and release gas. Immediate surgical debridement, antibiotic therapy, and supportive measures are crucial for management. Hyperbaric oxygen may also be considered to inhibit anaerobic bacterial growth. Option (A) Gas abscess is not a standard term for this rapidly progressive process. Option (C) Pyomyositis involves purulent muscle infection but typically lacks significant gas production. Option (D) Necrotizing fasciitis affects fascia and subcutaneous tissue more extensively, though it can sometimes generate gas, it is not the classic term for clostridial myonecrosis. Gas gangrene truly remains the definitive description.
(C) Endoscopic retrograde cholangiopan
creatogra phy (ERCP)
2. Which antibiotic is most commonly associated (D) Insertion of a percutaneous drain
with causing pseudomembranous colitis?
(A) Metronidazole
(B) Broad-spectrum antibiotics and supportive care
In acute cholecystitis, the initial approach includes pain control, intravenous fluids, and antibiotic therapy aimed at the most common pathogens, typically gram-negative and anaerobic bacteria. This step helps stabilize the patient and control the infection before definitive intervention. An urgent cholecystectomy is often planned within 72 hours once the inflammation has begun to subside, either laparoscopically or as an open procedure if the anatomy is challenging. However, it is typically not performed immediately without initial stabilization, making option (A) less appropriate. ERCP (C) is primarily indicated if there is suspicion of choledocholithiasis or cholangitis, not routine acute cholecystitis. Percutaneous cholecystostomy (D) may be performed in critically ill patients who are poor surgical candidates. Thus, broad-spectrum antibiotics and supportive management is the best first step, followed by early laparoscopic cholecystectomy in most stable patients.

(B) Vancomycin 7. A 45-year-old male presents with a painless


(C) Clindamycin swelling in the right groin region that enlarges on
(D) Piperacillin-Tazobactam standing and reduces spontaneously when lying
(C) Clindamycin
This antibiotic is historically associated with an elevated risk of pseudomembranous colitis caused by an overgrowth of Clostridioides difficile in the colon. Although many antibiotics can predispose to C. difficile infection, clindamycin is recognized for its particular propensity to disrupt normal gut flora. Clinically, patients present with watery diarrhea, abdominal pain, and fever. Endoscopic examination can reveal pseudomembranes along the colonic mucosa. Diagnosis typically involves detection of toxins in the stool. Management includes discontinuing the offending antibiotic and initiating targeted therapy such as oral vancomycin or fidaxomicin. Option (A) Metronidazole was previously used as a first-line treatment but is less favored currently. Option (B) Vancomycin can actually be used to treat C. difficile infection rather than cause it. Option (D) Piperacillin-Tazobactam has been implicated in C. difficile infection but is generally less notorious than clindamycin for this adverse effect. Hence, (C) is correct.
down. What is the most likely diagnosis?
(A) Direct inguinal hernia
3. A 50-year-old male with a 30-year history of (B) Indirect inguinal hernia
smoking presents with a chronic cough and (C) Femoral hernia
hemoptysis. Imaging reveals a mass in the apex (D) Epigastric hernia
of the lung. Which syndrome is most commonly
associated with this location?
(B) Indirect inguinal hernia
This type of hernia passes through the deep inguinal ring, along the inguinal canal, and can protrude into the scrotum. It is more common than direct inguinal hernias and tends to present as a groin swelling that increases in size with increased intra-abdominal pressure, such as standing or coughing. The swelling often reduces spontaneously when lying down. Indirect inguinal hernias occur due to a persistent processus vaginalis or a weakness in the transversalis fascia at the internal ring. In contrast, a direct inguinal hernia (A) protrudes through Hesselbach’s triangle and rarely descends into the scrotum. Femoral hernias (C) emerge below the inguinal ligament through the femoral canal, typically more common in females, and can present with a higher risk of strangulation. Epigastric hernias (D) occur in the midline between the xiphoid process and umbilicus, not in the groin region.

(A) Horner’s syndrome 8. Arrange the following steps of primary survey


(B) Superior vena cava syndrome in ATLS (Advanced Trauma Life Support) in the
(C) Carcinoid syndrome correct sequence:
(D) Lambert-Eaton myasthenic syndrome 1. Airway maintenance with cervical spine
(A) Horner’s syndrome
A Pancoast tumor, which arises in the apex of the lung, can invade nearby structures, specifically the sympathetic chain and stellate ganglion, resulting in ptosis, miosis, and anhidrosis. The location at the lung apex places it in close proximity to the lower part of the brachial plexus and sympathetic ganglia, leading to this characteristic triad. Chronic smoking significantly increases the risk of such tumors, and hemoptysis plus a chronic cough raise the suspicion of malignancy. In addition, these tumors can cause shoulder or arm pain if they invade local nerve roots. Option (B) Superior vena cava syndrome typically involves compression of the SVC, often by tumors in the right upper lobe or mediastinum, but not specifically at the apex. Option (C) Carcinoid syndrome arises from neuroendocrine tumors, generally not typical of a Pancoast lesion. Option (D) Lambert-Eaton myasthenic syndrome is associated with small cell lung cancer, not specifically apical tumors.
protection
2. Disability (neurological status evaluation)
4. All of the following are signs of acute arterial 3. Circulation with hemorrhage control
occlusion of a limb EXCEPT: 4. Breathing and ventilation
(A) Pain Options :
(B) Pallor (A) 1-2-3-4
(C) Pulselessness (B) 1-4-3-2
(D) Pitting edema (C) 2-1-3-4
(D) Pitting edema
The classic signs of acute arterial occlusion include the “six Ps”: Pain, Pallor, Pulselessness, Paresthesia, Paralysis, and Poikilothermia (coolness). These symptoms result from a sudden loss of blood flow to the affected limb, which jeopardizes tissue viability. Pain is often severe and may be accompanied by profound pallor and an absence of distal pulses. Paresthesia or numbness can occur as nerve tissue becomes ischemic, and muscle weakness or paralysis can develop. Coolness of the limb is also common due to reduced perfusion. In contrast, pitting edema is more indicative of venous or lymphatic compromise where fluid accumulates in the tissues. Acute arterial occlusion typically does not produce significant edema initially. Instead, the limb may appear shrunken or wrinkled due to ischemic changes. Hence, among the listed signs, pitting edema is the exception, making (D) the correct answer to this negative-based question.

(D) 3-1-4-2
5. Which surgical site infection is more likely to
(B) 1-4-3-2
In the primary survey of ATLS, the priorities are addressed using the ABCDE approach. First (1) Airway maintenance with cervical spine protection is critical to ensure a patent airway and prevent cervical spinal injury. Next (4) Breathing and ventilation are assessed to confirm adequate oxygenation and identify any life-threatening thoracic injuries such as tension pneumothorax or massive hemothorax. Following this, (3) Circulation with hemorrhage control is paramount to address hypovolemia from blood loss and maintain adequate tissue perfusion. Intravenous access, fluid resuscitation, and hemorrhage control are key interventions here. Finally, (2) Disability involves a rapid neurological evaluation, typically using the AVPU scale (Alert, Verbal, Pain, Unresponsive) or the Glasgow Coma Scale to detect significant brain or spinal cord injuries. By adhering to this sequence — Airway, Breathing, Circulation, Disability — life-threatening issues are addressed in an organized manner, improving patient outcomes in trauma settings.

present early with severe pain and systemic 9. A male patient has a chronic non-healing ulcer
toxicity? over the medial malleolus. On examination, there
(A) Cellulitis is hyperpigmentation around the area, and the ulcer
(B) Superficial surgical site infection is shallow with irregular borders. Which feature is
(C) Necrotizing fasciitis most crucial in identifying this ulcer type?
(D) Local abscess (A) Varicose veins in the leg
(C) Necrotizing fasciitis
This rapidly progressing, life-threatening infection involves the fascial planes and subcutaneous tissues, leading to severe pain, systemic toxicity, and necrosis. Patients often appear acutely ill, with signs of sepsis such as fever, tachycardia, and hypotension. The hallmark is intense pain out of proportion to examination findings, quickly followed by skin discoloration, crepitus from gas in tissue, and foul-smelling discharge. Surgical intervention for debridement is urgent because delay increases mortality. By contrast, cellulitis (A) is more superficial, presenting with erythema and warmth over a longer period. Superficial surgical site infections (B) are typically localized, show purulent drainage, and respond to antibiotics and local wound care. A local abscess (D) can cause pain and tenderness but lacks the rapidly advancing necrosis and toxicity characteristic of necrotizing fasciitis. Thus, necrotizing fasciitis presents early with severe systemic features and requires immediate aggressive management.

(B) Peripheral neuropathy


(C) Reduced dorsalis pedis pulse
(D) Palpable pedal pulses with normal ABPI
page 18 NEET PG Surgery

(B) Fine needle aspiration biopsy (FNAB)


(A) Varicose veins in the leg
A chronic ulcer over the medial malleolus with surrounding hyperpigmentation is strongly indicative of a venous stasis ulcer. These ulcers often arise from chronic venous insufficiency, where faulty valves or venous outflow obstruction lead to elevated venous pressure and subsequent skin changes. Varicose veins, representing dilated superficial veins, are commonly associated with chronic venous insufficiency and serve as a key clinical feature. In contrast, peripheral neuropathy (B) typically leads to neuropathic ulcers on pressure points of the foot, as seen in diabetic foot disease. Reduced dorsalis pedis pulse (C) suggests arterial disease, more likely to cause ulcers on the lateral malleolus or toes. Palpable pedal pulses with normal ankle-brachial pressure index (ABPI) (D) can still be consistent with venous ulcers but does not specifically indicate an essential clinical feature like varicose veins. Hence, (A) is the most crucial feature.

10. A 32-year-old woman complains of a breast lump (C) Thyroid scan using radioactive iodine
that has been slowly enlarging over six months. On (D) MRI of the neck
examination, it is mobile, well-defined, and non- (B) Fine needle aspiration biopsy (FNAB)
In evaluating a suspicious solitary thyroid nodule, the best initial step is cytological assessment via FNAB. This procedure is minimally invasive, safe, and provides valuable information to distinguish benign from malignant lesions. Ultrasound features that raise suspicion include microcalcifications, irregular margins, taller-than-wide shape, and increased nodular vascularity. Serum thyroglobulin (A) is not a routine initial diagnostic test for a solitary nodule because it lacks the specificity to differentiate malignant from benign nodules. A thyroid scan (C) using radioactive iodine may help identify a hyperfunctioning (“hot”) nodule, which is less likely to be malignant, but modern guidelines typically recommend FNAB first for suspicious ultrasound findings. MRI of the neck (D) can evaluate local extension or lymph node involvement but is not the primary test for initial diagnosis. Thus, FNAB is the most appropriate initial investigation.

tender. Mammography suggests a benign lesion.


What is the most appropriate next step? 16. A 65-year-old with a long history of alcohol
(A) Excisional biopsy consumption presents with progressive dysphagia
(B) Core needle biopsy to solids, weight loss, and a mid-esophageal mass
(C) Observation for six more months on endoscopy. What is the most likely diagnosis?
(D) Fine needle aspiration cytology (A) Esophageal adenocarcinoma
(B) Esophageal squamous cell carcinoma
(C) Benign esophageal stricture
(B) Core needle biopsy
For a suspicious breast lump, even if imaging suggests a benign lesion, a tissue diagnosis is essential to rule out malignancy. Core needle biopsy provides a sample of tissue architecture and is more accurate than fine needle aspiration cytology (D), which often yields only cells. In a woman of this age, with a slowly enlarging, well-defined lump, the most common benign possibility is a fibroadenoma. However, histopathological confirmation is prudent to exclude atypical or malignant changes. Excisional biopsy (A) may be performed if core biopsy findings are inconclusive or if the patient desires removal, but it is not the first diagnostic approach. Observation for six more months (C) might delay a potential diagnosis if there is any possibility of malignancy. Therefore, a core needle biopsy balances the need for diagnosis with minimal invasiveness, making (B) the most appropriate next step.

11. Which fluid is preferred for initial resuscitation in (D) Achalasia cardia
hemorrhagic shock according to Advanced Trauma (B) Esophageal squamous cell carcinoma
In patients with a history of chronic alcohol and tobacco use, squamous cell carcinoma is a common malignancy affecting the mid-esophagus. Progressive dysphagia to solids and significant weight loss are hallmark symptoms, reflecting the tumor’s obstructive impact. On endoscopic evaluation, squamous cell carcinomas often appear as ulcerative or exophytic lesions. Adenocarcinoma (A) typically involves the distal esophagus and is associated with Barrett’s esophagus from chronic gastroesophageal reflux. A benign esophageal stricture (C) might also cause dysphagia but would less likely present with rapid weight loss or a mass lesion. Achalasia cardia (D) is a motility disorder characterized by failure of the lower esophageal sphincter to relax, usually presenting with progressive dysphagia to solids and liquids, not typically forming a distinct mass. Thus, squamous cell carcinoma remains the most likely diagnosis in a long-term alcohol consumer with a mid-esophageal mass.

Life Support (ATLS) guidelines?


(A) 0.9 Normal saline 17. Arrange the following layers of the anterior
(B) Half-normal saline abdominal wall from superficial to deep:
(C) 5 Dextrose in water 1. External oblique muscle
(D) Ringer’s lactate 2. Transversalis fascia
3. Internal oblique muscle
4. Transversus abdominis muscle
(D) Ringer’s lactate
ATLS guidelines recommend balanced crystalloid solutions, such as Ringer’s lactate, as the initial fluid of choice in hemorrhagic shock. Ringer’s lactate has an electrolyte composition that more closely mirrors that of plasma, helping maintain acid-base balance better than 0.9 normal saline. Excessive normal saline can lead to hyperchloremic metabolic acidosis. Ringer’s lactate also contains lactate, which is metabolized in the liver to bicarbonate, potentially mitigating acidosis. Half-normal saline (B) provides inadequate sodium and can worsen hemodynamic instability if used alone in significant hemorrhage. Five percent dextrose in water (C) is a hypotonic solution that is not suitable for volume replacement in acute blood loss because it quickly shifts into the intracellular compartment. Early blood product transfusion is also essential in severe hemorrhage, but for initial rapid fluid resuscitation, Ringer’s lactate remains a preferred balanced crystalloid choice according to standard ATLS protocols.

12. A 35-year-old male is admitted with severe Select the option for correct order :
acute pancreatitis. Which local complication is (A) 1-3-4-2
characterized by a collection of pancreatic fluid (B) 1-4-3-2
enclosed by a wall of fibrous or granulation tissue? (C) 3-1-4-2
(A) Pancreatic pseudocyst (D) 4-3-2-1
(B) Pancreatic abscess (A) 1-3-4-2
When moving from superficial to deep in the lateral portion of the anterior abdominal wall, the first muscle layer encountered is the external oblique (1), recognized by fibers running inferomedially (like hands in pockets). Beneath it lies the internal oblique (3) with fibers oriented roughly perpendicular to those of the external oblique. The next layer is the transversus abdominis muscle (4), which has fibers running horizontally. Finally, the transversalis fascia (2) lines the inner surface of the abdominal wall, lying just external to the peritoneum. Understanding these layers is essential for surgeries such as hernia repairs and incisions, ensuring correct identification and closure of each plane. The arrangement 1-3-4-2 reflects external oblique → internal oblique → transversus abdominis → transversalis fascia, which is the standard anatomical sequence in most of the abdominal wall lateral to the rectus sheath.

(C) Acute peripancreatic fluid collection


(D) Chronic pancreatitis calcification 18. A 60-year-old male with a history of chronic
gastroesophageal reflux disease (GERD)
complains of progressive dysphagia. Upper GI
(A) Pancreatic pseudocyst
In cases of severe acute pancreatitis, a pseudocyst can form as a collection of pancreatic juice enclosed by a wall of fibrous or granulation tissue, typically lacking a true epithelial lining. It usually arises after four weeks from the onset of pancreatitis. Patients may present with persistent abdominal pain, a palpable mass, or symptoms caused by compression of adjacent structures. Imaging studies such as CT scan can confirm a fluid-filled collection. Management depends on symptoms and size; asymptomatic pseudocysts under 6 cm in diameter often regress spontaneously, while larger or symptomatic ones may require drainage. Pancreatic abscess (B) is an infected collection that generally appears later and exhibits signs of infection. Acute peripancreatic fluid collection (C) occurs earlier without a fibrous capsule. Chronic pancreatitis calcification (D) is seen in long-standing disease and does not describe an acute fluid collection.

13. All of the following are used to assess the severity endoscopy shows a distal esophageal stricture with
of acute pancreatitis EXCEPT: areas of columnar epithelium. Which key feature
(A) Ranson’s criteria differentiates Barrett’s esophagus from a simple
(B) Glasgow score peptic stricture?
(C) APACHE II score (A) Ulceration in the lower esophagus
(D) MELD score (B) Presence of intestinal metaplasia
(C) Esophageal webs
(D) Linear erosions in the distal esophagus
(D) MELD score
Ranson’s criteria (A), Glasgow score (B), and the APACHE II score (C) are common tools used to evaluate the severity of acute pancreatitis and predict patient outcomes. Ranson’s criteria include multiple clinical and laboratory parameters measured at admission and after 48 hours to gauge disease progression. The Glasgow score (also known as Imrie’s score) assesses biochemical and clinical factors to classify the severity. The APACHE II (Acute Physiology And Chronic Health Evaluation II) score is a widely used ICU scoring system that can be applied to patients with acute pancreatitis. In contrast, the MELD (Model for End-Stage Liver Disease) score is primarily utilized for assessing the severity of chronic liver disease and prioritizing liver transplantation; it is not routinely applied to acute pancreatitis. Therefore, MELD is the exception among these indices for evaluating acute pancreatitis severity, making option (D) correct.

14. Which type of shock is primarily characterized by (B) Presence of intestinal metaplasia
Barrett’s esophagus is characterized by the replacement of the normal squamous epithelium of the distal esophagus with columnar epithelium that contains goblet cells, a hallmark of intestinal metaplasia. Chronic GERD contributes to this metaplastic change, which predisposes to esophageal adenocarcinoma if left unmonitored. A peptic stricture may form from prolonged acid injury but does not necessarily involve intestinal metaplasia. Barrett’s epithelium appears salmon-colored on endoscopy, whereas simple erosive esophagitis or strictures may not exhibit the same distinctive mucosal change. Ulceration (A) is nonspecific and can be present in various conditions. Esophageal webs (C) are thin membranes found in the upper or mid-esophagus (e.g., Plummer-Vinson syndrome), unrelated to Barrett’s. Linear erosions (D) are seen in reflux esophagitis but do not confirm metaplasia. Therefore, the presence of intestinal metaplasia specifically differentiates Barrett’s esophagus from a simple peptic stricture.

a decrease in systemic vascular resistance due to


massive vasodilation? 19. In a 60-year-old female who presents with a
(A) Hypovolemic shock suspicious breast lesion on mammogram, which
(B) Cardiogenic shock is the most definitive next step to confirm the
(C) Neurogenic shock diagnosis?
(D) Obstructive shock (A) Digital breast tomosynthesis
(B) Core needle biopsy
(C) Breast MRI
(C) Neurogenic shock
This form of distributive shock occurs when there is a loss of sympathetic tone, as commonly seen in high spinal cord injuries or certain anesthetic complications. The hallmark is significant vasodilation, leading to reduced systemic vascular resistance and hypotension. Additionally, bradycardia may be observed because of unopposed parasympathetic activity. In contrast, hypovolemic shock (A) is caused by reduced intravascular volume, commonly from hemorrhage or fluid losses, and features increased systemic vascular resistance as the body attempts to maintain blood pressure. Cardiogenic shock (B) arises from the heart’s inability to pump effectively, resulting in decreased cardiac output. Obstructive shock (D) involves impaired cardiac filling or outflow due to conditions such as tension pneumothorax or pulmonary embolism. Thus, neurogenic shock differs in its characteristic loss of sympathetic tone and resultant vasodilation, making (C) the correct choice.

15. A 40-year-old female has a solitary thyroid nodule (D) Fine needle aspiration cytology
with suspicious features on ultrasound. Which is (B) Core needle biopsy
When imaging (mammogram) shows suspicious features, obtaining a histopathological diagnosis is critical, and core needle biopsy is generally regarded as the gold standard. It allows pathologists to examine tissue architecture and determine receptor status (e.g., ER, PR, HER2) for planning therapy if malignant. Digital breast tomosynthesis (A) can provide additional imaging detail but does not confirm histology. Breast MRI (C) can help in further characterization of lesions or to evaluate multifocal disease, but it is not the definitive diagnostic step. Fine needle aspiration cytology (D) offers cellular information but lacks the architectural detail core biopsy provides, leading to possible diagnostic ambiguity. Therefore, core needle biopsy is the most definitive next step for confirming malignancy and guiding subsequent management. This approach aligns with current guidelines for the workup of suspicious mammographic findings.

the best initial diagnostic test?


(A) Measurement of serum thyroglobulin
MEDINK GRAND TEST 02

20. A patient with confirmed papillary thyroid 25. A 55-year-old diabetic arrives with swelling in
carcinoma limited to one lobe of the thyroid has no the perineum, severe pain, fever, and crepitus on
evidence of extrathyroidal extension or metastasis. palpation. The scrotum and perianal region are
What is the most definitive surgical treatment? involved. Which diagnosis fits best?
(A) Hemithyroidectomy alone (A) Hidradenitis suppurativa
(B) Total thyroidectomy (B) Fournier’s gangrene
(C) Total thyroidectomy plus modified radical (C) Perianal abscess
neck dissection (D) Necrotizing fasciitis of the abdominal wall
(D) Subtotal thyroidectomy (B) Fournier’s gangrene
This rapidly progressing necrotizing fasciitis involves the perineum, scrotum, and sometimes extends to the anterior abdominal wall. It is particularly common in immunocompromised patients like diabetics. Clinical features include severe pain, edema, erythema, crepitus from gas-forming organisms, and systemic signs of sepsis. Hidradenitis suppurativa (A) involves chronic inflammation of apocrine glands, typically in the axilla, groin, or perianal area, but does not present acutely with extensive necrosis and gas formation. A perianal abscess (C) can be painful and may spread but usually is more localized, lacking the diffuse necrotizing pattern seen in Fournier’s gangrene. Necrotizing fasciitis of the abdominal wall (D) typically spares the scrotum and perineum unless it secondarily spreads. Hence, the acute, severe infection in the perineum with scrotal involvement and crepitus strongly suggests Fournier’s gangrene.

26. A 42-year-old male smoker presents with a


(B) Total thyroidectomy
Although papillary thyroid carcinoma may be small and confined to one lobe, current guidelines often recommend total thyroidectomy in most cases larger than 1 cm. Total thyroidectomy helps reduce recurrence risk in the remaining thyroid tissue and facilitates monitoring with serum thyroglobulin. It also allows for the use of radioactive iodine ablation if needed postoperatively. Hemithyroidectomy (A) may be an option for very small papillary carcinomas (e.g., less than 1 cm, low-risk), but for the typical case, total thyroidectomy is more definitive. Modified radical neck dissection (C) is indicated if there is evidence of lymph node metastasis. Subtotal thyroidectomy (D) leaves behind more tissue than recommended, possibly increasing recurrence risk. Therefore, total thyroidectomy remains the most definitive surgical step in appropriately selected patients, offering better long-term surveillance and management of papillary thyroid cancer.

21. A 55-year-old male smoker presents with left- chronic, non-healing ulcer on the left foot. He
sided hemiparesis, headache, and a pulsatile reports intermittent claudication and rest pain for
scalp swelling near the temporal region. Possible the past year. Examination shows absent dorsalis
diagnosis? pedis pulse. Arterial Doppler confirms decreased
(A) Meningioma blood flow to the foot. Which surgical procedure is
(B) Subdural hematoma often performed?
(C) Middle meningeal artery aneurysm (A) Femoro-popliteal bypass
(D) Pott’s puffy tumor (B) Endarterectomy of the carotid
(C) Middle meningeal artery aneurysm
The clinical vignette indicates a pulsatile swelling in the temporal region, headache, and neurological deficits (left-sided hemiparesis), raising the suspicion of an intracranial or dural vascular lesion. Aneurysms involving the middle meningeal artery can present with pulsatile scalp swellings, and the associated pressure effects or hemorrhage can lead to focal neurological signs. Although meningiomas (A) can cause headaches and focal deficits, they generally do not produce a pulsatile external swelling. Subdural hematoma (B) can induce hemiparesis and headache, but it typically does not create a pulsatile mass on the scalp. Pott’s puffy tumor (D) is a subperiosteal abscess associated with osteomyelitis of the frontal bone, often due to sinus infections, not localized to the temporal region or related to vascular anomalies. Therefore, a middle meningeal artery aneurysm best explains the pulsatile nature and neurological symptoms.
(C) Fasciotomy of the foot compartments
(D) Sclerotherapy of leg veins
22. A 60-year-old with dysphagia and odynophagia
has a circumferential malignant esophageal lesion
(A) Femoro-popliteal bypass
In patients with chronic limb ischemia due to occlusive arterial disease, particularly involving the superficial femoral artery, a bypass graft from the femoral to the popliteal artery can restore adequate blood flow to the lower limb. Clinical features such as intermittent claudication, rest pain, and a non-healing foot ulcer point to significant arterial compromise. Absent pedal pulses and decreased flow on Doppler studies confirm severe peripheral artery disease. Carotid endarterectomy (B) is performed to address carotid stenosis and prevent stroke, not peripheral ischemia. Fasciotomy (C) decompresses compartments in acute compartment syndrome but does not revascularize the limb. Sclerotherapy of leg veins (D) targets varicose veins and does not correct arterial blockages. Hence, femoro-popliteal bypass is the standard revascularization procedure for occlusive disease affecting the superficial femoral artery region, making (A) correct.

on endoscopy causing a near-complete luminal 27. A 68-year-old female with a longstanding


obstruction. What best describes this? hiatus hernia experiences worsening heartburn,
(A) Diffuse esophageal spasm regurgitation, and chest pain. Endoscopy reveals
(B) Esophageal stricture a large paraesophageal hernia with significant
(C) Napkin-ring carcinoma displacement of the stomach into the thorax.
(D) Schatzki ring Surgical intervention is advised. Which procedure
(C) Napkin-ring carcinoma
This descriptive term refers to a constricting growth that encircles the esophagus, creating a narrowed segment resembling a “napkin ring.” Such tumors often cause progressive dysphagia, initially to solids, then to liquids, and can lead to significant luminal obstruction. Diffuse esophageal spasm (A) produces intermittent, non-peristaltic contractions resulting in chest pain and intermittent swallowing difficulties rather than a fixed obstruction. An esophageal stricture (B) may result from chronic gastroesophageal reflux or caustic injury, but the question specifically notes a “malignant” lesion. A Schatzki ring (D) is a mucosal ring typically found at the squamocolumnar junction in the lower esophagus, causing intermittent dysphagia to solids without describing a malignant ring lesion. Therefore, a circumferential malignant esophageal lesion that severely narrows the lumen aligns best with the description of napkin-ring carcinoma, making (C) the most accurate choice.
is most commonly performed?
(A) Heller myotomy
23. A 28-year-old man with severe head trauma (B) Nissen fundoplication
develops hypertension, bradycardia, and irregular (C) Belsey Mark IV repair
respirations. Neurological examination reveals (D) Dor fundoplication
papilledema. Which triad is indicated by these
clinical findings?
(B) Nissen fundoplication
In paraesophageal hernias (often Type II, III, or IV), a significant portion of the stomach herniates alongside the esophagus. Surgical repair typically involves reducing the hernia, repairing the hiatal defect, and performing an antireflux procedure to prevent gastroesophageal reflux. The Nissen fundoplication, a complete 360-degree wrap of the stomach around the distal esophagus, is a well-established technique to address GERD and prevent reherniation. Heller myotomy (A) is used in achalasia to relieve the lower esophageal sphincter. Belsey Mark IV repair (C) is a partial fundoplication performed via a thoracic approach but is less common today. The Dor fundoplication (D) is an anterior partial wrap often done in conjunction with Heller myotomy. A complete Nissen wrap is widely favored in paraesophageal hernia repairs for its proven efficacy in controlling reflux and achieving durable repair.

(A) Beck’s triad 28. A 50-year-old patient presents with painless


(B) Charcot’s triad jaundice, dark urine, pale stools, and a palpable,
(C) Cushing’s triad non-tender gallbladder. Imaging suggests a mass at
(D) Virchow’s triad the head of the pancreas obstructing the common
(C) Cushing’s triad
This triad comprises hypertension (often with a widened pulse pressure), bradycardia, and irregular respirations, typically indicating raised intracranial pressure (ICP). The mechanism involves a reflex response to maintain cerebral perfusion pressure in the face of increasing ICP. Papilledema further supports elevated ICP. Beck’s triad (A) refers to hypotension, distant heart sounds, and jugular venous distension seen in cardiac tamponade. Charcot’s triad (B) comprises right upper quadrant pain, jaundice, and fever/chills, characteristic of cholangitis. Virchow’s triad (D) encompasses stasis of blood flow, endothelial injury, and hypercoagulability, describing risk factors for thrombosis. In this head trauma scenario, Cushing’s triad signals a critical increase in intracranial pressure demanding urgent intervention. Identifying these signs aids in preventing herniation and other life-threatening complications, making (C) the correct answer.
bile duct. What is the standard surgical procedure
performed for resectable tumors in this location?
24. A 40-year-old male with a deep neck space (A) Distal pancreatectomy
infection presents with edema of the floor of (B) Whipple procedure
the mouth, elevation of the tongue, and airway (C) Total pancreatectomy
compromise. Which space is primarily involved? (D) Endoscopic stent placement
(A) Ludwig’s space
(B) Retropharyngeal space
(B) Whipple procedure
In a patient with a tumor in the pancreatic head causing obstructive jaundice, the Whipple procedure is the standard curative approach if the tumor is resectable. This complex operation typically involves surgical resection of the pancreatic head, duodenum, proximal jejunum, gallbladder, and sometimes part of the stomach, followed by reconstruction to restore gastrointestinal continuity. Distal pancreatectomy (A) is indicated for lesions in the body or tail of the pancreas. Total pancreatectomy (C) removes the entire pancreas and is reserved for extensive disease or multifocal neoplasms, rendering the patient insulin-dependent. Endoscopic stent placement (D) can palliate obstructive jaundice in unresectable or high-risk patients but is not curative. Thus, the Whipple procedure remains the definitive surgical intervention for resectable pancreatic head tumors, especially in the setting of Courvoisier’s sign (painless jaundice with a palpable gallbladder).

(C) Parapharyngeal space 29. Assertion (A) : Sentinel lymph node biopsy is a
(D) Submental space valuable technique in early breast cancer.
(A) Ludwig’s space
Ludwig’s angina is a rapidly spreading cellulitis involving the submandibular, sublingual, and submental spaces. Patients often present with swelling of the floor of the mouth, tongue elevation, and, in severe cases, airway compromise. The condition typically arises from infections of the second or third mandibular molars. Retropharyngeal space (B) infections can lead to posterior pharyngeal wall swelling, neck stiffness, and potential mediastinal extension but not the classic tongue elevation. The parapharyngeal space (C) lies lateral to the pharynx; infections can displace the tonsils medially and cause neck swelling but usually do not elevate the tongue’s floor. The submental space (D) is just one of the compartments potentially involved, but Ludwig’s angina more specifically describes simultaneous submandibular and sublingual involvement, presenting with bilateral floor-of-mouth swelling. Therefore, Ludwig’s space infection best matches the clinical picture described.
Reason (R) : It accurately stages the axilla without
full axillary dissection in all patients.
page 20 NEET PG Surgery

(A) Both A and R are true, and R is the correct below :


explanation of A (A) 1-b, 2-c, 3-d, 4-a
(B) Both A and R are true, but R is not the correct (B) 1-a, 2-b, 3-c, 4-d
explanation of A (C) 1-c, 2-d, 3-a, 4-b
(C) A is true, but R is false (D) 1-d, 2-c, 3-b, 4-a
(D) A is false, but R is true (A) 1-b, 2-c, 3-d, 4-a
Billroth I (1-b) is a gastroduodenostomy often used for duodenal ulcers when the gastric antrum is resected but direct continuity with the duodenum is maintained, helping preserve some pyloric function. Billroth II (2-c) is a gastrojejunostomy typically used when the antrum must be removed (e.g., for a gastric ulcer) and the duodenum is not suitable for anastomosis. Highly selective vagotomy (3-d) targets only the acid-producing portion of the stomach (parietal cell mass) for duodenal ulcer disease when minimal acid reduction is required; the pylorus remains innervated to reduce complications. Lastly, Roux-en-Y gastric bypass (4-a) is a bariatric procedure indicated for severe obesity, creating a small gastric pouch and bypassing a portion of the small intestine to induce weight loss. Thus, (A) correctly aligns each procedure with its primary indication.

34. Consider the following statements regarding


(C) A is true, but R is false
Sentinel lymph node biopsy (SLNB) is indeed a valuable technique in early breast cancer (Assertion is true). It helps stage the axilla by identifying the first lymph node(s) to which the tumor drains. If the sentinel node is negative for metastasis, the likelihood of further nodal involvement is low, thus avoiding a full axillary lymph node dissection (ALND) and its potential morbidities such as lymphedema. However, the Reason states that SLNB is performed “in all patients,” which is not entirely accurate. SLNB is typically reserved for patients with clinically node-negative disease. Those with palpable axillary nodes or advanced tumors often undergo more extensive evaluation. In addition, if the sentinel node is positive, further axillary surgery may be indicated depending on treatment protocols. Therefore, while SLNB is valuable, it is not universally applied to “all” axillas without proper selection criteria.

30. In the standard management protocol for acute Helicobacter pylori infection :
limb compartment syndrome, which step is 1. It is associated with chronic gastritis.
INCORRECT? 2. Eradication reduces recurrence of peptic
(A) Immediate fasciotomy ulcers.
(B) Analgesia and sedation 3. It is typically gram-positive.
(C) Elevating the limb well above heart level 4. It plays no role in gastric MALT lymphoma.
(D) Monitoring compartment pressures Which of the statements given above are correct?
(C) Elevating the limb well above heart level
In acute compartment syndrome, increased pressure within a fascial compartment compromises circulation and tissue viability. Immediate fasciotomy (A) is a correct and essential step if compartment pressures are high or clinical signs are present. Adequate analgesia and sedation (B) help control pain while preparing for surgical intervention. Monitoring compartment pressures (D) is a standard approach to confirm the diagnosis when clinical exam is inconclusive. However, elevating the limb too high above the heart reduces arterial perfusion pressure further, exacerbating ischemia in the already compromised compartment. The recommended position is at or slightly above the level of the heart to maintain adequate perfusion. Therefore, significantly raising the limb is the incorrect step in managing acute compartment syndrome and can worsen tissue hypoxia, making option (C) the right answer for the incorrect action.
(A) 1, 2, and 3 are correct
(B) 1 and 2 are correct; 3 and 4 are incorrect
31. In patients with a history of familial adenomatous (C) 2 and 4 are correct; 1 and 3 are incorrect
polyposis (FAP), the likelihood of developing (D) Only 1 is correct
colorectal cancer by age 40 if untreated is
approximately:
(B) 1 and 2 are correct; 3 and 4 are incorrect
Helicobacter pylori is known to cause chronic gastritis (statement 1) and plays a major role in peptic ulcer disease, such that eradication reduces the recurrence of these ulcers (statement 2). The organism is actually gram-negative, spiral-shaped, not gram-positive (so statement 3 is incorrect). Furthermore, H. pylori infection is closely linked to gastric MALT (mucosa-associated lymphoid tissue) lymphoma, and its eradication can lead to lymphoma regression in some cases, so statement 4, stating it plays “no role,” is false. Hence, the combination of statements 1 and 2 being true, and 3 and 4 being false, matches option (B). Understanding H. pylori’s gram-negative status and its involvement in MALT lymphoma is crucial in the clinical approach to gastric pathologies and in tailoring appropriate antibiotic therapy.

(A) 10 35. A randomized trial compares laparoscopic


(B) 50 appendectomy (LA) versus open appendectomy
(C) 80 (OA). Results show significantly shorter hospital
(D) 100 stay and less postoperative pain for LA, with a
(D) 100
Familial adenomatous polyposis (FAP) is an autosomal dominant condition characterized by the development of hundreds to thousands of colonic polyps, usually during adolescence or early adulthood. If these polyps are not removed (generally by prophylactic colectomy), the risk of malignant transformation is nearly inevitable. The APC gene mutation underlies FAP, driving the progression from adenomatous polyps to carcinoma typically by the patient’s fourth decade. Though the exact figure is close to 100, some sources quote a slightly lower but still exceptionally high percentage. Regardless, FAP patients without prophylactic surgical intervention almost invariably develop colorectal cancer. Lower percentages like 10 (A) or 50 (B) severely underestimate the near inevitability of malignant progression. An 80 risk (C) also undervalues the well-documented near-total risk. Therefore, the recognized figure is effectively 100 likelihood if untreated, making option (D) correct.
p-value of 0.04. What does p = 0.04 mean?
(A) There is a 4 chance of a Type I error if LA
32. Chronic infection with Helicobacter pylori is most truly has no benefit
strongly associated with which of the following (B) There is a 4 chance the study results are due
gastric pathologies? to random chance
(A) Gastric adenocarcinoma (C) The results are not statistically significant
(B) Zollinger-Ellison syndrome (D) LA is 4 more effective than OA
(C) Gastric leiomyoma
(D) Menetrier’s disease
(A) There is a 4 chance of a Type I error if LA truly has no benefit
In clinical research, a p-value reflects the probability of observing the study results (or more extreme ones) if the null hypothesis is true. Here, the null hypothesis would be “no difference between laparoscopic and open appendectomy.” A p-value of 0.04 indicates a 4 probability of incorrectly rejecting the null hypothesis — i.e., stating there is a difference when in fact none exists. While commonly interpreted as “the findings are unlikely due to chance,” the more precise meaning relates to the chance of a Type I error. Option (B) is an oversimplification, and (C) is incorrect because p < 0.05 is typically considered significant. Option (D) conflates p-value with effect size. Thus, the best interpretation is that there is a 4 chance of concluding LA is better when no real difference exists.

(A) Gastric adenocarcinoma


Chronic Helicobacter pylori infection causes sustained inflammation within the gastric mucosa, leading to atrophic gastritis and intestinal metaplasia, which can progress to gastric adenocarcinoma — particularly the intestinal type. H. pylori has been classified by the WHO as a class I carcinogen for gastric cancer. Zollinger-Ellison syndrome (B) is caused by gastrin-secreting tumors (gastrinomas), most often arising in the duodenum or pancreas, not by chronic bacterial infection. Gastric leiomyoma (C) is a benign smooth muscle tumor also unrelated to H. pylori. Menetrier’s disease (D) involves giant rugal folds and protein-losing gastropathy, typically without a direct link to H. pylori. Therefore, while H. pylori can contribute to peptic ulcers and MALT lymphoma, its strongest association among the provided options is with gastric adenocarcinoma, making (A) correct.
36. In the management of acute variceal bleeding,
which step is INCORRECT?
33. Match the gastric surgical procedure in Column A (A) IV vasoactive drugs (e.g., octreotide)
with its primary indication in Column B: (B) Endoscopic band ligation
Column I Column II (C) Balloon tamponade if endoscopic therapy
1. Billroth I a. Severe obesity fails
gastrectomy management (D) Immediate surgical shunt as first-line
2. Billroth II b. Duodenal ulcer with
(D) Immediate surgical shunt as first-line
Management of acute variceal bleeding focuses on stabilizing the patient and controlling the hemorrhage. IV vasoactive drugs (A), such as octreotide, help reduce splanchnic blood flow and lower portal pressure. Endoscopic band ligation (B) is the mainstay of definitive therapy for actively bleeding varices. If bleeding persists or recurs, balloon tamponade (C) may be used as a temporizing measure. Transjugular intrahepatic portosystemic shunt (TIPS) is another salvage therapy if endoscopic measures fail. Immediate surgical shunt placement, on the other hand, is not the first-line approach due to high procedural risks and the less invasive alternatives now available. Surgery is generally reserved for cases in which other measures have failed or are contraindicated. Thus, the idea of performing an immediate surgical shunt is incorrect in the standard protocol, making option (D) the correct answer.

gastrectomy need to preserve 37. What term describes the retraction ring that forms
pylorus function
in obstructed labor, often signifying imminent
3. Highly selective c. Gastric ulcer uterine rupture?
vagotomy requiring removal (A) Jackson’s ring
of antrum (B) Bandl’s ring
4. Roux-en-Y gastric d. Duodenal ulcer with (C) Schatzki ring
bypass less acid reduction (D) McDonald’s ring
needed
(B) Bandl’s ring
This term refers to the pathological retraction ring observed in prolonged or obstructed labor. As the uterus continues to contract against an obstruction, the upper segment becomes progressively thicker and shorter, while the lower segment thins and elongates, creating a visible or palpable ring. When Bandl’s ring rises toward the umbilicus, it can indicate impending uterine rupture, posing grave risks to both mother and fetus. Jackson’s ring (A) is not a recognized term in obstetrics. Schatzki ring (C) is a mucosal ring in the lower esophagus. McDonald’s ring (D) is also not related to labor obstruction. Identifying Bandl’s ring is critical for prompt obstetric intervention, typically requiring surgical delivery to prevent uterine rupture. Thus, (B) is the accurate term describing this threatening retraction ring in obstructed labor.

Select the correct answer using the code given


MEDINK GRAND TEST 02

38. Which organism is most commonly implicated in (A) Cholecystitis


gas gangrene (clostridial myonecrosis)? (B) Ureteric colic
(A) Clostridium tetani (C) Appendicitis
(B) Clostridium perfringens (D) Right ovarian torsion
(C) Clostridioides difficile (C) Appendicitis
Classic presentation includes periumbilical pain migrating to the right lower quadrant, nausea or vomiting, and low-grade fever. Physical examination reveals tenderness at McBurney’s point, often with guarding or rebound. In females of childbearing age, differentials include gynecologic causes such as ovarian torsion or pelvic inflammatory disease, but rebound tenderness localized to McBurney’s point strongly suggests appendicitis. Cholecystitis (A) typically presents with right upper quadrant pain and positive Murphy’s sign. Ureteric colic (B) may cause flank pain radiating to the groin, hematuria, and a colicky pattern. Right ovarian torsion (D) typically presents with more sudden pelvic pain, often associated with an adnexal mass or abnormal Doppler findings on ultrasound. Appendicitis remains the most likely diagnosis given the clinical scenario of localized right lower quadrant pain, classical physical signs, and low-grade fever.

(D) Clostridium botulinum


44. Arrange the layers of the scrotum from superficial
to deep:
(B) Clostridium perfringens
Gas gangrene, also known as clostridial myonecrosis, is most frequently caused by this gram-positive, spore-forming anaerobe found in soil and the human gut. When introduced into devitalized tissue (e.g., deep wounds), C. perfringens proliferates and releases potent toxins, causing rapid muscle necrosis, gas production, and systemic toxicity. Clostridium tetani (A) causes tetanus, characterized by muscular spasms but not gas gangrene. Clostridioides difficile (C) leads to pseudomembranous colitis, typically following antibiotic therapy. Clostridium botulinum (D) is responsible for botulism, associated with neuroparalysis rather than myonecrosis. Timely recognition of gas gangrene is vital; treatment includes surgical debridement, antibiotics (e.g., penicillin), and possibly hyperbaric oxygen therapy. Thus, C. perfringens remains the most common cause of clostridial myonecrosis, making (B) correct.

39. After major abdominal surgery, a patient requires 1. External spermatic fascia
nutritional support. Which feeding route preserves 2. Dartos muscle/fascia
gut integrity and reduces infectious complications, 3. Cremaster muscle/fascia
if feasible? 4. Internal spermatic fascia
(A) Total parenteral nutrition (TPN) Select the option for correct order :
(B) Central venous catheter-based nutrition (A) 2-1-3-4
(C) Enteral nutrition via nasojejunal tube (B) 1-2-3-4
(D) Peripheral parenteral nutrition (C) 1-3-2-4
(C) Enteral nutrition via nasojejunal tube
Whenever possible, enteral feeding is preferred over parenteral routes because it maintains gastrointestinal function, preserves gut mucosal integrity, and reduces bacterial translocation. This can lower the incidence of infectious complications. In contrast, total parenteral nutrition (TPN) (A) bypasses the gut entirely, which can lead to mucosal atrophy and is associated with a higher risk of catheter-related bloodstream infections. Central venous catheter-based nutrition (B) is essentially TPN delivered centrally, similarly bypassing the gut. Peripheral parenteral nutrition (D) is typically reserved for short-term, less calorically dense feeding due to the limitations of peripheral vein tolerance. While parenteral nutrition is necessary in patients who cannot tolerate enteral feeding, it is usually a second-line option. Thus, enteral nutrition — particularly via a nasojejunal tube in major abdominal surgery patients — is the ideal choice for minimizing complications and supporting gut function, making option (C) correct.
(D) 2-3-1-4

40. All of the following are recognized complications


(A) 2-1-3-4
The scrotal wall and coverings of the testis have distinct layers derived from the abdominal wall. Superficially, the skin is followed by the dartos muscle/fascia (2), responsible for wrinkling of the scrotal skin. Below this lies the external spermatic fascia (1), derived from the external oblique aponeurosis. The cremaster muscle/fascia (3), derived from the internal oblique muscle, is next. Finally, the internal spermatic fascia (4), derived from the transversalis fascia, is the deepest covering. Remembering “Some Damn Englishman Called It The Testis” (Skin, Dartos, External spermatic fascia, Cremaster, Internal spermatic fascia, Tunica vaginalis, Testis) helps. So the correct sequence from superficial to deep is dartos fascia → external spermatic fascia → cremaster muscle/fascia → internal spermatic fascia, making (A) correct.

of total parenteral nutrition (TPN) EXCEPT: 45. A 55-year-old with chronic heavy alcohol use
(A) Catheter-related bloodstream infections presents with epigastric pain radiating to the back.
(B) Hyperglycemia Imaging suggests a pseudocyst in the lesser sac.
(C) Essential fatty acid deficiency Which key imaging feature best differentiates a
(D) Increased bowel motility pseudocyst from a simple fluid collection?
(D) Increased bowel motility
Total parenteral nutrition (TPN) bypasses the gastrointestinal tract and delivers nutrients intravenously. Common complications include catheter-related bloodstream infections (A) due to indwelling central lines, metabolic derangements such as hyperglycemia (B), and nutrient imbalances. Essential fatty acid deficiency (C) can develop if lipids are not adequately provided. Rather than increasing bowel motility, TPN can lead to reduced gastrointestinal stimulation, potentially resulting in mucosal atrophy and decreased gut motility over time due to disuse. Because the bowel is not actively processing enteral feed, motility is often diminished. Thus, “increased bowel motility” is not a typical complication of TPN, making (D) the exception. Proper monitoring of blood glucose, electrolytes, and line care is crucial to minimize adverse outcomes in patients requiring prolonged TPN.
(A) Thick fibrous capsule
(B) Multiloculated compartments
41. Which imaging modality is preferred to evaluate (C) Fistulous connection to the bowel
bile duct stones when endoscopic retrograde (D) Hemorrhagic content
cholangiopancreatography (ERCP) is not
immediately indicated?
(A) Thick fibrous capsule
A pancreatic pseudocyst is not a true cyst because it lacks an epithelial lining but develops a well-defined wall of fibrous or granulation tissue around the fluid collection, usually after four weeks of ongoing pancreatitis or post-acute inflammatory changes. Imaging with CT or MRI often reveals a fluid collection with a discrete, thick wall. Multiloculated compartments (B) can occur but are not the key feature distinguishing it from simple acute fluid. Fistulous connections (C) may develop in complicated pseudocysts, though not universally. Hemorrhagic content (D) suggests a bleeding complication, which can happen but is not the main definitional characteristic. Therefore, the presence of a thick fibrous capsule is the hallmark differentiating a pseudocyst from earlier or simpler fluid collections, making (A) correct.

(A) Ultrasound of the abdomen 46. A 45-year-old male with a penetrating abdominal
(B) MRCP (Magnetic Resonance trauma is hemodynamically unstable. Focused
Cholangiopancreatography) Assessment with Sonography for Trauma (FAST)
(C) Plain X-ray abdomen is positive for free fluid. What is the most
(D) CT scan of the abdomen without contrast appropriate immediate step?
(B) MRCP (Magnetic Resonance Cholangio pancreato graphy)
MRCP is a noninvasive imaging technique that provides detailed views of the biliary and pancreatic ductal systems, making it excellent for detecting choledocholithiasis (stones in the common bile duct). Although ultrasound (A) is the first-line imaging for suspected gallbladder disease and can sometimes detect bile duct stones, its sensitivity for common bile duct stones is operator-dependent and often lower than MRCP’s sensitivity. Plain X-ray abdomen (C) is rarely useful for bile duct stones because most stones are radiolucent. A CT scan of the abdomen without contrast (D) can demonstrate stones if they are calcified, but its sensitivity for biliary calculi is limited. MRCP, on the other hand, can identify even non-calcified stones and ductal strictures. Therefore, if ERCP is not immediately warranted or unavailable, MRCP is the preferred modality to evaluate for bile duct stones.
(A) Diagnostic peritoneal lavage
(B) CT scan of the abdomen
42. In a patient with suspected perforated duodenal (C) Exploratory laparotomy
ulcer and peritonitis, which investigation provides (D) Observation in the ICU
the quickest confirmation of free intraperitoneal
air?
(C) Exploratory laparotomy
In a trauma patient with penetrating abdominal injury who is hemodynamically unstable and has a positive FAST indicating intra-abdominal fluid (likely blood), the standard of care is urgent surgical exploration. FAST exam is a rapid bedside ultrasound detecting free fluid in the pericardial or peritoneal spaces. Hemodynamic instability suggests active hemorrhage or significant organ injury requiring operative intervention. Diagnostic peritoneal lavage (A) is more of a historical test, largely replaced by FAST. A CT scan of the abdomen (B) is suitable for stable patients to delineate injuries, but it delays intervention in someone unstable. Observation in the ICU (D) would be unsafe if active bleeding is ongoing. Therefore, the appropriate action is an immediate exploratory laparotomy to control hemorrhage and repair injuries, making (C) the best choice.

(A) Supine abdominal X-ray 47. A 35-year-old male presents with severe anal pain
(B) Left lateral decubitus abdominal X-ray and bleeding on defecation. Examination reveals a
(C) Chest X-ray (erect) posterior midline anal fissure with a sentinel pile.
(D) CT scan of the abdomen Initial conservative management fails. What is the
(C) Chest X-ray (erect)
When a perforated duodenal ulcer is suspected, free air (pneumoperitoneum) can rise under the diaphragm and be best visualized on an erect chest X-ray as a crescent of air below the right hemidiaphragm. This test is rapid, widely available, and easily performed. A supine abdominal X-ray (A) is much less sensitive for detecting free air, while a left lateral decubitus abdominal X-ray (B) can be used if the patient cannot stand but is somewhat less conventional for quick diagnosis. A CT scan (D) is the most sensitive test but requires more time and resources, so in an acute setting where immediate confirmation and surgical intervention may be necessary, an erect chest X-ray remains the fastest approach to identify pneumoperitoneum. Therefore, (C) is the best answer for rapid confirmation of free intraperitoneal air.
most definitive surgical step?
(A) Fissure excision and primary repair
43. A 30-year-old female presents with acute onset of (B) Lateral internal sphincterotomy
right lower abdominal pain, nausea, and low-grade (C) Anal dilation
fever. On examination, there is rebound tenderness (D) Hemorrhoidectomy
at McBurney’s point. What is the most likely
diagnosis?
(B) Lateral internal sphincterotomy
An anal fissure most commonly occurs in the posterior midline due to increased sphincter tone. Initial management includes stool softeners, topical nitroglycerin or calcium channel blockers, and sitz baths, aiming to reduce sphincter spasm and facilitate healing. If conservative measures fail, lateral internal sphincterotomy surgically reduces internal sphincter pressure, promoting blood flow and fissure healing. Fissure excision and primary repair (A) is not as effective in the long term and can be more painful. Anal dilation (C) was once used but has higher rates of fecal incontinence. Hemorrhoidectomy (D) addresses hemorrhoids, not fissures, although sentinel piles can appear with fissures. Therefore, lateral internal sphincterotomy is the most definitive surgical treatment for a chronic anal fissure that has not responded to conservative therapy, making option (B) correct.
page 22 NEET PG Surgery

48. An elderly diabetic man complains of a foul- (A) Metastatic carcinoma of unknown origin
smelling wound on the foot with blackish (B) In-transit metastasis
discoloration of surrounding tissue. Likely (C) Occult primary tumor
diagnosis? (D) Carcinoma showing cervical node metastasis
(A) Wet gangrene with a known primary
(B) Dry gangrene (C) Occult primary tumor
Occult primary tumor, often termed “carcinoma of unknown primary” (CUP) when initially no obvious primary site is detected. In head and neck oncology, a “cervical metastasis from an unknown primary” is a not-uncommon presentation, typically manifesting as a painless neck mass. Further workup, including endoscopic evaluation and imaging, may ultimately reveal the primary lesion in the oropharynx, nasopharynx, or hypopharynx. Sometimes, tonsillar crypts or base-of-tongue submucosal malignancies can remain hidden until more advanced stages. However, once the primary is identified, it no longer fits the definition of “unknown.” Options (A) “Metastatic carcinoma of unknown origin” is a general term, but it typically applies prior to localizing the primary. In-transit metastasis (B) refers to metastatic deposits between the primary lesion and regional lymph nodes, more common in melanoma. (D) is inaccurate if the primary was initially unknown. Thus, (C) best describes the scenario of a found head-neck primary after detecting a cervical node.

(C) Gas gangrene


(D) Diabetic neuropathic ulcer 54. Which imaging modality offers the greatest
sensitivity for detecting small hepatic metastases?
(A) Ultrasound
(A) Wet gangrene
In diabetic patients, compromised blood supply and infection can lead to necrotic tissue that becomes foul-smelling, edematous, and discolored — a description consistent with wet gangrene. It typically represents a surgical emergency because the infection and toxins can spread rapidly. Dry gangrene (B) involves mummification of the tissue, typically without active infection, and is less malodorous. Gas gangrene (C) is caused by clostridial species, marked by subcutaneous crepitus, but the stem specifically highlights foul smell and blackish discoloration without explicit mention of gas production. A diabetic neuropathic ulcer (D) can become infected, yet the term “blackish discoloration” and a “foul-smelling wound” strongly suggest necrosis with infection rather than just a neuropathic ulcer. Thus, wet gangrene is the most likely diagnosis in this scenario, making (A) the correct choice.

49. A 40-year-old man experiences sudden severe (B) Dynamic contrast-enhanced CT scan
testicular pain after heavy lifting. Examination (C) MRI with liver-specific contrast
reveals a high-riding testis with a horizontal lie. (D) Plain abdominal X-ray
Best diagnosis? (C) MRI with liver-specific contrast
When searching for small hepatic metastases, MRI, particularly with gadolinium-based or hepatobiliary-specific contrast (e.g., gadoxetic acid), provides excellent soft tissue resolution and sensitivity. These specialized contrasts accumulate in normal hepatocytes but not in malignant lesions, enhancing lesion detection. Dynamic contrast-enhanced CT (B) offers good overall detection but can miss very small or subtle lesions compared to high-resolution MRI. Ultrasound (A) remains a first-line screening tool due to cost and convenience but may not detect smaller metastases, especially in obese patients or those with fatty liver. A plain abdominal X-ray (D) has very limited utility for identifying liver lesions. Thus, for detailed liver imaging, MRI with liver-specific agents is the superior modality to detect small metastatic nodules, aiding treatment planning and prognosis.

(A) Epididymitis
(B) Orchitis 55. A congenital herniation of abdominal contents
(C) Torsion of the testis through the lateral ventral abdominal wall,
(D) Testicular appendage torsion typically to the right of the umbilicus, is termed:
(A) Omphalocele
(B) Gastroschisis
(C) Torsion of the testis
Sudden onset of severe testicular pain, often precipitated by physical exertion, is classic for testicular torsion. A “high-riding” testis with a “horizontal lie” (often described as the “bell clapper deformity”) is a key clinical sign, reflecting twisting of the spermatic cord that compromises blood supply. Urgent diagnosis and surgical intervention are necessary to save the testis. Epididymitis (A) typically presents with gradual scrotal pain, pyuria, or urinary tract symptoms, and relief with scrotal elevation (Prehn’s sign). Orchitis (B) often follows mumps infection and presents with diffuse testicular swelling and tenderness rather than a “high-riding” testis. Torsion of the testicular appendage (D) can cause pain but is less severe, and a “blue dot sign” may be seen on the scrotal skin. The acute, severe presentation with a horizontal lie fits best with testicular torsion.

50. A 28-year-old male with a neck swelling reports (C) Epigastric hernia
difficulty swallowing and hoarseness of voice. (D) Spigelian hernia
Imaging reveals a mass involving the thyroid gland (B) Gastroschisis
This congenital defect results from improper closure of the abdominal wall, usually just to the right of the umbilicus, allowing herniation of abdominal contents directly into the amniotic cavity without a covering sac. The exposed bowel can become edematous and inflamed. Omphalocele (A) is a midline defect at the base of the umbilical cord with a peritoneal covering sac. Epigastric hernia (C) appears through the linea alba above the umbilicus, containing preperitoneal fat or omentum, not typically seen as a neonatal defect with extruded bowel. Spigelian hernia (D) occurs along the semilunar line (lateral border of the rectus sheath), typically in adults or older children, not as a classic congenital protrusion near the umbilical region. Therefore, (B) Gastroschisis best describes the lateral ventral abdominal wall defect often noted at birth with exposed bowel.

extending retrosternally. Likely diagnosis?


(A) Subacute thyroiditis 56. Arrange the following steps in laparoscopic
(B) Multinodular goiter with retrosternal cholecystectomy in the correct sequence:
extension 1. Identify Calot’s triangle structures
(C) Thyroid adenoma 2. Clip and divide the cystic duct and artery
(D) Thyroid lymphoma 3. Insert laparoscopic ports and establish
pneumoperitoneum
4. Dissect gallbladder from the liver bed
(B) Multinodular goiter with retrosternal extension
When a thyroid swelling grows large enough to extend into the thoracic inlet, it can cause compressive symptoms such as dysphagia and hoarseness due to recurrent laryngeal nerve involvement. Multinodular goiter commonly presents as an enlarged thyroid with multiple nodules, and retrosternal extension is relatively frequent in large or long-standing goiters. Subacute thyroiditis (A) typically presents with a painful, tender thyroid gland and systemic inflammatory symptoms. A solitary thyroid adenoma (C) might cause a discrete nodule but not usually a significant retrosternal extension. Thyroid lymphoma (D) can present as a rapidly enlarging thyroid mass, often in older individuals, and may cause compressive symptoms, but it is less common and typically associated with a history of Hashimoto’s thyroiditis. Therefore, a multinodular goiter with retrosternal extension best fits this clinical scenario.

51. All of the following are common complications of Options :


cirrhosis leading to portal hypertension EXCEPT: (A) 3-1-2-4
(A) Esophageal varices (B) 3-2-1-4
(B) Splenomegaly (C) 1-3-4-2
(C) Hepatorenal syndrome (D) 2-1-4-3
(D) Hiatal hernia (A) 3-1-2-4
In a standard laparoscopic cholecystectomy, the first step (3) is to establish pneumoperitoneum (often via a Veress needle or open Hassan technique) and place laparoscopic ports. Next (1), the surgeon identifies the structures of Calot’s triangle (cystic duct, cystic artery, and common hepatic duct) to ensure safe dissection. After clear visualization, (2) the cystic duct and artery are typically clipped and divided. Finally (4), the gallbladder is dissected from the liver bed using electrocautery or ultrasonic energy devices and removed through a port. Proper identification and sequential handling of cystic duct and artery reduces the risk of bile duct injury. Any deviation from this sequence can lead to misidentification and complications. Hence, 3-1-2-4 is the correct order, reflecting the standard approach to laparoscopic cholecystectomy.

57. Consider the following statments regarding acute


(D) Hiatal hernia
Portal hypertension arises in cirrhosis when increased resistance to portal blood flow develops in the liver. Common complications include the formation of portosystemic collaterals such as esophageal varices (A), which can bleed profusely. Splenomegaly (B) results from congestion in the splenic vein, while hepatorenal syndrome (C) reflects severe circulatory changes and renal vasoconstriction secondary to advanced liver disease. In contrast, a hiatal hernia (D) is a diaphragmatic defect allowing the stomach to herniate into the thorax, typically unrelated to portal hypertension. Although patients with cirrhosis can have other abdominal issues, hiatal hernias are not considered a direct or common portal hypertension complication. Therefore, option (D) is the exception. Recognizing typical portal hypertension complications is crucial for managing cirrhotic patients and preventing life-threatening events, such as variceal bleeding and end-organ failure.

52. Which of the following is the most common benign pancreatitis:


tumor of the salivary glands? 1. Serum lipase is more specific than amylase
(A) Warthin’s tumor 2. Grey-Turner’s sign is bruising in the flank
(B) Pleomorphic adenoma region
(C) Oncocytoma 3. Most cases are related to gallstones or alcohol
(D) Basal cell adenoma 4. ERCP is the first-line treatment for all acute
(B) Pleomorphic adenoma
This tumor, also known as a benign mixed tumor, is the most frequent benign neoplasm of the salivary glands, particularly the parotid gland. Histologically, it exhibits a mixture of epithelial and myoepithelial components embedded in a stromal matrix, accounting for its “pleomorphic” character. Clinically, it presents as a painless, slow-growing parotid mass with a smooth surface. Despite being benign, pleomorphic adenomas can transform into a malignant pleomorphic adenoma (carcinoma ex pleomorphic adenoma) if left untreated. Warthin’s tumor (A) is the second most common benign salivary gland tumor, often arising in the parotid gland of older, often smoking patients. Oncocytoma (C) and basal cell adenoma (D) occur less frequently. Therefore, (B) Pleomorphic adenoma remains the most common benign salivary gland tumor overall and is a key entity in head and neck surgery.
pancreatitis
Which of the statements given above are correct?
53. A 65-year-old male smoker has a painless, hard (A) 1, 2, and 3 are correct
cervical lymph node in the posterior triangle. (B) 1 and 4 are correct; 2 and 3 are incorrect
Further evaluation reveals a primary squamous (C) 2 and 3 are correct; 1 and 4 are incorrect
cell carcinoma in the upper aero-digestive tract. (D) 1, 2, 3 are correct; 4 is incorrect
This phenomenon is known as:
MEDINK GRAND TEST 02

63. A 40-year-old with severe right flank pain


(D) 1, 2, 3 are correct; 4 is incorrect
In acute pancreatitis, serum lipase (1) indeed has higher specificity than amylase for diagnosing pancreatic inflammation. Grey-Turner’s sign (2) refers to ecchymosis in the flank area, while Cullen’s sign is periumbilical bruising; both suggest retroperitoneal bleeding in severe pancreatitis. Gallstones and alcohol (3) account for the majority of acute pancreatitis cases. However, ERCP (4) is not the first-line treatment for all cases. It is primarily indicated for acute pancreatitis caused by choledocholithiasis or accompanying cholangitis when there is confirmed or suspected common bile duct obstruction. Many patients with mild or moderate pancreatitis recover with supportive measures (IV fluids, pain control, etc.) without requiring ERCP. Therefore, statements 1, 2, and 3 are correct, while statement 4 is incorrect, matching option (D).

58. In familial medullary thyroid carcinoma (MTC) radiating to the groin presents with hematuria and
due to RET proto-oncogene mutation, the chance colicky discomfort. Non-contrast CT shows a 6
of passing this mutation to offspring is: mm ureteric stone at the ureteropelvic junction.
(A) 25 Best next step?
(B) 50 (A) Trial of medical expulsive therapy
(C) 75 (B) Extracorporeal shock wave lithotripsy
(D) 100 (ESWL)
(C) Ureteroscopic stone retrieval
(D) Open surgical removal
(B) 50
Familial medullary thyroid carcinoma (MTC) arises from autosomal dominant mutations in the RET proto-oncogene. Autosomal dominant inheritance implies that each child of an affected parent has a 50 chance of inheriting the mutant allele and potentially developing MTC (and possibly other endocrine abnormalities as seen in Multiple Endocrine Neoplasia type 2). Option (A) 25 would be more characteristic of an autosomal recessive condition when two carrier parents each have a 50 chance of passing on the defective gene. Option (C) 75 and (D) 100 do not match the standard Mendelian pattern for a single mutant allele in an autosomal dominant trait. Thus, each offspring of an individual with the RET mutation stands a one-in-two risk, making (B) 50 correct.

59. Which is an INCORRECT statement regarding (A) Trial of medical expulsive therapy
A 6 mm ureteric stone can often pass spontaneously, especially if located near the ureteropelvic junction. Medical expulsive therapy using alpha-blockers (e.g., tamsulosin) can relax ureteric smooth muscle and increase the likelihood of stone passage. Adequate hydration and pain control are also essential. ESWL (B) is indicated for stones that fail to pass or are larger (>10 mm), particularly if located in the kidney or upper ureter. Ureteroscopic stone retrieval (C) is preferred for stones that do not pass with conservative measures or are in locations less responsive to ESWL (e.g., lower ureter). Open surgical removal (D) has largely been replaced by minimally invasive methods. Because the question states a 6 mm stone, a trial of medical expulsive therapy is an appropriate initial approach, making (A) the best next step.

colorectal cancer (CRC) screening?


(A) Colonoscopy is considered the gold standard 64. A 55-year-old presents with progressive dysphagia
screening tool and weight loss. Barium swallow shows a “rat-
(B) Fecal occult blood test (FOBT) or FIT can be tail” narrowing at the distal esophagus. Endoscopy
used for initial screening reveals a tight LES. Which diagnostic test confirms
(C) Screening starts at age 50 in average-risk the motility disorder?
individuals (recent guidelines suggest 45) (A) Esophageal pH monitoring
(D) Sigmoidoscopy alone is sufficient to evaluate (B) Esophageal manometry
the entire colon (C) CT scan of the chest
(D) Sigmoidoscopy alone is sufficient to evaluate the entire colon
Sigmoidoscopy visualizes only the distal portion of the large intestine (the rectum and sigmoid colon), missing potential lesions in the transverse or ascending colon. A full colonoscopy is considered the gold standard (A) because it allows direct visualization of the entire colon, detection of polyps, and the ability to biopsy or remove them. Fecal occult blood testing (FOBT) or fecal immunochemical test (FIT) (B) are acceptable noninvasive tools that can be repeated annually or biennially, although any positive test warrants further colonoscopic evaluation. Screening guidelines typically recommend starting at age 50 for average-risk individuals, though many now suggest age 45 (C). Therefore, stating that sigmoidoscopy alone suffices for comprehensive colonic evaluation is incorrect, making (D) the best choice for the inaccurate statement.
(D) Bernstein acid perfusion test
(B) Esophageal manometry
In a patient with progressive dysphagia, weight loss, and barium swallow findings suggestive of a “rat-tail” narrowing, achalasia cardia is a prime consideration. This condition results from failure of the lower esophageal sphincter (LES) to relax and a loss of normal peristalsis in the esophagus. Esophageal manometry is the gold standard test for diagnosing achalasia, demonstrating high resting LES pressure and aperistalsis of the esophageal body. Esophageal pH monitoring (A) helps evaluate gastroesophageal reflux disease, while a CT scan of the chest (C) could exclude malignancy or other structural abnormalities but does not confirm motility disorders. The Bernstein acid perfusion test (D) is an older test used to reproduce heartburn symptoms in reflux disease, not relevant for achalasia diagnosis. Therefore, option (B) is correct.

60. A 25-year-old man with a stab injury in the left


chest develops hypotension, distended neck veins, 65. A 60-year-old male, long-term smoker with
and muffled heart sounds. Diagnosis? chronic obstructive pulmonary disease (COPD),
(A) Tension pneumothorax presents for elective laparotomy. During induction
(B) Cardiac tamponade of anesthesia, he suddenly develops respiratory
(C) Massive hemothorax distress, hypotension, and distended neck veins.
(D) Traumatic aortic rupture Suspecting acute tension pneumothorax, what
immediate intervention should the anesthesiologist
perform?
(B) Cardiac tamponade
The triad of hypotension, jugular venous distension (distended neck veins), and muffled heart sounds is known as Beck’s triad, which strongly suggests tamponade physiology. In penetrating chest trauma, a stab wound near the heart can cause hemorrhage into the pericardial sac, compressing the heart and impairing filling. Tension pneumothorax (A) can also present with hypotension and elevated jugular venous pressure, but breath sounds are diminished on the affected side, and tracheal deviation may occur. Massive hemothorax (C) would show hypotension with absent breath sounds and dullness on percussion over the hemithorax. Traumatic aortic rupture (D) can cause sudden cardiovascular collapse or different findings on imaging, typically a widened mediastinum on chest X-ray. Therefore, the distinct triad points to cardiac tamponade as the correct diagnosis.

61. A middle-aged male with hematemesis is found (A) Endotracheal intubation


to have a pulsatile mass in the epigastrium on (B) Immediate needle decompression
endoscopy, suggesting aortoenteric fistula. Usual (C) Increase positive end-expiratory pressure
location? (PEEP)
(A) Duodenum (D) Administer 100 oxygen and wait
(B) Stomach (B) Immediate needle decompression
In a patient with acute respiratory distress, hypotension, and distended neck veins — particularly in the context of positive-pressure ventilation for anesthesia — tension pneumothorax is a prime suspect. Positive-pressure ventilation can exacerbate air trapping in the pleural space, rapidly increasing intrathoracic pressure. This impedes venous return to the heart, causing shock. Needle decompression in the second intercostal space at the midclavicular line (or the fifth intercostal space at the anterior axillary line, per current guidelines) relieves the pressure instantly. While endotracheal intubation (A) may already be in place during anesthesia induction, it will not fix a tension pneumothorax. Increasing PEEP (C) would worsen the tension physiology. Administering 100 oxygen (D) alone is insufficient to manage the life-threatening hemodynamic compromise. Hence, immediate decompression is essential to reverse the tension.

(C) Esophagus
(D) Jejunum 66. A 50-year-old female with rheumatoid arthritis
presents with progressive cervical myelopathy.
Imaging reveals atlantoaxial subluxation causing
(A) Duodenum
An aortoenteric fistula typically occurs between the abdominal aorta and the third or fourth part of the duodenum due to the aorta’s anatomical proximity. Primary aortoenteric fistula can be caused by an atherosclerotic aneurysm eroding into the bowel, while secondary fistulas arise from vascular graft complications. Patients often present with a “herald bleed,” sometimes followed by massive hemorrhage if untreated. Although fistulas involving the stomach (B), esophagus (C), or jejunum (D) can occur in rare cases (especially with penetrating trauma or advanced malignancy), the duodenum, particularly its third or fourth part, is the most common location because of direct contact with the aorta. Recognition of aortoenteric fistula is critical, given its high mortality if diagnosis and surgical intervention are delayed. Therefore, duodenum is the correct and most typical site.

62. A 65-year-old with prostate cancer on hormonal cord compression at C1–C2. She complains of
therapy has sudden onset of paraplegia. MRI shows neck pain, difficulty walking, and tingling in her
metastatic compression of the thoracic spinal cord. arms. Neurological exam shows upper motor
Initial management step? neuron signs in all limbs. What surgical procedure
(A) High-dose corticosteroids is typically performed to stabilize this region and
(B) Radiotherapy to the spine decompress the spinal cord?
(C) Surgical decompression (A) Anterior cervical discectomy and fusion
(D) Chemotherapy escalation (B) Posterior atlantoaxial fusion
(A) High-dose corticosteroids
In acute spinal cord compression due to metastatic cancer, the immediate step is to reduce edema and inflammation around the cord. High-dose corticosteroids, such as intravenous dexamethasone, can rapidly alleviate compression symptoms and stabilize neurological function. Definitive treatment, typically radiotherapy (B) and/or surgical decompression (C), is arranged urgently based on imaging findings, tumor type, and overall patient status. However, starting steroids promptly helps prevent irreversible neurological damage while other interventions are coordinated. Chemotherapy escalation (D) is not the immediate measure for acute spinal cord compression. Therefore, administering high-dose corticosteroids is recognized as the crucial first-line action in metastatic spinal cord compression, improving the chance of preserving or recovering neurological function before definitive therapy commences.
(C) Laminectomy of C3–C7
(D) Occipitocervical fusion
page 24 NEET PG Surgery

(A) Excess active Vitamin D synthesis


(B) Posterior atlantoaxial fusion
Patients with rheumatoid arthritis often experience instability at the atlantoaxial (C1–C2) joint due to chronic inflammatory changes. This can lead to subluxation and spinal cord compression, manifesting as myelopathy. Posterior fusion techniques (e.g., C1–C2 transarticular screws or other fixation methods) provide stability, preventing further subluxation while decompressing the spinal cord. Anterior cervical discectomy and fusion (A) targets pathology mainly at the subaxial cervical spine (C3–C7). Laminectomy of C3–C7 (C) could decompress the spinal cord in lower cervical pathologies but does not address atlantoaxial instability. Occipitocervical fusion (D) stabilizes the joint from the occiput to the upper cervical spine and may be considered in more extensive disease or when C1–C2 alone is inadequate; however, atlantoaxial fusion is usually sufficient for isolated C1–C2 subluxation. Hence, posterior atlantoaxial fusion remains the standard procedure.

67. A 70-year-old male with a history of type 2 (B) Hypophosphatemia stimulating parathyroid
diabetes and peripheral vascular disease presents hormone secretion
with an infected ulcer on the plantar aspect of his (C) Phosphate retention and reduced calcitriol
foot. He has had multiple debridements with poor production
wound healing. Examination reveals diminished (D) Direct parathyroid gland infiltration by
pedal pulses, cool extremities, and dependent amyloid
rubor. Arterial Doppler shows severe stenosis of (C) Phosphate retention and reduced calcitriol production
In chronic renal failure, the kidneys cannot excrete phosphate efficiently, leading to hyperphosphatemia. High phosphate levels and the resultant decrease in serum calcium stimulate the parathyroid glands. Moreover, diseased kidneys cannot adequately convert 25-hydroxyvitamin D to its active form (1,25-dihydroxyvitamin D, or calcitriol), reducing calcium absorption from the gut. Persistent low calcium levels further drive parathyroid hormone (PTH) secretion, culminating in secondary hyperparathyroidism. Over time, parathyroid hyperplasia may progress to a state of autonomous PTH secretion, known as tertiary hyperparathyroidism. In contrast, excess active Vitamin D (A) would suppress PTH. Hypophosphatemia (B) would not occur in renal failure; instead, phosphate retention is common. Direct amyloid infiltration (D) is not a usual mechanism for persistent hyperparathyroidism. Thus, (C) remains the key pathophysiological cause in these patients.

the anterior tibial artery. He is at risk of major


amputation if revascularization is not achieved. 71. Which term refers to the protrusion of the meninges
Which intervention is most suitable? and spinal cord through a vertebral defect?
(A) Endovascular angioplasty and stenting (A) Meningocele
(B) Below-knee amputation directly (B) Myelomeningocele
(C) Compression therapy (C) Encephalocele
(D) Antibiotics alone (D) Meningomyelocele of the brainstem
(B) Myelomeningocele
In this neural tube defect, both the spinal cord (myelo-) and the meninges (-meningo-) protrude through a vertebral defect, forming a sac-like cyst on the back. This defect often involves nerve tissue and can lead to significant neurological deficits such as paralysis and sensory loss below the level of the lesion. Meningocele (A) involves only the meninges protruding without neural elements in the sac, thus typically less severe neurologically. Encephalocele (C) refers to a cranial defect where brain and meninges protrude, usually occurring in the occipital region. “Meningomyelocele of the brainstem” (D) is not a recognized standard term; the correct usage is “myelomeningocele” to describe spinal cord involvement. Therefore, (B) myelomeningocele is the appropriate term describing the protrusion of both meninges and spinal cord through a vertebral defect.

(A) Endovascular angioplasty and stenting


In diabetic patients with peripheral vascular disease, non-healing foot ulcers commonly result from inadequate arterial perfusion and infection. Given the Doppler findings of significant anterior tibial artery stenosis, restoring blood flow is crucial to salvage the limb. Endovascular angioplasty with or without stenting offers a minimally invasive approach to revascularization, potentially improving wound healing and avoiding major amputation. Below-knee amputation (B) may become necessary if revascularization efforts fail or infection is uncontrollable, but it is not the first step unless the limb is nonviable. Compression therapy (C) primarily benefits venous insufficiency and is not a standard measure for arterial disease. Antibiotics alone (D) address infection but will not correct ischemia. Therefore, angioplasty/stenting is the most suitable intervention to improve perfusion and facilitate ulcer healing, often preventing major limb loss.

68. Assertion (A) : Hyperbaric oxygen therapy is used 72. In thyroid surgery, injury to the external branch of
in the treatment of clostridial myonecrosis (gas the superior laryngeal nerve can lead to:
gangrene). (A) Loss of sensation above the vocal cords
Reason (R) : Hyperbaric oxygen inhibits the (B) Inability to abduct the vocal folds
proliferation of anaerobic bacteria and improves (C) Difficulty with high-pitched singing
leukocyte function in devitalized tissues. (D) Complete aphonia
(A) Both A and R are true, and R is the correct (C) Difficulty with high-pitched singing
The external branch of the superior laryngeal nerve innervates the cricothyroid muscle, which tenses the vocal cords. Injury to this nerve impairs the ability to produce high-frequency sounds and modulate pitch. Sensory innervation above the vocal cords (A) is primarily provided by the internal branch of the superior laryngeal nerve, not the external branch. Inability to abduct the vocal folds (B) results from injury to the recurrent laryngeal nerve, specifically affecting the posterior cricoarytenoid muscle. Complete aphonia (D) typically requires bilateral vocal cord impairment or severe damage to both recurrent laryngeal nerves. Thus, an isolated lesion of the external branch of the superior laryngeal nerve presents with difficulty in singing high notes or projecting the voice at higher pitches. Surgeons must be cautious with nerve preservation during thyroidectomy, especially near the superior thyroid artery branches.

explanation of A
73. A 50-year-old female has gallstones. Which
(B) Both A and R are true, but R is not the correct
factor primarily contributes to cholesterol stone
explanation of A
formation?
(C) A is true, but R is false
(A) Low serum cholesterol levels
(D) A is false, but R is true
(B) Decreased hepatic secretion of bile acids
(A) Both A and R are true, and R is the correct explanation of A
Hyperbaric oxygen therapy can play a supportive role in clostridial myonecrosis by increasing the partial pressure of oxygen in tissues, thereby suppressing the growth of anaerobic bacteria such as Clostridium perfringens. Furthermore, high oxygen levels enhance neutrophil oxidative killing mechanisms. While the mainstay of gas gangrene treatment remains prompt surgical debridement and antibiotic therapy, hyperbaric oxygen can be a useful adjunct in severe infections. The reason that it inhibits proliferation of anaerobes and augments leukocyte function provides the rationale for its use in these necrotizing infections. Hence, both the assertion and the reason are correct, with the reason directly explaining why hyperbaric oxygen therapy is relevant in treating clostridial myonecrosis.

(C) Increased phospholipid content in bile


69. Match the incision type (Column A) with its typical (D) Low estrogen states
surgical use (Column B): (B) Decreased hepatic secretion of bile acids
Cholesterol gallstone formation is multifactorial but hinges on biliary cholesterol supersaturation, which occurs when there is insufficient bile salt (bile acid) and phospholipid (mainly lecithin) relative to cholesterol. If hepatic secretion of bile acids is reduced (or if cholesterol secretion is inappropriately high), the bile becomes supersaturated with cholesterol, predisposing to crystal and stone formation. Low serum cholesterol (A) would not increase gallstone risk. Increased phospholipid (C) in bile generally helps solubilize cholesterol, reducing stone formation. High estrogen states (not low) can increase hepatic secretion of cholesterol and reduce bile salt secretion, raising the risk of gallstones. Thus, among the listed factors, decreased bile acid excretion is the primary contributor, making (B) correct.

Column I Column II 74. When comparing Hodgkin’s lymphoma (HL) with


1. Kocher’s incision a. Appendectomy Non-Hodgkin’s lymphoma (NHL), which is more
2. Pfannenstiel b. Open commonly associated with contiguous lymph node
incision cholecystectomy spread?
(A) NHL
3. Lanz incision c. Renal transplant
(B) HL
4. Rutherford d. Lower segment
(C) Both have equally contiguous spread
Morrison incision Caesarean section
(D) Neither typically spreads contiguously
Select the correct answer using the code given
below :
(B) HL
Hodgkin’s lymphoma is known for its orderly and contiguous spread from one lymph node group to the next. This pattern often begins in cervical lymph nodes and advances down the lymphatic chain. In contrast, Non-Hodgkin’s lymphoma (NHL) can spread noncontiguously, frequently involving multiple lymph node regions or extranodal sites early in the disease. HL typically presents with the characteristic Reed-Sternberg cells in lymph nodes and is more regionally localized. NHL, due to its heterogeneity, exhibits more unpredictable dissemination. Therefore, HL is the classic lymphoma for contiguous node-to-node progression, making option (B) correct.

(A) 1-b, 2-d, 3-a, 4-c 75. All of the following are signs of hypercalcemia
(B) 1-a, 2-c, 3-d, 4-b EXCEPT:
(C) 1-d, 2-b, 3-c, 4-a (A) Polyuria
(D) 1-c, 2-a, 3-b, 4-d (B) Abdominal pain
(A) 1-b, 2-d, 3-a, 4-c
Explanation:
1.
2.
3.
Kocher’s incision (b): A subcostal incision typically used for open cholecystectomy, providing good exposure of the gallbladder and biliary tree.
Pfannenstiel incision (d): A transverse, curved suprapubic incision commonly employed for lower segment Caesarean section and various pelvic surgeries.
Lanz incision (a): A modification of the traditional McBurney’s point incision used for appendectomy; it is transverse, situated in the right iliac fossa.
(C) Trousseau’s sign
(D) Constipation
4. Rutherford Morrison incision (c): An oblique retroperitoneal approach in the iliac fossa often utilized for renal transplants, permitting direct vascular anastomoses in the pelvis.
Thus, the matching is Kocher’s incision → open cholecystectomy, Pfannenstiel incision → lower segment Caesarean, Lanz incision → appendectomy, and Rutherford Morrison incision → renal transplant, making option (A) correct.

70. Persistent hyperparathyroidism in a patient with (C) Trousseau’s sign

chronic renal failure is primarily due to:


Hypercalcemia often presents with symptoms remembered by the phrase “bones, stones, abdominal groans, and psychic moans,” indicating bone pain, kidney stones, gastrointestinal distress (like abdominal pain, constipation), and neuropsychiatric changes. Polyuria (A) can occur due to decreased kidney concentrating ability. Abdominal pain (B) and constipation (D) are also common. In contrast, Trousseau’s sign (C) is a clinical sign typically associated with hypocalcemia, not hypercalcemia. It is elicited by inflating a blood pressure cuff on the arm and observing carpal spasm. This spasm, known as Trousseau’s phenomenon, arises from increased neuromuscular irritability due to low calcium levels. Consequently, Trousseau’s sign is the exception in this list of hypercalcemia manifestations.
MEDINK GRAND TEST 02

76. A 55-year-old with atrial fibrillation on (B) Carotid endarterectomy


anticoagulation presents with an acutely ischemic (C) Carotid artery stenting in all cases
foot. The best initial investigation to confirm (D) Echolucent plaque monitoring every 6
arterial occlusion is: months
(A) Duplex ultrasound (B) Carotid endarterectomy
In patients with significant (typically >70) symptomatic carotid artery stenosis, carotid endarterectomy provides a proven survival and stroke prevention benefit compared to medical therapy alone. “Symptomatic” indicates that the patient has had transient ischemic attacks or minor strokes correlating with the ipsilateral carotid lesion. While antiplatelet therapy, statins, and risk factor control are essential (A), they may not suffice for high-grade stenosis in symptomatic cases. Carotid artery stenting (C) is an alternative for some patients who are high-risk surgical candidates or have contraindications to endarterectomy, but endarterectomy remains standard for most. Monitoring plaque (D) applies to lower-grade or asymptomatic stenoses. In a 70-year-old man with 80 symptomatic stenosis, carotid endarterectomy is the recommended definitive intervention to reduce stroke risk, making (B) correct.

(B) CT angiography
(C) Plain radiograph of the foot 81. A 45-year-old with a 5 cm fibroadenoma of the
(D) MR venography breast persists despite conservative follow-up.
Which surgical treatment is most definitive?
(A) Core needle biopsy only
(A) Duplex ultrasound
In a patient with a sudden onset cold, pulseless, painful foot — likely from an arterial embolus — rapid bedside assessment with duplex ultrasound can quickly detect absent or diminished arterial flow. Duplex ultrasound combines B-mode imaging with Doppler flow analysis, allowing for real-time evaluation of arterial patency and identifying the approximate location of the occlusion. CT angiography (B) provides detailed vascular anatomy but requires contrast administration and the time necessary to perform a CT scan, which might delay urgent intervention. A plain radiograph (C) cannot directly show arterial occlusion. MR venography (D) is used for venous structures rather than acute arterial occlusion. Since time is critical to salvage ischemic tissue, a fast, noninvasive test like duplex ultrasound is the best initial diagnostic step to confirm acute arterial occlusion.

77. A 45-year-old male complains of a painless (B) Simple excision of the lump
swelling in the submandibular region for 3 months. (C) Wide local excision with margins
It is firm, mobile, and not tender. Ultrasound shows (D) Mastectomy
a well-defined solid mass in the submandibular (B) Simple excision of the lump
Fibroadenomas are benign breast tumors commonly found in younger women. They often regress spontaneously or remain stable in size, and many cases do not require intervention. However, in a persistent or large fibroadenoma (especially exceeding 3–4 cm and causing cosmetic concerns or anxiety), surgical removal can be offered. Simple excision (enucleation) of the lump is typically sufficient for definitive management. Core needle biopsy (A) is primarily diagnostic, not therapeutic. Wide local excision with margins (C) is more common for malignant lesions, and a mastectomy (D) would be far too aggressive for a benign fibroadenoma. Therefore, the most definitive step in managing a persistent 5 cm fibroadenoma is a straightforward surgical excision of the lump, making (B) correct.

gland. Most likely diagnosis?


(A) Pleomorphic adenoma 82. A 55-year-old smoker complains of hematuria.
(B) Warthin’s tumor Cystoscopy reveals a papillary lesion in the
(C) Mucoepidermoid carcinoma bladder. Which is most likely?
(D) Sialadenitis (A) Renal cell carcinoma
(B) Transitional cell carcinoma
(C) Squamous cell carcinoma
(A) Pleomorphic adenoma
While pleomorphic adenomas are most commonly found in the parotid gland, they can also arise in other salivary glands, including the submandibular gland. They typically present as slow-growing, painless, firm masses that are well-demarcated. Warthin’s tumor (B), also known as papillary cystadenoma lymphomatosum, is more often in the parotid and associated with smoking. Mucoepidermoid carcinoma (C) tends to have variable presentations, sometimes painful or with facial nerve involvement, if in the parotid. Sialadenitis (D) usually presents with pain, swelling, and signs of infection or inflammation, not a painless, well-defined mass. Given the gradual, painless nature and firm, mobile characteristics, plus ultrasound findings of a solid lesion, pleomorphic adenoma is the most likely diagnosis in this submandibular region scenario.

78. Arrange the muscle layers encountered in a right (D) Adenocarcinoma


paramedian incision (lateral to the linea alba) from (B) Transitional cell carcinoma
Hematuria in an older smoker with a papillary bladder lesion on cystoscopy is highly suggestive of TCC, the most common form of bladder cancer. It frequently presents with painless hematuria and is strongly linked to risk factors like smoking and occupational exposures (e.g., aromatic amines). Renal cell carcinoma (A) is a primary kidney tumor, not typically presenting as a papillary lesion within the bladder. Squamous cell carcinoma (C) of the bladder is less common in developed nations, often associated with chronic irritation or schistosomiasis in endemic areas. Adenocarcinoma (D) of the bladder is rare and linked to urachal remnants or chronic inflammation. Therefore, TCC is the prime suspect in a papillary bladder lesion, making (B) correct.

superficial to deep:
1. Anterior rectus sheath 83. A 40-year-old male presents with severe right
2. Rectus abdominis muscle lower quadrant pain, fever, and guarding. CT
3. Posterior rectus sheath scan suggests an inflamed Meckel’s diverticulum.
4. Peritoneum Which surgery is recommended?
Select the option for correct order : (A) Simple diverticulectomy
(A) 1-2-3-4 (B) Right hemicolectomy
(B) 2-1-3-4 (C) Ileocecal resection
(C) 1-3-2-4 (D) Diverticulectomy with wedge resection of
(D) 2-3-1-4 mesentery
(A) Simple diverticulectomy
Meckel’s diverticulum is a true diverticulum of the ileum, typically located about 2 feet proximal to the ileocecal valve. It can become inflamed, mimicking appendicitis and causing right lower quadrant pain and fever. In an inflamed or symptomatic Meckel’s diverticulum, surgical removal is indicated. Simple diverticulectomy (i.e., excision of the diverticulum) is often sufficient if the bowel base is healthy and free of ulceration or ectopic tissue. However, a small bowel resection (with reanastomosis) may be required if there is involvement of the ileal wall or suspicion of ectopic gastric mucosa extending into the bowel. A right hemicolectomy (B) or ileocecal resection (C) is not generally indicated unless there is an extensive disease process or overlap with the cecum. Diverticulectomy with wedge resection of mesentery (D) is uncommon unless the mesentery itself is diseased. Thus, simple diverticulectomy is recommended for an inflamed Meckel’s.

(A) 1-2-3-4
A paramedian incision, placed lateral to the linea alba but medial to the lateral border of the rectus abdominis, involves incising the anterior rectus sheath (1) first. Then, the rectus abdominis muscle (2) is split or retracted. Beneath that is the posterior rectus sheath (3) in the upper and mid abdomen (above the arcuate line). Finally, the peritoneum (4) is opened to access the abdominal cavity. Below the arcuate line (roughly midway between the umbilicus and pubic symphysis), the posterior rectus sheath is absent, but in a typical paramedian approach to the upper or mid abdomen, these four layers remain consistent. Therefore, the sequence from superficial to deep is 1 → 2 → 3 → 4, making option (A) correct.

79. A 35-year-old female with a history of multiple 84. A 30-year-old female with a painful lump in her
renal stones is found to have elevated serum left subareolar region has sticky nipple discharge.
calcium, elevated parathyroid hormone (PTH), and Examination suggests periductal mastitis.
a single enlarged parathyroid gland on imaging. Preferred initial management?
What key feature confirms a parathyroid adenoma (A) Antibiotics and reassurance
rather than hyperplasia? (B) Subareolar duct excision
(A) Elevation of all parathyroid glands (C) Fine needle aspiration
(B) A single hyperfunctioning gland (D) Core needle biopsy
(C) Low urinary calcium excretion (A) Antibiotics and reassurance
Periductal mastitis is an inflammatory condition often occurring beneath the nipple-areolar complex. Patients may present with subareolar pain, swelling, and occasionally discharge. Smoking can be a risk factor due to ductal irritation. Initial management involves ruling out abscess formation and starting antibiotic therapy targeting common skin flora, such as staphylococci. Most cases respond to conservative measures (antibiotics, warm compresses, and supportive care). Surgical excision of the involved ducts (B) may be considered for recurrent or persistent disease. Fine needle aspiration (C) or core needle biopsy (D) is indicated if there is suspicion of malignancy, atypical features, or a discrete mass requiring pathological evaluation. However, first-line management in a straightforward presentation of periductal mastitis is antibiotics and observation, making (A) the best initial approach.

(D) Normal PTH but high serum calcium


85. A 55-year-old alcoholic presents with upper GI
bleed and endoscopy reveals bleeding esophageal
(B) A single hyperfunctioning gland
Primary hyperparathyroidism can be caused by a parathyroid adenoma, hyperplasia, or carcinoma. In the scenario of an adenoma, only one gland (occasionally two) is typically enlarged and hyperfunctioning, secreting excess PTH, resulting in hypercalcemia. Hyperplasia (A) involves multiple glands being enlarged, while carcinoma is less common but often presents with significantly elevated calcium levels. Low urinary calcium excretion (C) would be unusual in classic primary hyperparathyroidism; typically, urinary calcium is elevated. Normal PTH with high serum calcium (D) would suggest a different etiology, perhaps malignancy-mediated hypercalcemia without elevated PTH, or familial hypocalciuric hypercalcemia (FHH) where PTH is inappropriately normal but urinary calcium excretion is low. Thus, finding a single enlarged and overactive gland correlates with a parathyroid adenoma, matching (B) as the key feature.

80. A 70-year-old man with symptomatic carotid varices. Despite endoscopic banding, bleeding
stenosis of 80 undergoes imaging. What is the persists. What is the next interventional step?
recommended definitive treatment to prevent (A) Sengstaken-Blakemore tube placement
stroke? (B) Beta-blocker therapy
(A) Medical management with antiplatelets only (C) Sclerotherapy again immediately
page 26 NEET PG Surgery

(D) Transjugular intrahepatic portosystemic indicated to re-establish blood flow and relieve
shunt (TIPS) symptoms?
(D) Transjugular intrahepatic portosystemic shunt (TIPS)
In the acute management of bleeding esophageal varices, first-line therapy includes endoscopic band ligation and vasoactive drugs such as octreotide. If endoscopic measures fail or bleeding persists, TIPS provides a salvage procedure to decompress the portal system by creating a channel between a hepatic vein and portal vein. While Sengstaken-Blakemore tube (A) can temporarily tamponade bleeding varices, it is a short-term measure and carries risk of complications like aspiration. Beta-blocker therapy (B) aids in preventing rebleeding long-term but is not the immediate solution for active refractory bleeding. Repeated sclerotherapy (C) might be an option but has lower efficacy than TIPS in ongoing uncontrolled bleeding. Therefore, TIPS is the next interventional step to achieve definitive control of persistent variceal hemorrhage after endoscopic failure.
(A) Aorto-bifemoral bypass
(B) Femorofemoral crossover bypass
86. A 70-year-old ex-smoker presents with chronic (C) Femoro-popliteal bypass
cough, hemoptysis, and weight loss. Imaging shows (D) Axillo-bifemoral bypass
a central lung mass with hilar lymphadenopathy,
and bronchoscopy detects malignant cells. Which
(C) Femoro-popliteal bypass
This patient has severe peripheral arterial disease involving the superficial femoral artery, manifesting as claudication and progressing to rest pain (ABI of 0.4 indicates critical or near-critical ischemia). When endovascular attempts fail, a surgical bypass is the next step. A femoro-popliteal bypass uses a vein or prosthetic graft from the femoral artery to the popliteal artery distal to the occlusion, effectively bypassing the diseased segment. Aorto-bifemoral bypass (A) addresses aortoiliac disease, typically not used for isolated femoral artery occlusions. Femorofemoral crossover bypass (B) is performed when one iliac artery is patent, and the other is occluded, providing flow from the healthy side. Axillo-bifemoral bypass (D) is a more proximal extra-anatomic approach, usually reserved for high-risk patients who cannot tolerate an aorto-bifemoral procedure. Therefore, (C) Femoro-popliteal bypass is indicated for superficial femoral artery occlusion with lifestyle-limiting ischemia.

histological subtype is most likely? 90. Assertion (A) : A sliding inguinal hernia typically
(A) Adenocarcinoma contains colon or bladder as part of its wall.
(B) Squamous cell carcinoma Reason (R) : The posterior wall of the inguinal
(C) Large cell carcinoma canal is formed by the transversalis fascia in all
(D) Small cell lung cancer its extent.
(B) Squamous cell carcinoma
A central lung mass in an older smoker causing cough, hemoptysis, and weight loss is highly suggestive of squamous cell carcinoma. This subtype of non-small cell lung cancer often arises in the central bronchi, can invade hilar structures, and commonly presents with postobstructive pneumonia or bleeding. Adenocarcinoma (A) tends to be more peripheral and is currently the most common subtype overall, especially in non-smokers, but it is not typically central. Large cell carcinoma (C) can occur anywhere but lacks the distinguishing features of squamous or glandular differentiation. Small cell lung cancer (D) often presents centrally but is characterized by very aggressive behavior, strong paraneoplastic associations, and a different therapeutic approach (chemotherapy-based). The classic presentation of a central, cavitating tumor in a heavy smoker with hemoptysis points strongly to squamous cell carcinoma.
(A) Both A and R are true, and R is the correct
explanation of A
87. A 35-year-old male complains of chronic diarrhea, (B) Both A and R are true, but R is not the correct
abdominal pain, and weight loss. Colonoscopy explanation of A
reveals segmental inflammation with skip lesions, (C) A is true, but R is false
transmural involvement, and granulomas on (D) A is false, but R is true
biopsy. There is a stricture in the terminal ileum.
Which surgical procedure is commonly performed
(B) Both A and R are true, but R is not the correct explanation of A
In a sliding inguinal hernia, part of the hernia sac wall is formed by a viscus, commonly the sigmoid colon on the left or the cecum/bladder on the right. This happens because these structures “slide” into the inguinal canal, effectively becoming part of the hernia sac. Meanwhile, the posterior wall of the inguinal canal is predominantly formed by the transversalis fascia, reinforced by the conjoint tendon medially. While this statement (R) is accurate, it does not explain why certain viscera form part of the wall in sliding hernias. The presence of sliding hernia is more related to anatomic attachments of these organs to the lateral pelvic wall, rather than simply the nature of the inguinal canal’s posterior wall. Therefore, both statements are correct, but R does not directly explain A.

for refractory disease in this location? 91. In the management protocol for post-thyroidectomy
(A) Subtotal colectomy hypocalcemia, which step is INCORRECT?
(B) Ileocolic resection (A) Check corrected serum calcium and albumin
(C) Stricturoplasty levels
(D) Proctocolectomy (B) Immediate IV calcium gluconate if
(B) Ileocolic resection
Crohn’s disease commonly affects the terminal ileum and may result in strictures, fistulas, and malabsorption. Surgery is reserved for complications or disease refractory to medical therapy. In cases where the terminal ileum and adjacent colon are involved or severely strictured, ileocolic resection (resecting the diseased segment of the ileum and the nearby colon) is a standard approach. Subtotal colectomy (A) removes the majority of the colon but is not targeted specifically to the terminal ileum. Stricturoplasty (C) can be performed to widen narrowed segments, preserving bowel length, especially in patients with multiple or long strictures, but if the segment is severely diseased, resection may be necessary. Proctocolectomy (D) is more appropriate for extensive colonic disease or severe ulcerative colitis. Thus, ileocolic resection is the typical surgical choice for refractory terminal ileal Crohn’s disease.
symptomatic
(C) Long-term magnesium supplementation in
88. A 28-year-old athlete presents with chronic groin all patients
pain. Clinical examination reveals tenderness (D) Oral calcium and vitamin D supplementation
over the pubic symphysis and pain aggravated if mild
by adductor muscle testing. Imaging shows
sclerosis and bony changes at the pubic symphysis
(C) Long-term magnesium supplementation in all patients
After thyroidectomy, transient or permanent hypoparathyroidism can result in hypocalcemia. Management includes checking corrected serum calcium and albumin (A), since low albumin can produce a falsely low total calcium. If the patient is symptomatic (e.g., perioral numbness, tetany), intravenous calcium gluconate (B) is given promptly. For milder cases, oral calcium and vitamin D (D) are sufficient to restore and maintain normal calcium levels. Magnesium is relevant if the patient’s hypocalcemia is refractory to standard treatment and hypomagnesemia is confirmed, as adequate magnesium is required for PTH secretion. However, automatically placing all post-thyroidectomy patients on long-term magnesium is not routine or warranted unless they demonstrate persistent low magnesium. Therefore, (C) is the incorrect step in a standard hypocalcemia management protocol.

consistent with osteitis pubis. Conservative 92. In a female with BRCA1 mutation, the lifetime
management, including rest and physiotherapy, risk of developing breast cancer approaches:
fails to relieve symptoms. What surgical option (A) 10–15
may be considered to alleviate chronic pain? (B) 20–30
(A) Pubic symphysis fusion (C) 45–60
(B) Adductor tenotomy (D) 55–80
(C) Core muscle release (D) 55–80
Women carrying a BRCA1 mutation face a significantly elevated lifetime risk of breast cancer compared to the general population. While estimates vary depending on family history and other modifiers, a commonly cited range is 55–80. This high likelihood supports enhanced screening, chemoprevention, or prophylactic surgery (e.g., bilateral mastectomy) in some cases. BRCA2 mutations also carry a high risk but slightly lower than BRCA1 in many studies (often cited around 45–70). Lower percentages such as 10–15 or 20–30 are closer to baseline population or slightly elevated risk categories, while 45–60 is a bit lower than typical estimates for BRCA1. Thus, the best recognized approximation for BRCA1-associated breast cancer risk is (D) 55–80.

(D) Herniorrhaphy
93. Chylous ascites is most commonly caused by:
(A) Low albumin states
(B) Adductor tenotomy
Osteitis pubis is an overuse injury leading to inflammation around the pubic symphysis, often exacerbated by repetitive stress on the pelvic stabilizing muscles. Patients, especially athletes, may experience persistent groin pain that worsens with adductor testing. While most improve with rest, NSAIDs, and targeted physiotherapy, some remain symptomatic. In refractory cases, surgical intervention targets relieving tension from the involved muscles. An adductor tenotomy (specifically partial adductor longus release) can reduce tension across the pubic symphysis, alleviating chronic pain. Pubic symphysis fusion (A) is rarely performed and can limit pelvic motion significantly. Core muscle release (C) or herniorrhaphy (D) might be indicated in sports hernias or posterior inguinal wall deficiencies, but these do not directly address chronic osteitis pubis if the primary driver is adductor tension. Therefore, adductor tenotomy is the favored procedure in such refractory cases.

89. A 50-year-old male smoker presents with (B) Cirrhosis


claudication of the left calf, progressing to rest (C) Lymphatic obstruction or disruption
pain. On examination, there is hair loss on the foot, (D) Pseudomembranous colitis
pale skin, and weak dorsalis pedis pulse. Ankle- (C) Lymphatic obstruction or disruption
Chylous ascites results from the accumulation of lymph fluid in the peritoneal cavity, which appears milky due to high triglyceride content. The most common etiologies include malignancies (e.g., lymphoma) that obstruct lymphatic channels, surgical or traumatic disruption of the lymphatics, and some congenital anomalies. Low albumin states (A) lead to transudative ascites, while cirrhosis (B) causes portal hypertension and similarly results in transudative ascites. Pseudomembranous colitis (D) is an infectious/inflammatory colonic disease caused by Clostridioides difficile, not a recognized cause of chylous ascites. Therefore, the underlying cause for chylous ascites typically involves either blockage or damage to the lymphatic system, making option (C) correct.

brachial index is 0.4. Arterial Doppler confirms


superficial femoral artery occlusion. After 94. A study reports a hazard ratio of 0.5 (p < 0.05)
unsuccessful angioplasty, the patient has lifestyle- for death using a new chemotherapy regimen vs.
limiting ischemia. Which surgical procedure is standard therapy in advanced colon cancer. This
suggests:
MEDINK GRAND TEST 02

(A) The new regimen doubles the death rate (A) High-risk, so avoid surgery completely
(B) The new regimen halves the risk of death (B) Elective surgical repair with appropriate
(C) The result is not statistically significant perioperative management of ascites
(D) The new regimen is equivalent to the standard (C) Emergency surgery only if strangulation
therapy occurs
(B) The new regimen halves the risk of death
A hazard ratio (HR) compares the hazard (or risk) of an event occurring at any point in time in two groups. An HR of 0.5 indicates that the risk of the outcome (death, in this case) is 50 lower in the group receiving the new chemotherapy regimen compared to the standard therapy group. The p-value < 0.05 implies statistical significance at the conventional threshold, suggesting the finding is unlikely due to chance. Doubling the death rate (A) would correspond to an HR of 2.0, not 0.5. If the result were not statistically significant (C), the p-value would typically exceed 0.05. Equivalence (D) would suggest an HR of around 1.0. Therefore, an HR of 0.5 with p < 0.05 means the new therapy significantly reduces the risk of death by half.
(D) Paracentesis followed by immediate repair

95. Match the antibiotic (Column A) with its primary


(B) Elective surgical repair with appropriate perioperative management of ascites
Umbilical hernias are common in cirrhotic patients with ascites due to increased intra-abdominal pressure. Although cirrhosis elevates surgical risk, an untreated hernia can lead to complications such as strangulation or rupture. Current guidelines favor elective repair once ascites is controlled, using measures like diuretics (e.g., spironolactone, furosemide) or paracentesis if needed, to reduce tension on the repair. Avoiding surgery completely (A) leaves the patient vulnerable to emergencies. Emergency surgery (C) in a strangulated hernia carries higher morbidity. Paracentesis alone (D) without proper timing and optimization is insufficient. By optimizing ascites management and nutritional status, elective surgical repair can proceed more safely. Thus, (B) is the best approach for preventing serious complications while acknowledging the underlying liver disease.

mechanism (Column B): 98. A 35-year-old man has a confirmed perforation


of the tympanic membrane with chronic otorrhea.
Column I Column II
Medical therapy fails to control persistent
1. Vancomycin a. Inhibits cell wall
discharge. What is the most definitive surgical
synthesis by binding
step?
to D-Ala-D-Ala
(A) Myringoplasty
2. Fluoroquinolones b. Blocks DNA gyrase (B) Tympanostomy tube insertion
and topoisomerase
(C) Mastoidectomy with tympanoplasty
3. Aminoglycosides c. Interferes with (D) Antibiotic ear drops indefinitely
bacterial protein
synthesis at 30S
(C) Mastoidectomy with tympanoplasty
Chronic otitis media (COM) with persistent ear discharge and tympanic membrane perforation often implicates disease in the mastoid air cells. A simple myringoplasty (A) might fail if infection remains in the mastoid. Tympanostomy tube insertion (B) is more appropriate for recurrent middle ear effusions (otitis media with effusion), not longstanding purulent discharge. Antibiotic ear drops indefinitely (D) only provide temporary control and do not definitively eradicate the infection source. Mastoidectomy (removal of infected mastoid air cells) combined with tympanoplasty (repair of the tympanic membrane and middle ear structures) addresses the underlying mastoid disease and reconstructs the drum. This combined procedure is the most definitive means to stop the chronic discharge and restore hearing, making (C) correct.

ribosomal subunit 99. A 42-year-old man presents with painless


4. Beta-lactams d. Inhibits hematuria. Imaging detects a renal mass. Which
peptidoglycan finding is most characteristic of renal cell
cross-linking carcinoma?
by binding to (A) Hormone production
penicillin-binding (B) Varicocele formation
proteins (C) Hypotension
Select the correct answer using the code given (D) Low ESR
below: (B) Varicocele formation
Renal cell carcinoma (RCC) can extend into the renal vein and even up the inferior vena cava, potentially obstructing venous outflow. In males, this may present as a left-sided varicocele that fails to decompress when the patient lies down, an important clinical clue. Other paraneoplastic features of RCC include ectopic hormone production (A) such as erythropoietin (leading to polycythemia) or PTHrP (hypercalcemia), but varicocele formation is particularly characteristic of left renal tumors due to the drainage pathway of the left gonadal vein into the left renal vein. Hypotension (C) is not typical; hypertension is more common. Elevated ESR (opposite of D) can occur as a nonspecific inflammatory marker. Therefore, the presence of a non-resolving varicocele in a male with a renal mass strongly suggests RCC, making (B) correct.

(A) 1-a, 2-b, 3-c, 4-d


(B) 1-b, 2-a, 3-d, 4-c 100. Question 100.A 60-year-old with diabetes has
(C) 1-d, 2-c, 3-a, 4-b a diabetic foot ulcer with exposed bone. MRI
(D) 1-c, 2-b, 3-d, 4-a suggests osteomyelitis. What is the best next step?
(A) Wound swab culture
(B) Bone biopsy and culture
(A) 1-a, 2-b, 3-c, 4-d
1. Vancomycin (a): Binds the D-Ala-D-Ala terminus of peptidoglycan precursors, blocking cell wall synthesis.
2. Fluoroquinolones (b): Inhibit bacterial DNA gyrase (topoisomerase II) and topoisomerase IV.
3. Aminoglycosides (c): Bind to the 30S ribosomal subunit, causing misreading of mRNA and disruption of protein synthesis.
4. Beta-lactams (d): Bind to penicillin-binding proteins and inhibit the cross-linking of the peptidoglycan layer, preventing cell wall synthesis.

96. In gastric ulcer disease: (C) Empirical broad-spectrum IV antibiotics


1. Pain often worsens with meals (D) Debride all necrotic tissue without biopsy
2. H. pylori infection is a common cause (B) Bone biopsy and culture
In a diabetic foot ulcer with suspected osteomyelitis (especially when imaging is suggestive), obtaining an accurate microbiologic diagnosis is vital to guide targeted antibiotic therapy. A bone biopsy (preferably percutaneous or surgical) provides a more definitive sample of the infected bone than a superficial wound swab. Wound swab cultures (A) often yield colonizing organisms that do not reflect the true pathogen causing osteomyelitis. Empirical broad-spectrum IV antibiotics (C) can be initiated after samples are taken if the patient is systemically unwell, but the best step for precise management is to secure a bone biopsy culture. Debridement of necrotic tissue (D) is indeed important; however, performing it without biopsy risks losing the opportunity for pathogen identification. Hence, (B) bone biopsy and culture is the most appropriate next step.

3. Most commonly found on the greater


curvature
4. Malignant transformation can occur in some
cases
Which of the statements given above are correct?
(A) 1, 2, and 4 are correct; 3 is incorrect
(B) 1 and 3 are correct; 2 and 4 are incorrect
(C) 2 and 4 are correct; 1 and 3 are incorrect
(D) 1, 2, 3, and 4 are all correct
(A) 1, 2, and 4 are correct; 3 is incorrect
Gastric ulcers typically produce epigastric pain that can worsen with meals (1) because acid secretion aggravates the exposed ulcer surface. Helicobacter pylori (2) is a frequent cause, along with NSAIDs. While peptic ulcers can occur in various gastric locations, a significant portion arises along the lesser curvature, especially in the region of the incisura angularis, rather than the greater curvature (3), making statement 3 incorrect. Additionally, some gastric ulcers can undergo malignant transformation or might be malignant at presentation, hence endoscopic biopsy is essential (4). So statements 1, 2, and 4 are correct, and 3 is incorrect, matching (A).

97. A 60-year-old with a small, symptomatic umbilical


hernia has cirrhosis and mild ascites. What is the
best approach?
page 28 NEET PG Surgery

GRAND TEST 03

1. Which statement best defines a “clean- (D) Similar recurrence rate with significantly
contaminated” surgical wound? higher cost
(A) An uninfected operative wound in which no (B) Lower recurrence rate and less postoperative pain
Tension-free hernia repair, which commonly uses prosthetic mesh, reduces the tension on surrounding tissues, thus improving healing and decreasing the likelihood of recurrence. Because the tissue edges are not forcibly approximated, the procedure generally causes less postoperative pain. This directly makes option (B) correct. Option (A) incorrectly suggests a faster recurrence rate, contradicting evidence that tension-free techniques usually reduce recurrence. Option (C) states that tension-free repair has higher complication rates, which is not accurate — while mesh-related complications can occur, tension-free repairs do not universally carry significantly higher overall complication rates compared to conventional repairs. Option (D) mentions a significantly higher cost but states the same recurrence rate; however, tension-free repairs have demonstrated better long-term outcomes with decreased recurrence. Hence, the primary advantage is indeed a lowered recurrence risk and reduced pain, correlating with better patient satisfaction and improved overall results.

inflammation is encountered
(B) An operative wound in which the respiratory 6. Which surgical prophylaxis measure is best
tract is entered under controlled conditions for preventing surgical site infections in clean-
(C) An accidental wound with extensive tissue contaminated cases?
trauma and gross contamination (A) Administering antibiotics postoperatively for
(D) An old traumatic wound with retained 7 days
devitalized tissue and existing infection (B) Giving a single preoperative dose of a
suitable antibiotic
(C) Starting antibiotics 24 hours after surgery
(B) An operative wound in which the respiratory tract is entered under controlled conditions
Option (B) is correct because “clean-contaminated” wounds typically involve planned entry into the respiratory, alimentary, or genitourinary tract under controlled circumstances without unusual contamination. These procedures are not entirely sterile since potentially colonized organs or tracts are opened. However, if the procedure is well-controlled and without spillage, the wound is classified as clean-contaminated. In contrast, option (A) describes a clean wound where no organ system with bacteria is entered. Option (C) indicates a contaminated or even dirty wound if there is gross contamination, usually from an uncontrolled entry or trauma. Option (D) describes a dirty wound with established infection or necrotic tissue. Understanding proper wound classification is essential for deciding antibiotic prophylaxis and for predicting postoperative infection risk. Since surgical classification guides infection control measures, recognizing that option (B) specifies a controlled environment, yet with potential bacterial flora, fits the criteria for a clean-contaminated wound classification.

2. Which nerve is most commonly injured in a (D) Avoiding antibiotic use unless signs of
McBurney’s incision? infection occur
(A) Iliohypogastric nerve (B) Giving a single preoperative dose of a suitable antibiotic
The best prophylaxis for clean-contaminated surgeries is a single dose of an appropriate antibiotic administered within 30-60 minutes before incision. This matches option (B). The rationale is that antibiotic levels in the tissue should be sufficient at the time of the initial incision to combat potential bacterial contamination. Option (A) prolongs antibiotic use to 7 days, which is generally not recommended unless complications arise, as it may promote antibiotic resistance and increase cost. Option (C) starts antibiotics too late, missing the critical window when contamination first occurs. Option (D) suggests withholding prophylaxis altogether, which is not the standard for clean-contaminated operations; prophylactic antibiotics are beneficial in surgeries where mucosal surfaces with bacterial flora will be entered. Thus, the single preoperative dose is optimal for preventing surgical site infections while minimizing the risk of antibiotic-associated side effects and resistance.

(B) Ilioinguinal nerve


(C) Genitofemoral nerve 7. A 45-year-old smoker presents with a 2 cm ulcer
(D) Lateral femoral cutaneous nerve on the lateral border of the tongue, non-healing for
3 months. The most likely diagnosis is:
(A) Leukoplakia
(A) Iliohypogastric nerve
The iliohypogastric nerve (option A) is most at risk in a McBurney’s incision, which is commonly used for open appendectomy. This nerve provides sensory innervation to the skin above the pubis and part of the gluteal region. Because the incision is made obliquely in the right lower quadrant, it may cut through or damage the iliohypogastric nerve as it travels superficially. The ilioinguinal nerve (option B) also lies in that region but typically runs slightly lower and is more commonly implicated in inguinal incisions rather than the classic McBurney approach. The genitofemoral nerve (option C) emerges at L1-L2 and primarily supplies the cremaster muscle and parts of the thigh but is less frequently involved with McBurney incisions. The lateral femoral cutaneous nerve (option D) passes near the anterior superior iliac spine and is less likely to be injured during a standard right lower quadrant incision for an appendectomy.

3. During laparoscopic procedures, why is CO₂ the (B) Squamous cell carcinoma
preferred insufflation gas? (C) Oral thrush
(A) It is highly flammable and helps identify (D) Traumatic ulcer
leaks (B) Squamous cell carcinoma
A chronic, non-healing ulcer on the lateral border of the tongue, especially in a middle-aged smoker, strongly suggests squamous cell carcinoma (option B). Smoking and tobacco use are major risk factors, and the lateral border of the tongue is a common site for malignant lesions in the oral cavity. Leukoplakia (option A) typically appears as a white patch that cannot be scraped off, and while it can undergo malignant transformation, it is not typically presented solely as an ulcer. Oral thrush (option C) tends to be a fungal infection characterized by white plaques that can be rubbed off, exposing a raw surface beneath. A traumatic ulcer (option D) may occur from a sharp tooth edge or repeated trauma, but it should resolve when the source of irritation is removed. The chronicity, risk factors, and location all point to squamous cell carcinoma as the likely diagnosis.

(B) It is easily absorbed and has low flammability


(C) It offers better visibility by drying out tissues 8. Arrange the phases of wound healing in correct
(D) It causes minimal acidosis compared to chronological order:
helium 1. Maturation (remodeling)
2. Hemostasis
3. Proliferation
(B) It is easily absorbed and has low flammability
Carbon dioxide (CO₂) is used because it is rapidly absorbed from the peritoneal cavity and is essentially nonflammable, reducing the risk of fire during electrosurgery. This feature makes option (B) the best explanation. Furthermore, CO₂ is inexpensive and widely available. Contrary to option (A), CO₂ is not flammable — this feature is actually why it is used, as a flammable gas would be hazardous in surgical settings involving cautery. Option (C) suggests that a gas might dehydrate tissues, but while some dryness may occur, it is not the primary reason for CO₂ use. Option (D) states it causes minimal acidosis compared to helium, but CO₂ can, in fact, cause a mild respiratory acidosis when absorbed, requiring careful monitoring of ventilation, although it is more easily managed than if a less soluble gas were used. Hence, its ease of absorption and low flammability remain the main reasons for its widespread use.

4. All of the following are advantages of minimal 4. Inflammatory phase


invasive surgery (MIS) except: Select the option for correct order :
(A) Reduced postoperative pain (A) 2 → 4 → 3 → 1
(B) Faster recovery and shorter hospital stay (B) 4 → 2 → 1 → 3
(C) Higher likelihood of massive hemorrhage (C) 2 → 3 → 4 → 1
(D) Improved cosmesis (D) 4 → 3 → 2 → 1
(C) Higher likelihood of massive hemorrhage (A) 2 → 4 → 3 → 1
Minimally invasive surgery (MIS) offers several benefits compared to open surgery, such as decreased postoperative pain, shorter hospital stays, and better cosmetic results. Options (A), (B), and (D) are true advantages of MIS. Option (C) is incorrect because MIS does not typically have a higher likelihood of massive hemorrhage. While any surgery can lead to bleeding complications, laparoscopic or robotic approaches are associated with smaller incisions and more precise surgical dissection, which can reduce blood loss. However, if bleeding does occur laparoscopically, it may be challenging to control depending on the surgeon’s expertise and visualization, but the incidence of massive hemorrhage is not inherently higher with MIS. Identifying the one exception among multiple commonly recognized benefits highlights the question’s emphasis on analyzing standard knowledge regarding laparoscopic or endoscopic techniques. Thus, (C) is the “except” or incorrect statement in the list. Wound healing proceeds through a well-recognized sequence of events. Immediately after injury, hemostasis (step 2) occurs as blood vessels constrict and the clotting cascade is activated to prevent further blood loss. This is followed by the inflammatory phase (step 4), during which neutrophils, macrophages, and other inflammatory cells clean the wound and release cytokines. Next comes proliferation (step 3), where fibroblasts synthesize collagen, new blood vessels form (angiogenesis), and the wound bed is built up. Finally, the remodeling or maturation phase (step 1) refines and strengthens the collagen network over time. Option (A) correctly places these stages in chronological order: Hemostasis → Inflammation → Proliferation → Maturation. Option (B), (C), and (D) are incorrect sequences that do not align with the standard wound-healing timeline. This systematic progression is crucial to understanding appropriate wound management and promoting optimal healing conditions.

5. In comparing tension-free hernia repair with 9. In the management of acute limb ischemia, which
conventional herniorrhaphy, the main advantage feature is most indicative of irreversible limb
of tension-free repair is: damage?
(A) Faster recurrence rate but less pain (A) Severe pain
(B) Lower recurrence rate and less postoperative (B) Poikilothermia (coldness)
pain (C) Paralysis
(C) Higher complication rates but minimal (D) Pallor
discomfort (C) Paralysis
Acute limb ischemia is often summarized by the “6 Ps”: Pain, Pallor, Pulselessness, Paresthesia, Paralysis, and Poikilothermia. Of these, paralysis (option C) is a late, grave sign indicating possible irreversible muscle and nerve damage. While severe pain (option A) is the earliest and most common symptom, it does not necessarily imply permanent damage if promptly treated. Pallor (option D) and poikilothermia (option B) highlight compromised blood flow, but they can still be potentially reversible if revascularization is achieved in time. Paralysis signals that necrosis of muscle and nerve tissues may have already occurred, meaning salvage of the limb becomes far less likely. Identifying key features in acute limb ischemia is vital for swift intervention with anticoagulation, thrombolysis, or surgical embolectomy. Recognizing paralysis as a late sign underscores the urgency to treat earlier stages when the limb can still be saved and function preserved.
MEDINK GRAND TEST 03

10. In a patient with suspected acute cholecystitis, the 15. In a patient with multiple rib fractures and flail
best initial imaging study is: chest, the key action to stabilize breathing is:
(A) MRI of the abdomen (A) Immediate chest tube insertion
(B) Plain X-ray abdomen (B) Positive pressure ventilation with adequate
(C) Ultrasound of the right upper quadrant analgesia
(D) CT scan of the abdomen (C) Application of external splint over the chest
(C) Ultrasound of the right upper quadrant
The best initial imaging modality for suspected acute cholecystitis is a right upper quadrant ultrasound (option C). Ultrasound is highly sensitive for detecting gallstones and can demonstrate gallbladder wall thickening, pericholecystic fluid, and a positive sonographic Murphy’s sign, helping confirm the diagnosis. It is cost-effective, readily available, and avoids ionizing radiation. MRI (option A) may be useful for complicated cases or when more detailed biliary anatomy is required (MRCP), but it is not the standard initial test. A plain X-ray (option B) has limited utility in diagnosing cholecystitis because most gallstones are not radiopaque. CT scan (option D) can be helpful if the diagnosis is uncertain or to evaluate complications, but it is not the first-line imaging study. Thus, ultrasound remains the cornerstone for initial evaluation of acute cholecystitis due to its high specificity and the ability to visualize gallstones in real time.
(D) Simple observation until healing occurs

11. A 65-year-old patient with symptomatic carotid


(B) Positive pressure ventilation with adequate analgesia
A flail chest occurs when a segment of the rib cage becomes detached from the rest of the thoracic cage due to multiple rib fractures. This leads to paradoxical movement of the flail segment and impairs ventilation. The key management step is ensuring adequate ventilation and pain control, typically through positive pressure ventilation (option B). This approach stabilizes the chest wall internally, providing the essential support for effective gas exchange. While a chest tube (option A) may be necessary if there is an associated pneumothorax or hemothorax, it is not the universal first step in all flail chest cases. External splints or wraps (option C) are largely outdated and can restrict normal respiratory mechanics. Simple observation (option D) fails to address the acute respiratory compromise. Adequate analgesia with nerve blocks or epidurals is also critical, as pain leads to inadequate ventilation and possible respiratory deterioration.

artery stenosis of 75 is best managed by: 16. A clinical trial comparing laparoscopic versus
(A) Carotid endarterectomy open appendectomy shows a p-value of 0.03 for
(B) Antiplatelet therapy alone a difference in operative time. Interpreting this
(C) Immediate anticoagulation with heparin result, we can say:
(D) Carotid artery stenting in all cases (A) There is a 3 chance the result is due to random
(A) Carotid endarterectomy
For symptomatic carotid stenosis of more than 70, carotid endarterectomy (CEA) is the most definitive treatment (option A). Clinical trials have shown that CEA significantly reduces the risk of stroke in patients with high-grade stenosis (70–99) who are symptomatic, provided surgical risk is acceptable. While antiplatelet therapy (option B) is important for medical management, it may not be sufficient alone in high-grade stenosis. Immediate anticoagulation with heparin (option C) can be used in certain acute settings, but it is not the definitive therapy for significant symptomatic stenosis. Carotid artery stenting (option D) can be considered in selected high-risk surgical patients or those with unfavorable neck anatomy, but it is not automatically indicated for all. Thus, the gold standard definitive intervention for a 75 symptomatic carotid stenosis in a fit patient is carotid endarterectomy, offering a proven stroke reduction benefit in properly selected individuals.
variation
(B) The operative time difference is clinically
12. A 30-year-old male presents with a two-day insignificant
history of right groin pain and a tender swelling (C) The difference is statistically significant at
that becomes more prominent on coughing but the 5 level
cannot be reduced on examination. (D) The difference must always be clinically
(A) Reducible inguinal hernia significant
(B) Strangulated indirect inguinal hernia
(C) Femoral hernia
(C) The difference is statistically significant at the 5 level
A p-value of 0.03 indicates that if there were truly no difference in operative time between laparoscopic and open appendectomy, the probability of observing the measured difference purely by chance is 3. Conventionally, a p-value below 0.05 suggests statistical significance (option C). Interpretation of this p-value means the difference found is unlikely to be due to random variation. Option (A) is not fully accurate — though 3 is the nominal chance, the statement can be misleading because the p-value is not the exact probability that the result is due to chance in all contexts. Option (B) incorrectly conflates statistical significance with clinical significance; a p-value does not necessarily address clinical relevance. Option (D) also errs by assuming statistical significance automatically equates to clinical significance. Statistical significance at the 5 level is the correct interpretation, highlighting a likely genuine difference in operative times between the two groups.

(D) Psoas abscess 17. In the management protocol of acute pancreatitis,


(B) Strangulated indirect inguinal hernia
A tender swelling in the right groin that is exacerbated by coughing but cannot be reduced suggests a strangulated hernia. Since the swelling is in the inguinal region, an indirect inguinal hernia (option B) is more likely than a femoral hernia (option C). Strangulation implies compromised blood supply to the herniated contents, indicated by increased tenderness and pain. A reducible hernia (option A) would be less tender and could be pushed back (reduced) into the abdominal cavity. A psoas abscess (option D) generally presents with back or flank pain and is less likely to protrude in the groin region in a manner that is accentuated by coughing. The inability to reduce the swelling in conjunction with acute pain points toward strangulation. Strangulated indirect inguinal hernias necessitate prompt surgical intervention to avoid ischemic injury to the entrapped bowel.
which step is incorrect?
(A) Providing adequate IV fluids
13. A 60-year-old man presents with a painful, (B) Early enteral feeding if possible
bluish discoloration around the anus suggestive (C) Immediate ERCP for all patients
of a thrombosed external hemorrhoid. The best (D) Pain control with analgesics
immediate management is:
(A) Sitz baths and stool softeners alone
(C) Immediate ERCP for all patients
In acute pancreatitis management, aggressive intravenous fluid resuscitation (option A) is essential to maintain intravascular volume, as significant fluid sequestration can occur. Early enteral feeding (option B), preferably within 24–48 hours, has been shown to reduce infections and improve outcomes compared to prolonged fasting. Adequate pain control (option D) with opioids or other analgesics is vital for patient comfort. Routine immediate ERCP (option C) is not indicated for all patients. ERCP is reserved for cases with concurrent cholangitis, persistent biliary obstruction, or severe gallstone pancreatitis with ongoing biliary obstruction. Performing ERCP in every patient with acute pancreatitis can expose them to unnecessary procedural risks, including further pancreatic ductal irritation, bleeding, or infection. Thus, option (C) is the incorrect step, whereas the other measures are standard, appropriate elements of acute pancreatitis management protocols.

(B) High-fiber diet and analgesics 18. A 20-year-old with acute appendicitis has an
(C) Urgent excision of the thrombosed Alvarado score of 9. The likelihood of true
hemorrhoid under local anesthesia appendicitis is approximately:
(D) Observation for spontaneous resolution (A) 50
(C) Urgent excision of the thrombosed hemorrhoid under local anesthesia
In an acutely painful thrombosed external hemorrhoid, urgent excision of the clot under local anesthesia can provide rapid pain relief (option C). The characteristic bluish lump near the anal margin is often excruciating, and immediate surgical intervention can significantly reduce symptoms. Options (A) and (B), which focus on conservative measures like sitz baths, stool softeners, high-fiber diet, and analgesics, are beneficial for non-thrombosed or less painful hemorrhoids, but they may not immediately relieve the severe pain associated with thrombosis. Observation (option D) is an option for minor discomfort, as thrombosed hemorrhoids can resolve spontaneously over weeks, but it prolongs the patient’s pain. Hence, if seen early (particularly within 48–72 hours of onset), an urgent excision yields the best short-term relief. The procedure is relatively simple and is often done in an outpatient setting, significantly improving the patient’s quality of life.
(B) 60
(C) 80
14. Regarding pheochromocytoma management, (D) Greater than 90
which of the following statements are correct?
1. Alpha-blockade is initiated before beta-
(D) Greater than 90
The Alvarado score is a clinical tool used to quantify the likelihood of acute appendicitis based on symptoms, signs, and lab findings. A high score, such as 9 or 10, is strongly suggestive of acute appendicitis, with a diagnostic accuracy typically exceeding 90 (option D). Lower scores (less than 5) make appendicitis less likely, whereas scores between 5 and 7 warrant further investigation, often with imaging. Options (A), (B), and (C) underestimate the probability of acute appendicitis at an Alvarado score of 9. In most clinical settings, a score this high would often lead to an urgent surgical consultation for appendectomy without necessarily requiring further imaging, depending on the patient’s presentation. Thus, (D) is correct, reflecting that a patient with a score of 9 has a high probability — usually above 90 — of having true acute appendicitis.

blockade 19. Long-standing gastroesophageal reflux disease


2. Beta-blockade may be started first to control (GERD) can lead to Barrett’s esophagus due to:
tachycardia (A) Fibrous tissue deposition in the esophagus
3. Surgical resection is definitive (B) Metaplastic change in the lower esophageal
4. A CT scan or MRI is used for localization mucosa
Options : (C) Infiltration of eosinophils causing esophagitis
(A) 1, 2, 4 are correct (D) Acute ulcerative lesions in the proximal
(B) 1, 3, 4 are correct esophagus
(C) 2 and 3 are correct (B) Metaplastic change in the lower esophageal mucosa
Chronic exposure to acid reflux in GERD can induce a metaplastic change in the distal esophageal epithelium, where normal squamous cells are replaced by columnar epithelium more typical of intestinal mucosa. This phenomenon, known as Barrett’s esophagus (option B), is significant because it increases the risk of developing esophageal adenocarcinoma. Option (A) describes fibrotic changes, which could lead to strictures but not Barrett’s. Eosinophilic infiltration (option C) is more characteristic of eosinophilic esophagitis rather than Barrett’s. Acute ulcerative lesions in the proximal esophagus (option D) are not typical of chronic GERD-induced Barrett’s, which occurs in the lower esophagus. Barrett’s esophagus exemplifies a cause-and-effect relationship where chronic acid reflux leads to adaptive, potentially precancerous changes in the lining cells. Identifying Barrett’s endoscopically is crucial because of the malignant transformation risk and the potential need for surveillance endoscopy.

(D) All are correct


(B) 1, 3, 4 are correct
Before surgical resection of a pheochromocytoma, alpha-blockade must be established first, commonly using nonselective alpha-adrenergic blockers like phenoxybenzamine (statement 1). If the patient has tachycardia or other beta-mediated effects, beta-blockers are added only after adequate alpha-blockade to prevent unopposed alpha stimulation. Hence, statement 2 suggesting beta-blockade may be started first is incorrect. Surgical excision (statement 3) is the definitive curative option once the patient is hemodynamically stabilized. Imaging with CT or MRI (statement 4) is routinely used to localize the adrenal tumor or extra-adrenal paragangliomas. Thus, options 1, 3, and 4 are correct. Recognizing the sequence of blockade is crucial, as initiating beta-blockade first can precipitate a hypertensive crisis. Therefore, the correct combination is option (B). This K-type question tests knowledge of both the pharmacologic management and the diagnostic workup needed prior to definitive surgical removal of pheochromocytoma.
20. Match the surgical instrument with its primary use:
page 30 NEET PG Surgery

Column I Column II (C) Sengstaken-Blakemore tube placement with


(Instrument (Use) resuscitation
(D) Transjugular intrahepatic portosystemic
1. Kocher clamp a. Grasping delicate
shunt (TIPS) as first-line
tissues (e.g., bowel)
without trauma (C) Sengstaken-Blakemore tube placement with resuscitation
In a patient with acute variceal bleeding who is hemodynamically unstable and continues to bleed, a Sengstaken-Blakemore tube can be placed as a temporizing measure to mechanically compress the varices. Concurrent resuscitation with blood products and IV fluids is critical. Endoscopic therapies, such as variceal ligation or sclerotherapy (options A and B), remain essential treatments, but in severe ongoing bleeding with instability, balloon tamponade can stabilize the patient before endoscopic intervention. Vasoactive drugs like octreotide may also be administered. TIPS (option D) is generally reserved for cases that fail endoscopic therapy or rebleed despite adequate treatment and is not typically the immediate first-line in an actively unstable presentation. By achieving tamponade, option (C) helps control life-threatening hemorrhage, buying time for definitive endoscopic or interventional radiologic procedures. Thus, the combination of mechanical tamponade with aggressive resuscitation is the correct immediate approach in this critical scenario.

2. Babcock clamp b. Grasping tough 24. A 25-year-old male is rushed to the ER after a
structures (e.g., fascia) high-speed road accident. He has an open fracture
with teeth
of the tibia with significant soft tissue damage.
3. Allis forceps c. Clamping bleeding (A) Immediate wound closure in the emergency
vessels securely with a room
tooth (B) Thorough debridement, stabilization, and
4. Lane’s tissue d. WWHolding or delayed closure
forceps retracting tissues with (C) Casting the limb without debridement
multiple teeth (D) Splinting and discharge with oral antibiotics
Select the correct answer using the code given (B) Thorough debridement, stabilization, and delayed closure
In an open tibial fracture with substantial soft tissue damage, the priority is to prevent and control infection while facilitating appropriate bone stabilization. Thorough surgical debridement (removal of contaminated, devitalized tissue) and irrigation reduce the bacterial load, aiding in infection prevention. Stabilization of the fracture, commonly with an external fixator or internal fixation depending on the scenario, is essential. Delayed closure or use of skin grafts/flaps may be required once the wound bed is clean and viable tissue is confirmed. Immediate wound closure (option A) risks sealing in infection. Casting the limb without debridement (option C) invites deep infection. Splinting with oral antibiotics and discharge (option D) is inappropriate because severe open fractures demand operative intervention and likely intravenous antibiotics. Hence, the correct management is thorough debridement, fracture stabilization, and delayed closure when safe, ensuring better long-term healing and reduced complications.

below:
(A) 1-c, 2-a, 3-d, 4-b 25. A 32-year-old woman complains of a painless
(B) 1-d, 2-b, 3-a, 4-c breast lump for 2 months. On examination, a 3 cm
(C) 1-c, 2-a, 3-b, 4-d mobile mass in the upper outer quadrant with no
(D) 1-b, 2-d, 3-c, 4-a axillary lymphadenopathy is noted.
(A) Immediate mastectomy
(B) Ultrasound-guided core needle biopsy
(A) 1-c, 2-a, 3-d, 4-b
The Kocher clamp (1) features a transverse serration and a sharp tooth at the tip, making it ideal for clamping bleeding vessels securely, corresponding to (c). The Babcock clamp (2) is designed with a fenestrated, smooth grasping surface suitable for delicate tissues like the intestine or fallopian tubes without causing trauma, matching (a). Allis forceps (3) have multiple teeth, commonly used to hold or retract heavy tissue or edges of fascia, which is (d). Lane’s tissue forceps (4) typically have strong teeth designed to grip tougher tissues or fascia, matching (b). Thus, (A) 1-c, 2-a, 3-d, and 4-b is the correct match. The arrangement ensures each instrument is linked with its primary use, testing familiarity with common surgical tools. Proper instrument selection is critical in surgery to minimize tissue damage while effectively controlling bleeding or manipulating structures.

21. Assertion (A) : In chronic venous insufficiency, (C) Fine needle aspiration cytology and observe
compression stockings are recommended. (D) Mammography alone is sufficient
Reason (R) : Compression improves venous (B) Ultrasound-guided core needle biopsy
A painless breast lump in a relatively young woman warrants a diagnostic workup to rule out malignancy. Ultrasound-guided core needle biopsy (option B) provides tissue for histopathological analysis, offering more definitive information than fine needle aspiration cytology (FNAC), which only yields cytological data. Option (A), immediate mastectomy, is too radical without a confirmed diagnosis. Option (C), FNAC and observation, might be insufficient because even if the FNAC is benign, a core biopsy is generally recommended for a suspicious mass. Mammography alone (option D) is not adequate to establish a definitive diagnosis; while imaging can suggest features of malignancy or benign lesions, tissue diagnosis remains necessary for definitive management. Ultrasound guidance improves the accuracy of the biopsy, especially if the lesion is more apparent on ultrasound. Therefore, (B) is the correct next step in evaluating this suspicious breast mass.

return and reduces edema.


(A) Both A and R are true, and R is the correct 26. A 60-year-old patient with chronic renal failure
explanation of A on dialysis presents with severe abdominal pain.
(B) Both A and R are true, but R is not the correct Imaging suggests acute mesenteric ischemia of
explanation of A the superior mesenteric artery with signs of bowel
(C) A is true, but R is false infarction and peritonitis.
(D) A is false, but R is true (A) Immediate laparotomy and resection of
nonviable bowel
(B) Angioplasty and stenting without surgery
(A) Both A and R are true, and R is the correct explanation of A
In chronic venous insufficiency (CVI), damaged or incompetent venous valves lead to pooling of blood in the lower limbs, causing edema, skin changes, and varicose veins. Compression stockings help by externally supporting the venous system and promoting venous blood return to the heart, thus reducing edema. Therefore, the assertion that compression stockings are recommended (A) is correct. The reason (R) — that compression improves venous return and reduces edema — is also correct and directly explains why compression therapy is beneficial in CVI. Option (B) suggests that while both are correct, the reason would not explain the assertion, which is not the case here. Options (C) and (D) imply falsities in either the assertion or the reason. Clearly, compression therapy’s effect on venous return is precisely the rationale for its recommendation, making option (A) accurate.

22. A 55-year-old diabetic with a swollen, painful left (C) Conservative management with antibiotics
foot and foul-smelling discharge is diagnosed with (D) Exploratory laparoscopy only
gas gangrene on radiographic evaluation. (A) Immediate laparotomy and resection of nonviable bowel
In acute mesenteric ischemia with suspected bowel infarction and peritonitis, time is critical. The correct management (option A) is an urgent laparotomy to assess bowel viability, resect necrotic segments, and potentially restore blood flow, often via embolectomy or vascular bypass if feasible. Patients with chronic renal failure are at elevated risk of vascular disease, and rapid surgical intervention is necessary to prevent sepsis and further complications. Angioplasty or stenting (option B) may be appropriate in certain cases of mesenteric ischemia without infarction or peritonitis but is not sufficient when necrosis is suspected. Conservative management (option C) is inappropriate for infarction and peritonitis, as bowel perforation and sepsis are imminent risks. Exploratory laparoscopy (option D) might be diagnostic in uncertain cases, but in the setting of acute infarction and peritonitis, a formal laparotomy is more practical for immediate definitive treatment. Thus, urgent surgical intervention is mandatory.

(A) Debridement plus intravenous antibiotics


(B) Hyperbaric oxygen therapy alone 27. A 70-year-old male with a three-month history of
(C) Foot elevation and local wound dressing only lower urinary tract symptoms, including nocturia
(D) Immediate minor amputation without and difficulty initiating urination, now has acute
antibiotic coverage urinary retention. On examination, a palpable
bladder is noted, and a digital rectal exam reveals
an enlarged, firm prostate. Serum PSA is slightly
(A) Debridement plus intravenous antibiotics
Gas gangrene, frequently caused by Clostridium perfringens, is a surgical emergency characterized by tissue necrosis, gas production, and a risk of systemic toxicity. The cornerstone of treatment (option A) is prompt and aggressive surgical debridement to remove all devitalized tissue that fosters bacterial growth. Intravenous antibiotic therapy, typically with penicillin and/or clindamycin, is essential to control the infection systemically. Hyperbaric oxygen therapy (option B) can be an adjunct, improving oxygen delivery to tissues and inhibiting the anaerobic bacteria, but it should not be used alone. Foot elevation and local wound care (option C) are supportive measures but are wholly insufficient for this life-threatening infection. Immediate amputation (option D) can be required in severe, limb-threatening cases, but must be coupled with appropriate antibiotic therapy. Early debridement plus antibiotics is the priority, aiming to halt the rapid progression of gas gangrene and preserve as much viable tissue as possible.

23. A 50-year-old alcoholic male with hematemesis elevated.


is found to have bleeding esophageal varices on (A) Immediate transurethral resection of the
endoscopy, with hemodynamic instability and prostate (TURP)
ongoing blood loss. (B) Trial of alpha-1 blocker medication and
(A) Endoscopic variceal ligation only catheterization
(B) Endoscopic sclerotherapy plus vasoactive (C) MRI pelvis and prostate biopsy immediately
drugs (D) Watchful waiting with no intervention
MEDINK GRAND TEST 03

(B) Trial of alpha-1 blocker medication and catheterization (B) 50–150 pg/mL
A 70-year-old man with bothersome lower urinary tract symptoms and acute urinary retention most likely has benign prostatic hyperplasia (BPH), although prostate cancer remains a differential due to firm enlargement and mildly elevated PSA. The initial management usually involves relieving retention via catheterization and starting an alpha-1 blocker, such as tamsulosin, to improve urine flow. Immediate TURP (option A) is usually not the first step unless medical therapy fails or the patient experiences recurrent retention and complications. MRI pelvis and biopsy (option C) are considered if there is a high suspicion of malignancy, but mild PSA elevation and firmness do not alone mandate urgent biopsy without further evaluation (such as repeated PSA, imaging, or suspicion via DRE). Watchful waiting (option D) is not appropriate when the patient is in acute retention and symptomatic. Thus, (B) is the best initial approach to manage both the retention and the possible BPH. The normal fasting serum gastrin level in adults typically ranges between 50 and 150 pg/mL, making option (B) correct. Gastrin is a hormone produced by G cells in the antrum of the stomach that stimulates gastric acid secretion. Extremely elevated levels of serum gastrin are associated with conditions such as Zollinger-Ellison syndrome (ZES), where gastrinoma tumors produce excessive gastrin. Levels in ZES can reach well above 1000 pg/mL, causing severe peptic ulcer disease and diarrhea. Option (A) is too low to represent the normal range, and options (C) and (D) are significantly elevated, indicating a pathological state. Accurate knowledge of normal gastrin levels aids clinicians in suspecting and investigating hypergastrinemic states, particularly in patients presenting with refractory peptic ulcer disease or recurrent ulcers. Measuring gastrin levels, along with gastric pH, can help guide further diagnostic steps such as secretin stimulation tests or imaging for gastrinoma.

28. A 58-year-old obese female with poorly controlled 32. In the pathophysiology of acute cholecystitis, the
diabetes presents with a non-healing ulcer over key initiating event is:
the plantar surface of her foot for two months. (A) Bacterial infection from hematogenous
Examination reveals foul-smelling discharge, spread
undermined edges, and exposed tendon. She has (B) Obstruction of the cystic duct, leading to
peripheral neuropathy and diminished pedal pulses. distension
Laboratory results show elevated inflammatory (C) Ischemic necrosis of the gallbladder wall
markers and poor glycemic control (HbA1c of 10). (D) Immune-mediated damage to gallbladder
(A) Continue local dressing and outpatient mucosa
follow-up (B) Obstruction of the cystic duct, leading to distension
Acute cholecystitis typically begins with obstruction of the cystic duct, most often by a gallstone impacted in the Hartmann’s pouch or cystic duct. This obstruction (option B) leads to accumulation of bile, increased pressure, and distension within the gallbladder. Subsequently, the gallbladder wall becomes inflamed, which can become infected secondarily by bacteria. Although bacterial infection may play a role, it usually occurs after the initial obstruction rather than being the primary trigger (option A). Ischemic necrosis (option C) can develop as a complication if the inflammation and pressure compromise the gallbladder’s blood supply, but it is not the initiating event. Immune-mediated damage (option D) is not typically the main pathway in acute calculous cholecystitis. Recognizing that cystic duct obstruction triggers the inflammatory process is crucial, explaining why prompt relief of the obstruction (e.g., cholecystectomy) is the definitive treatment to prevent progression to gangrene or perforation.

(B) Surgical debridement, appropriate antibiotics,


and glycemic optimization 33. All of the following are risk factors for breast
(C) Immediate below-knee amputation cancer except:
(D) Intravenous antibiotics alone (A) Early menarche and late menopause
(B) Nulliparity or late first pregnancy
(C) Use of combined estrogen-progestin hormone
(B) Surgical debridement, appropriate antibiotics, and glycemic optimization
A diabetic foot ulcer with foul-smelling discharge and exposed tendon suggests a severe, possibly infected wound with underlying osteomyelitis risk. In such patients, surgical debridement is crucial for removing necrotic tissue that sustains the infection. Concurrent antibiotic therapy tailored to common diabetic foot pathogens (including Gram-positive, Gram-negative, and anaerobes) is essential for controlling infection. Optimizing glycemic control is equally critical because hyperglycemia impairs wound healing and immune response. Local dressing alone (option A) is inadequate given the severity of infection. Immediate amputation (option C) may be necessary if there is extensive necrosis or life-threatening infection, but it should not be the first intervention unless tissue is unsalvageable. Intravenous antibiotics alone (option D) would not suffice if devitalized tissue persists. Thus, the combined approach of debridement, antibiotic therapy, and better glucose management (option B) is key to limb salvage and improved outcomes in diabetic foot ulcers.

29. A 45-year-old man with a history of chronic replacement


alcoholic pancreatitis presents with worsening (D) High intake of citrus fruits
epigastric pain radiating to the back. Over the (D) High intake of citrus fruits
Early menarche and late menopause (option A) lengthen the exposure to estrogen over a lifetime, thereby increasing the risk of breast cancer. Nulliparity or having a first pregnancy at a later age (option B) similarly prolongs uninterrupted estrogen stimulation. Combined estrogen-progestin hormone replacement therapy (option C) is also associated with a small but significant increase in breast cancer risk with prolonged use. High intake of citrus fruits (option D) has not been identified as a known risk factor; in fact, dietary habits that include fruits, vegetables, and a balanced diet might be protective or neutral rather than risk-raising. Therefore, (D) is the exception among the listed factors. Understanding breast cancer risk helps in early identification and possible risk reduction through lifestyle modification, screening, and judicious use of hormone replacement when needed.

past six months, he has lost 15 kg, complains of


steatorrhea, and now appears emaciated. CT scan 34. Which statement best compares laparoscopic
of the abdomen shows a dilated pancreatic duct cholecystectomy to open cholecystectomy?
with multiple calcifications. Laboratory tests reveal (A) Laparoscopic approach guarantees zero risk
low serum albumin and reduced fecal elastase, of bile duct injury
indicating significant exocrine insufficiency. (B) Laparoscopic approach typically has shorter
(A) Immediate pancreatoduodenectomy hospital stay
(B) Pancreatic enzyme replacement therapy and (C) Open approach has fewer postoperative
nutritional support complications overall
(C) Endoscopic drainage of pseudocyst (D) Open approach is always the procedure of
(D) No specific intervention required choice for acute cholecystitis
(B) Laparoscopic approach typically has shorter hospital stay
Laparoscopic cholecystectomy is associated with several advantages over the open procedure, including reduced postoperative pain, faster recovery, and shorter hospital stay (option B). However, bile duct injury can still occur with the laparoscopic approach (option A), particularly when the anatomy is misidentified, so it does not provide a “zero risk.” While the open approach remains an important option in complicated cases, it is not devoid of complications and generally leads to a longer recovery period (negating option C). Early laparoscopic cholecystectomy is considered safe and beneficial in acute cholecystitis, so claiming the open procedure is always the choice for acute cholecystitis (option D) is incorrect. The shorter hospital stay is a key element when comparing both procedures, which correlates with earlier return to normal activities and improved patient satisfaction in uncomplicated gallbladder disease.

(B) Pancreatic enzyme replacement therapy and nutritional support


Chronic alcoholic pancreatitis with a dilated pancreatic duct, calcifications, and exocrine insufficiency (evidenced by steatorrhea and low fecal elastase) requires comprehensive management focusing on pain control and nutritional optimization. Pancreatic enzyme replacement therapy helps with malabsorption and steatorrhea, improving nutritional status. Adequate nutritional support, including high-protein diets and correction of vitamin deficiencies, is essential for rebuilding body mass and preventing further deterioration. Option (A), pancreatoduodenectomy, is a major surgery typically reserved for refractory cases or when malignancy cannot be excluded. Option (C), endoscopic drainage, might be necessary if a pseudocyst or obstructive lesion is present causing persistent pain or complications, but the question indicates calcifications in the duct rather than a single large pseudocyst. Suggesting no specific intervention (option D) ignores the patient’s ongoing weight loss, malabsorption, and pain. Hence, pancreatic enzyme replacement and nutritional support remain the mainstay of treatment.

30. Which of the following best defines a “neoadjuvant 35. In a patient with deep vein thrombosis (DVT), the
therapy” approach in surgical oncology? best initial anticoagulation management is:
(A) Chemotherapy given after surgery to (A) Low molecular weight heparin (LMWH)
eliminate microscopic disease subcutaneously
(B) Radiotherapy given before surgery to shrink (B) Warfarin initiated alone orally
a tumor (C) Aspirin 75 mg daily
(C) Supportive therapy to manage symptoms (D) Direct oral anticoagulant (DOAC) 24 hours
during surgical recovery after diagnosis
(D) Hormonal therapy given at any stage of (A) Low molecular weight heparin (LMWH) subcutaneously
The best initial management for acute DVT involves administering low molecular weight heparin (LMWH) subcutaneously, typically enoxaparin. LMWH provides predictable anticoagulant effects and does not require constant monitoring as with unfractionated heparin. Warfarin (option B) can be started concurrently but must overlap with LMWH or unfractionated heparin for at least five days, or until the INR is within the therapeutic range for at least 24 hours, because warfarin initially decreases protein C and S, potentially leading to a transient hypercoagulable state. Aspirin at a low dose (option C) is not sufficient as sole therapy for DVT prophylaxis or treatment. While direct oral anticoagulants (DOACs) (option D) are valid treatment options, the standard approach, particularly in hospitalized or more severe cases, remains starting LMWH. Hence, LMWH subcutaneously is the best initial strategy to rapidly achieve a therapeutic anticoagulant effect and prevent clot propagation.

cancer
36. A 25-year-old male with a painless testicular
swelling that does not reduce on lying down and
(B) Radiotherapy given before surgery to shrink a tumor
Neoadjuvant therapy refers to treatment given before the primary surgical intervention, aiming to reduce the tumor size and improve the likelihood of complete resection. Radiotherapy administered before surgery (option B) to reduce tumor burden exemplifies this concept. Alternatively, neoadjuvant treatment can also include chemotherapy or targeted agents, but the core principle remains the same: treating the cancer prior to definitive surgery. Option (A), chemotherapy given after surgery, describes adjuvant therapy, not neoadjuvant. Supportive therapy (option C) is neither adjuvant nor neoadjuvant but rather symptomatic management. Hormonal therapy (option D) can be used in different cancer stages but does not necessarily define neoadjuvant therapy. Identifying and using correct terminology in oncology is crucial: neoadjuvant therapy can downstage tumors, potentially allowing for more conservative surgical approaches and better outcomes in certain malignancies such as rectal cancer, breast cancer, or esophageal cancer.

31. In an adult, the normal fasting serum gastrin level transilluminates is most likely:
is approximately: (A) Indirect inguinal hernia
(A) 10–40 pg/mL (B) Spermatocele
(B) 50–150 pg/mL (C) Hydrocele
(C) 500–1000 pg/mL (D) Varicocele
(D) 1500–2000 pg/mL (C) Hydrocele
A painless, cystic scrotal swelling that transilluminates on examination strongly suggests a hydrocele (option C). Hydroceles are collections of fluid within the tunica vaginalis or along the spermatic cord, generally presenting as painless swelling. They typically remain unchanged or do not reduce upon lying down. An indirect inguinal hernia (option A) may descend into the scrotum, but it often reduces or changes size with position and does not typically transilluminate. A spermatocele (option B) can also transilluminate, but it is generally located above or behind the testis, arising from the epididymal head, and is usually smaller than a typical hydrocele. A varicocele (option D) presents with a “bag of worms” feel and typically does not transilluminate; it often enlarges when standing and reduces on lying down. Hence, a painless swelling that transilluminates and persists in all positions most closely aligns with a hydrocele.
page 32 NEET PG Surgery

37. Arrange the layers of the abdominal wall from


(B) Metastatic carcinoma of the breast
A firm, non-tender, and fixed axillary lymph node in an older female is highly suspicious for malignancy, particularly metastatic breast cancer. Mammography findings of a suspicious lesion further support this possibility (option B). Many breast cancers present with axillary lymph node enlargement, sometimes even before a palpable breast mass is evident. Fibroadenoma (option A) typically presents as a mobile, painless breast lump rather than lymph node involvement. Infective lymphadenopathy (option C) could be tender or associated with signs of infection. A lipoma (option D) is usually a soft, mobile, painless lump without the characteristics of a suspicious lymph node. In this scenario, metastatic carcinoma of the breast is the most likely diagnosis, warranting further diagnostic steps such as a core needle biopsy of both the axillary node and the breast lesion to confirm malignancy and guide treatment.

superficial to deep in the region of the anterolateral 42. A 40-year-old with recurrent epigastric pain and a
abdomen: documented gastric ulcer not healing despite PPI
1. External oblique aponeurosis therapy. Biopsy shows Helicobacter pylori. The
2. Transversalis fascia next step is:
3. Skin (A) H. pylori eradication therapy
4. Internal oblique muscle (B) Immediate partial gastrectomy
5. Subcutaneous tissue (C) Surveillance endoscopy in 6 months
6. Transversus abdominis muscle (D) Increase PPI dose alone
Select the option for correct order :
(A) 3 → 5 → 1 → 4 → 6 → 2
(A) H. pylori eradication therapy
A gastric ulcer documented by endoscopy that persists despite proton pump inhibitor therapy should prompt evaluation for factors impairing healing, with Helicobacter pylori infection being a common culprit. Once H. pylori is confirmed, eradication therapy (option A) is critical for ulcer healing and preventing recurrence or complications such as bleeding or perforation. Standard triple therapy includes a PPI, clarithromycin, and amoxicillin (or metronidazole if the patient is penicillin-allergic) for 10–14 days. Immediate partial gastrectomy (option B) is reserved for refractory cases with suspicion of malignancy or complications. Surveillance endoscopy (option C) may be considered after treatment, particularly for gastric ulcers, but the first priority is eradicating H. pylori. Simply increasing the PPI dose alone (option D) neglects the key causative role of H. pylori. Proper antibiotic therapy is essential to achieve ulcer resolution and reduce the risk of gastric cancer in chronic ulcers.

(B) 3 → 1 → 5 → 4 → 2 → 6 43. Concerning complications of total parenteral


(C) 3 → 5 → 4 → 1 → 2 → 6 nutrition (TPN), which statements are correct?
(D) 3 → 4 → 1 → 5 → 6 → 2 1. Catheter-related sepsis is common
(A) 3 → 5 → 1 → 4 → 6 → 2
From the outside inward, the layers of the anterolateral abdominal wall are: Skin (3), Subcutaneous tissue (5), the External oblique aponeurosis (1), the Internal oblique muscle (4), the Transversus abdominis muscle (6), then the Transversalis fascia (2). After the transversalis fascia, one would encounter the extraperitoneal fat and the parietal peritoneum, but those are not listed in this sequence. Option (A) accurately reflects this anatomical order. External oblique becomes aponeurotic as it courses medially, lying superficial to internal oblique. Next is the transversus abdominis muscle, followed by the relatively thin transversalis fascia. Other options incorrectly rearrange these layers or omit the correct order. Recognizing the proper arrangement is vital for procedures like surgical incisions, ensuring each layer is dealt with appropriately and safely, and helps avoid inadvertent injury to underlying structures.
2. Fatty liver and cholestasis can occur
3. Hypoglycemia is a persistent problem
38. When evaluating a neck mass in an adult over 40, 4. Metabolic bone disease may develop
the most critical “key feature” to rule out is: Options :
(A) Congenital cystic lesion (A) 1 and 2 are correct
(B) Cervical lymph node metastasis (B) 1, 2, and 4 are correct
(C) Infectious lymphadenitis (C) 2 and 3 are correct
(D) Thyroid nodule (D) All are correct
(B) Cervical lymph node metastasis
Any new neck mass in an adult, especially those over age 40, must be considered malignant until proven otherwise. Therefore, excluding cervical lymph node metastasis (option B) is of utmost importance. The mass could be a metastatic deposit from a primary head and neck cancer (e.g., squamous cell carcinoma) or an occult primary tumor. While congenital cystic lesions (option A), thyroid nodules (option D), and infectious lymphadenitis (option C) are possible causes, they are less concerning than metastatic disease in this demographic. A thorough workup including imaging (ultrasound, CT scan) and tissue diagnosis via fine needle aspiration or core needle biopsy is necessary to determine the nature of the mass. Early detection and intervention for malignant causes significantly improve prognosis. Therefore, systematically ruling out metastatic lymphadenopathy is the key diagnostic step for an adult neck swelling.

(B) 1, 2, and 4 are correct


TPN is a valuable nutritional support modality but carries certain risks. Catheter-related sepsis (statement 1) is indeed common due to the indwelling central venous catheter. Fatty liver and cholestasis (statement 2) can develop from the altered metabolism associated with high glucose and lipid infusions. Metabolic bone disease (statement 4), including osteopenia and osteoporosis, may emerge with long-term TPN use due to imbalances in minerals and vitamins. However, hypoglycemia is not typically a persistent problem (statement 3); in fact, hyperglycemia is more common if TPN is not carefully monitored. While episodes of rebound hypoglycemia can occur if TPN is abruptly discontinued, it is not a continuous issue. Thus, 1, 2, and 4 are correct complications, making option (B) accurate. Carefully managing TPN protocols, strict asepsis, and routine monitoring of liver function, glucose levels, and bone density can mitigate these risks.

39. A patient with obstructive jaundice due to a 44. In a patient with an acute extradural hematoma
suspected choledocholithiasis on ultrasound (EDH) and signs of raised intracranial pressure,
should undergo which next step? the key action is:
(A) Immediate laparotomy (A) Mannitol infusion alone
(B) MRCP (Magnetic Resonance Cholangi (B) Burr hole evacuation or craniotomy
opancreatography) (C) Immediate MRI brain
(C) ERCP with therapeutic intent (D) Steroid therapy to reduce vasogenic edema
(D) HIDA scan (B) Burr hole evacuation or craniotomy
An extradural hematoma (EDH) occurs when bleeding accumulates between the skull and dura, often due to arterial rupture (commonly the middle meningeal artery). This can rapidly raise intracranial pressure. While mannitol (option A) or steroids (option D) may temporize by reducing intracranial pressure, they do not address the underlying cause: the expanding hematoma. An MRI brain (option C) can provide detailed imaging but is time-consuming. In an acute EDH with evidence of significant mass effect and raised ICP, emergent surgical evacuation of the hematoma via burr hole or craniotomy (option B) is the definitive management. Immediate decompression prevents further deterioration and potential herniation. Mannitol may be used concurrently to reduce ICP before surgery, but surgical evacuation remains the crucial step. Recognizing that EDH is a neurosurgical emergency underscores why burr hole or craniotomy is the key action to save the patient’s life and preserve neurologic function.

(B) MRCP (Magnetic Resonance Cholangio pancreatography)


When choledocholithiasis (stones in the common bile duct) is suspected in a patient with obstructive jaundice, MRCP (option B) is a noninvasive imaging modality that provides detailed visualization of the biliary tree, confirming the presence, size, and location of stones. If stones are visualized, the patient can then proceed to ERCP for endoscopic stone extraction. Although ERCP with therapeutic intent (option C) is frequently used to remove CBD stones, it is typically done after confirming the stone burden, especially when there is uncertainty about CBD stones. Immediate laparotomy (option A) is more invasive and usually not the first-line approach unless there are complicating factors like perforation or severe cholangitis that require urgent surgical intervention. A HIDA scan (option D) assesses cystic duct patency and gallbladder function, not primarily indicated for choledocholithiasis. Therefore, MRCP is the best next diagnostic step before definitive therapeutic intervention.

45. A randomized trial reports a “relative risk reduction”


40. The most definitive management for a large (RRR) of 50 for postoperative infections with the
symptomatic sliding hiatal hernia (Type I) is: use of a new antibiotic protocol. This means:
(A) Lifestyle modifications alone (A) The absolute risk of infection is reduced by
(B) Proton pump inhibitors (PPIs) indefinitely 50
(C) Laparoscopic surgical repair with (B) Half the number of patients will benefit from
fundoplication the therapy
(D) Endoscopic surveillance every 5 years (C) The risk in the treatment group is half that of
(C) Laparoscopic surgical repair with fundoplication
For large symptomatic sliding hiatal hernias (Type I) with significant reflux symptoms or complications such as esophagitis or chronic anemia from Cameron ulcers, surgical repair is the definitive treatment (option C). This typically involves hiatal closure and fundoplication (like Nissen, Toupet, or partial wrap) to restore the gastroesophageal junction anatomy and prevent reflux. Lifestyle modifications (option A) and proton pump inhibitors (option B) can improve symptoms but may not resolve mechanical factors in large hernias. Endoscopic surveillance (option D) is advised for Barrett’s esophagus or high-risk cases, but it does not address the hernia’s mechanical issues. Thus, laparoscopic repair with fundoplication remains the most definitive option, alleviating reflux and preventing further herniation. The procedure has a high success rate in properly selected patients, offering symptomatic relief and reducing the risk of complications like strangulation or severe esophagitis.

the control
(D) The difference in infection rates between
41. A 65-year-old female presents with a firm, non- groups is 50
tender, fixed axillary lymph node. She has no
palpable breast mass, but mammography shows a
(C) The risk in the treatment group is half that of the control
Relative risk reduction (RRR) describes the proportion by which the event rate is reduced in the treatment group compared to the control group. An RRR of 50 means the risk of infection in the treatment group is half that in the control group, not necessarily that 50 of all patients avoid infection. Option (A), absolute risk reduction (ARR), denotes the difference in infection rates between the two groups in percentage points, not the relative measure. Option (B) suggesting half the patients benefit is an oversimplification that confuses RRR with the absolute risk. Option (D) states the difference in infection rates is 50, which conflates absolute difference with relative difference. Properly understanding RRR is critical in evaluating new interventions. Even with a high RRR, the actual clinical impact depends on the baseline risk, emphasizing the importance of both RRR and ARR.

suspicious lesion in the upper outer quadrant. 46. In the initial trauma management (ATLS protocol),
(A) Fibroadenoma of the breast which is incorrect?
(B) Metastatic carcinoma of the breast (A) Securing the airway with cervical spine
(C) Infective lymphadenopathy protection
(D) Lipoma in the axilla (B) Breathing and ventilation assessment
MEDINK GRAND TEST 03

(C) Checking distal pulses last, after addressing (C) 1-d, 2-c, 3-a, 4-b
Insulinoma (1) classically presents with Whipple’s triad: hypoglycemia, neuroglycopenic symptoms, and relief by administration of glucose (d). Gastrinoma (2) causes Zollinger-Ellison syndrome, characterized by gastric acid hypersecretion, leading to recurrent peptic ulcers (c). VIPoma (3) leads to WDHA syndrome — watery diarrhea, hypokalemia, and achlorhydria (a). Glucagonoma (4) often presents with hyperglycemia and a distinctive rash called necrolytic migratory erythema (b). Thus, option (C) 1-d, 2-c, 3-a, 4-b accurately matches each pancreatic neuroendocrine tumor with its characteristic clinical syndrome. The correct associations are critical for diagnosing and guiding the management of these rare endocrine tumors, often requiring biochemical assays, imaging, and surgical resection for definitive treatment.

fractures
50. Assertion (A) : Bariatric surgery can cure type 2
(D) Circulation with hemorrhage control
diabetes in morbidly obese patients.
Reason (R) : Significant weight loss improves
(C) Checking distal pulses last, after addressing fractures
According to the Advanced Trauma Life Support (ATLS) protocol, the primary survey follows the ABCDE approach: Airway (with cervical spine protection), Breathing, Circulation, Disability, and Exposure. Airway management (option A) is the top priority, followed by evaluating and supporting breathing and ventilation (option B). Circulation with hemorrhage control (option D) is addressed next, ensuring major bleeding is controlled. Assessing neurovascular status, including distal pulses, should not be delayed until after fracture management; a compromised distal pulse can indicate severe vascular injury requiring urgent attention. Therefore, option (C) is the incorrect step because pulses must be checked early in the circulation assessment. Addressing fractures before confirming adequate circulation risks missing a limb-threatening vascular compromise. Timely identification of circulatory issues is essential to stabilize the patient and preserve tissue viability.

47. A 70-year-old heavy smoker has a solitary insulin sensitivity and reduces insulin resistance.
pulmonary nodule on chest X-ray. The likelihood (A) Both A and R are true, and R is the correct
of malignancy is approximately: explanation of A
(A) < 10 (B) Both A and R are true, but R is not the correct
(B) 20–30 explanation of A
(C) 40–50 (C) A is true, but R is false
(D) > 60 (D) A is false, but R is true
(A) Both A and R are true, and R is the correct explanation of A
Bariatric surgery (e.g., gastric bypass, sleeve gastrectomy) often leads to sustained weight loss and can induce complete remission of type 2 diabetes in many morbidly obese patients, effectively “curing” the disease or at least leading to long-term control without medications. Thus, the assertion (A) is true in a significant subset of patients. The mechanism is multifactorial, including an alteration in gut hormones and substantial improvement in insulin sensitivity due to reduced adipose tissue. Hence, the reason (R) that significant weight loss improves insulin sensitivity and reduces insulin resistance is a primary factor explaining how bariatric surgery resolves diabetes. Option (A) is correct because the reason directly supports the assertion. The other options imply a disconnect between the assertion and reason or that one might be false, which is incorrect. Therefore, the statement stands that successful bariatric intervention improves metabolic parameters, often leading to diabetes remission.

(D) > 60
In older patients with a significant smoking history, a solitary pulmonary nodule (SPN) on imaging carries a high likelihood of malignancy, often exceeding 60 (option D). Smoking is a well-established risk factor for lung cancer, and the incidence of malignant nodules escalates with age and smoking duration. Typically, nodules larger than 2 cm, with spiculated margins, or showing growth on serial imaging are also suspicious. Although certain benign causes such as granulomas or hamartomas exist, the pretest probability in a heavy smoker is notably high. Options (A), (B), and (C) underestimate the malignancy risk for this demographic and clinical scenario. Confirmation requires further investigation, such as CT chest characterization, PET scan, or tissue sampling. Early diagnosis is pivotal to enable potential surgical resection or other treatment modalities, which can greatly affect prognosis in lung cancer.

48. Chronic irritation by gallstones in the gallbladder 51. In a patient with a thyroid nodule, which
can lead to gallbladder cancer because: investigation is considered the most informative
(A) Gallstones produce direct carcinogens initial test?
(B) Persistent inflammation induces cellular (A) Thyroid ultrasound
dysplasia (B) Serum thyroglobulin
(C) Stones release viral oncogenes (C) Fine Needle Aspiration Cytology (FNAC)
(D) Stones cause immunosuppression in the (D) Thyroid scintigraphy (Technetium-99m scan)
gallbladder (C) Fine Needle Aspiration Cytology (FNAC)
Fine Needle Aspiration Cytology (FNAC) is the most informative initial diagnostic test for evaluating a thyroid nodule. It helps categorize nodules into benign, suspicious, or malignant, thus guiding further management. While a thyroid ultrasound (option A) is also essential for characterizing nodule size, echogenicity, and vascularity, its role is largely complementary. Serum thyroglobulin (option B) is not routinely used in the initial workup because it can be elevated in various thyroid conditions and lacks the specificity needed for a solitary nodule. Thyroid scintigraphy (option D) can be considered if the TSH level is low to differentiate “hot” from “cold” nodules, but it is not the universal first test. FNAC provides cytological details that can directly point to malignancy, colloid nodules, or suspicious lesions. Therefore, in patients with thyroid nodules, FNAC remains the key procedure to determine the need for surgery versus conservative follow-up.

52. All of the following are complications of


(B) Persistent inflammation induces cellular dysplasia
Gallstones predispose to chronic inflammation of the gallbladder wall, which over time can trigger cellular changes including dysplasia and, in some cases, malignant transformation (option B). This phenomenon is a recognized risk factor for gallbladder carcinoma. Gallstones themselves do not produce direct carcinogens (option A). Viral oncogenes (option C) are unrelated to gallstone pathogenesis. Likewise, gallstones do not cause localized immunosuppression in the gallbladder (option D). Rather, the continuous mechanical irritation and inflammation leads to repeated injury, DNA damage, and eventually neoplastic changes in susceptible cells. This cause-and-effect mechanism underscores the importance of prompt cholecystectomy for symptomatic cholelithiasis, as removing the gallbladder may reduce the long-term cancer risk in patients with chronic gallstones. Although gallbladder cancer remains relatively uncommon, chronic cholelithiasis is one of the most significant risk factors linked to its development.

49. Match the pancreatic tumors with their laparoscopic cholecystectomy except:
characteristic features: (A) Bile duct injury
(B) Subcutaneous emphysema
Column I Tumor Column II (C) Acute pancreatitis
Characteristic (D) Pulmonary embolism
Syndrome
1. Insulinoma a. WDHA syndrome
(B) Subcutaneous emphysema
Common complications of laparoscopic cholecystectomy include bile duct injury (option A), which can occur if the critical view of safety is not properly identified. Acute pancreatitis (option C) may occasionally follow instrumentation near the ampulla or due to other perioperative factors. Pulmonary embolism (option D) is a recognized complication of any surgery if prophylaxis against thromboembolism is inadequate. Subcutaneous emphysema (option B), however, is typically associated with laparoscopic procedures when the gas (CO₂) dissects into the subcutaneous tissue, but it is more accurately described as surgical emphysema rather than a recognized “complication” that causes morbidity. Mild surgical emphysema can occur transiently around port sites, but it is generally self-limiting and not considered a classic complication. By contrast, injuries to the biliary system or cardiopulmonary complications can cause significant morbidity. Hence, (B) is the best answer to identify as an “exception” among recognized laparoscopic cholecystectomy complications.

(watery diarrhea, 53. In pediatric inguinal hernias, which statement


hypokalemia, best explains why early surgical intervention is
achlorhydria) recommended?
2. Gastrinoma b. Severe (A) They are always strangulated at presentation
hyperglycemia, (B) They do not spontaneously close after infancy
necrolytic migratory (C) They typically contain omentum only
erythema (D) They resolve with conservative management
3. VIPoma c. Zollinger-Ellison in most cases